You are on page 1of 186

130

Pharmacy

685.
1.

In general gastrointestinal absorption is favoured by an empty


stomach. The absorption rate but not the extent of absorption is
reduced in presence of foods. Any factor such as food which
delays gastric emptying time will decrease the rate of absorption
of most drugs because it takes longer for the drug to reach the
small intestines where most drugs are absorbed.
A) Rate of absorption from an oily suspension.
B) Rate of absorption from an aqueous solution
A. a>b
B. a<b
C. a=b

B
The interface
between the
gastrointestinal
lumen and the
mucosa is an
aqueous phase.
Therefore, an
aqueous solution will
more easily mix with
that interface.

686.
2.

Intrathecal administration is necessary to:


A. Achieve high concentration within brain
B. Decrease systemic side effects
C. Provide spinal anesthesia to a specific nerve level
D. All of the above

687.
3.

t 1/2 is a function of
A. Distribution
B. Metabolism
C. Excretion
D. All of the above

688.
4.

Which of the following are influenced by plasma protein binding?


A. Apparent volume of distribution
B. Half - life of a drug
C. Onset of drug action
D. All of the above

689.
5.

Drugs may cross membranes by


A. Diffusion in accordance with lipid solubility of the drug
B. Filtration through aqueous pores
C. Active transport
D. All of the above

690.
6.

The main site of drug metabolism is the


A. Liver
B. Red Blood Cells
C. Small Intestine
D. Kidney

691.
7.

The oxidative metabolism of many drugs is mediated by enzymes


located in the
A. Mitochondrial Fraction
B. Lyzosomal Fraction
C. Cell Membrane
D. Microsomal Fraction

131
Pharmacy

692.
8.

Microsomal oxidation is inhibited by such agents as:


A. Erythromycin
B. Ketoconazole
C. Phenobarbitone
D. Phenytoin

A&B

693.
9.

Hydrolyis reactions are important for biotransformation of which


of the following functional groups?
A. Ethers
B. Amides
C. Sulphides
D. Esters

B&D

694.
10.

Glucuronide conjugation:
A. Occurs only in the microsomal fractions of the liver
B. Is not important for excretion
C. Is not fully developed in the neonate
D. Is induced by phenobarbitone

C&D

695.
11.

Which disease states significantly affect drug metabolism?


A Alcoholic Cirrhosis
B. Chronic Active Hepatitis
C. Glomerulonephritis
D. Aminoglycoside Toxicity

A&B

696.
12.

Which are potent inhibitors of CYP3A4 metabolism?


A. Phenytoin
B. Erythromycin
C. Ketoconazole
D. Carbamazepine

C&B

The majority of drug receptors are located on


A. Cell Surfaces
B. RNA
C. DNA
D. Mitochondria

Which of the following processes require input of energy?


A. Aqueous Filtration
B. Facilitated Diffusion
C. Lipid Diffusion
D. Active Transport

697.
13.

698.
14.

Both Active transport and


facilitated diffusion are
carrier mediated process but
only active transport is
energy dependent and will
transport substances against
an energy gradient.

132
Pharmacy

699.
17.

(A)Rapidity of absorption after oral administration


(B)Rapidity of absorption after pulmonary administration
A. a b
B. a b
C. a=b

B
Pulmonary epithelium has
very large surface area to
which drugs is exposed.
The stomach which initially
meets the drug has a small
surface area.

700.
18.

For mild symptoms of gastro-oesophageal reflux disease, initial


management include use of

A. Antacids
B. Sodium bicarbonate
C. Domperidone
D. Metoclopramide

701.
19.

The absorption of a drug is a function of


A. Physiochemical properties of the drug
B. Vehicle in which the drug is dispersed.
C. Route of administration
D. All of the above

702.
20.

The primary factor that influences the rate of absorption from


subcutaneous or intramuscular site of administration is
A. Speed of administration
B. Blood Flow
C. Volume of fluid administered
D. Diameter of the needle

Which of the following factors can decrease the rate of absorption


from a subcutaneous injection site?
A. Massage of injection site
B. Addition of epinephrine to injection
C. Application of local heat
D. All of the above

Which of the following is not acceptable for long term (days to


weeks) drug delivery system.
A. Subcutaneous Implantation of solid pellets.
B. Timed released oral prescription
C. Intramuscular injection of drug in oil suspension
D. Transdermal delivery of drug through a matrix patch

703.
21.

704.
22.

The rate of absorption from


a subcutaneous or intra
muscular site is a function
of the vasculature at that site
and the rate of blood flow
through that vasculature
system

Epinephrine
being
a
vasoconstrictor can delay
absorption.

Although Gastrointestinal
absorption can be prolonged by
enteric coating the maximum
time is still limited by transit
time through the gastrointestinal
system.

133
Pharmacy

705.
24.

706.
25.

Buccal or sublingual administration can be important for drugs


which meet which of the following criteria?
A. Drugs must be potent
B. Drug must be highly water soluble
C. Rapid effect is most desirable
D. Drug must have limited first pass effect
Which of the following factors may vary from brand name and
generic equivalent drug?
A.
B.
C.
D.

Chemical structure
Rate of disintegration
Salt form
Particle size

A&C
Sublingually administered
drugs must be very potent
and highly lipid soluble
because of the very limited
surface area of the buccal
mucosa.

B&D
It is important to know the
quality
of
generic
drug
prescribed since poor control of
tablet/capsule
production
process can lead to great
variability in the rate of
absorption and sometimes in the
total dose absorbed. This will
effect the peak and duration of
plasma concentration.

26.
707.

Intra-arterial injection is acceptable for


A. Radilogically active agent necessary to visualize specific
organs.
B. Vascular access when venous attempts area unsuccessful
C. For organ specific delivery of chemotherapeutic agents.
D. Any dug delivery when an existing arterial line is in place for
monitoring vascular status

A&C

708.
27.

Which of the following are major advantages of rectal versus oral


administration of a drug?
A. Can be given to unconscious patients
B. The absorption is more regular
C. Most of the drug does not pass through liver before entry
into the systemic circulation.
D. The action of the drug is prolonged

A&C

709.
28.

A drug may accumulate in milk if it is


A. highly ionized
B. water soluble
C. binds to calcium
D. None of the above

710.
29.

Ability of a drug to bind to a receptor is referred to as its:


A. Potency
B. Efficacy
C. Allergenicity
D. Affinity

711.
30.

Maximal effect produced by adrug is referred to as its:


A. Efficacy

134
Pharmacy

B. Antagonism
C. Potency
D. Affinity
31.
712.

32.
713.

The term often used interchangeably with intrinsic activity is:


A. Partial agonist
B. Affinity
C. Efficacy
D. Competitive antagonist
(a) Potency of a drug which requires 10 mg to elicit its maximal
response
(b) Potency of a drug which requires 100 mg to elicit its maximal
response

A. a>b
B. a<b
C. a=b
714.
33.

35.
715.

An agonist can reverse the effect of a:


A. Competitive antagonist
B. Non competitive antagonist
C. Both competitive and non competitive antagonists
D. Neither of the above

Which one of the following is TRUE for a drug whose elimination


from plasma shows first order kinetics?

A. The half life of the drug is proportional to the drug


concentration in plasma.
B. The amount eliminated per unit of time is constant.
C. The rate of elimination is proportional to the plasma
concentration.
D. Elimination involves a rate limiting enzyme reaction
operating at its maximal velocity (Vmax)
716.
36.

Diazoxide has been used by .............. route in hypertensive


emergencies
A.
B.
C.
D.

Intravenous
Intramuscular
Subcutaneous
Oral

135
Pharmacy

717.
37.

The addition of glucuronic acid to a drug:


A. Decrease its water solubility.
B. Usually leads to inactivation of the drug
C. Is an example of a phase 1 reaction
D. Occurs at the same rate in adults and newborns

718.
38.

Drugs showing zero order kinetics of elimination:


A. Are more common than those showing first order kinetics.
B. Decrease In concentration exponentially with time
C. Have a half life independent of dose.
D. Show a plot of drug concentration versus time that is linear

719.
41.

Which of the following results in a doubling of the steady state


A
concentration of a drug?
A. Doubling the rate of concentration
B. Maintaining the rate of infusion, but doubling the loading
dose.
C. Doubling the rate of infusion and doubling the concentration
of the infused drug.
D. Tripling the rate of infusion

720.
42.

Which of the following statements is NOT true of Drug Tolerance? D


A. Unusually large amount of drug is required to produce
drug effect
B. Tolerance is usually developed for Nitrates
C. Tachyphylaxis is a type of tolerance
D. Sedative hypnotic drugs do not exhibit tolerance

721.
43.

Variation in the sensitivity of a population of individuals to


increasing doses of a drug is best determined by which of the
following?
A. Efficacy
B. Potency
C. Therapeutic Index
D. Quantal dose response curve

44.
722.

Absorption of drugs from the lung is facilitated by :


A. Large surface area of the alveolar membranes
B. limited thickness of these membranes
C. High blood flow to the alveolar region
D. All of the above

136
Pharmacy

723.
45.

A patient with an acute attack of glaucoma is treated with D


pilocarpine. The primary reason for its effectiveness in this
condition is its:
A. Action to terminate acetylcholinesterase.
B. Selectivity for nicotinic receptors.
C. Ability to inhibit secretions, such as tears, saliva and sweat.
D. Ability to lower intraocular pressure.

724.
46.

A Soldier's unit has come under attack with a nerve agent. The C
symptoms exhibited are skeletal muscle paralysis, profuse
bronchial secretions, miosis, bradycardia, and convulsions. The
alarm indicates exposure to an organophosphate. What is the
correct treatment?
A. Do nothing until you can confirm the nature of the nerve
agent.
B. Administer atropine, and attempt to confirm the nature of
the nerve agent.
C. Administer atropine and 2 PAM (Pralidoxime)
D. Administer 2 PAM

47.
725.

A patient being diagnosed for myasthenia gravis would be B


expected to have improved neuromuscular function after being
treated with:
A. donezepil
B. edrophonium
C. atropine
D. isoflurophate

726.
48.

The drug of choice for treating decreased salivation accompanying D


head and neck irradiation is :
A. physostigmine
B. scopolamine
C. carbachol
D. pilocarpine

49.
727.

A 68 year old man presents to the emergency department with acute B


heart failure. You decide that this patient requires immediate drug
therapy to improve his cardiac function. Which one of the
following drugs would be most beneficial?
A. Albuterol
B. Dobutamine
C. Epinephrine
D. Norepinephrine

728.
50.

Remedies for nasal stuffiness often contain which one of the


following drugs?
A. Salbutamol
B. Atropine
C. Epinephrine
D. Phenylephrine

137
Pharmacy

729.
51.

Which one of the following drugs, when administered A


intravenously, can decrease blood flow to the skin, increase blood
flow to skeletal muscle, and increase the force and rate of cardiac
contraction?
A. Epinephrine
B. Isoproterenol
C. Norepinephrine
D. Phenylephrine

730.
53.

A 38 year old male has recently started monotherapy for mild B


hypertension. At his most recent office visit, he complains of
tiredness and not being able to complete three sets of tennis.
Which one of the following drugs is he most likely to be taking for
hypertension?
A.
B.
C.
D.

Albuterol
Atenolol
Ephedrine
Phentolamine

731.
54.

A 60 year old asthmatic man comes in for a checkup and complains A


that he is having some difficulty in "starting to urinate". Physical
examination indicates that the man has a blood pressure of
160/100 mmHg and a slightly enlarg ed prostate. Which of the
following medications would be useful in treating both of these
conditions?
A. Doxazosin
B. Labetalol
C. Phenotolamine
D. Propranolol

732.
55.

Which one of the following combinations of antiparkinson drugs B


is an appropriate therapy?
A. Amantadine, carbidopa, and entacapone
B. Levodopa, carbidopa, and entacapone
C. Pergolide, carbidopa and entacapone
D. Ropinirole, selegiline, and entacapone

733.
56.

Peripheral adverse effects of levodopa including nausea, C


hypotension, and cardiac arrhythmias, can be diminished by
including which of the following drugs in the therapy?
A. Amantadine
B. Bromocriptine
C. Carbidopa
D. Entacapone

138
Pharmacy

734.
57.

735.
58.

736.
59.

60.
737.

61.
738.

62.
739.

740.
63.

Which of the following antiparkinson drugs may cause peripheral


vasospasm?
A. Amantadine
B. Bromcriptine
C. Carbidopa
D. Entacapone
Modest improvement in the memory of patients with Alzheimer
disease may occur with drugs that increase transmission at which
of the following receptors?
A. Adrenergic
B. Cholinergic
C. Dopaminergic
D. GABAergic
Which one of the following statements is correct?
A. Benzodiazepines directly open chloride channels
B. Benzodiazepines show analgesic actions
C. Clinical improvement of anxiety requires two to four weeks
of treatment with benzodiazepines.
D. All benzodiazepines have some sedative effects.
Which one of the following is a short acting hypnotic?
A. Phenobarbitol
B. Diazepam
C. Chlordiazepoxide
D. Triazolam
Which one of the following statements is correct?
A. Phenobarbital shows analgesic properties.
B. Diazepam and Phenobarbital induce the P450 enzyme
system.
C. Phenobarbital is useful in the treatment of acute intermittent
porphyria.
D. Phenobarbital induces respiratory depression which is enhanced
by consumption of ethanol.
A 45yearold man who has been injured in a car accident is brought
into the emergency room. His blood alcohol level on admission is
275mg/dL. Hospital records show a prior hospitalization for
alcohol related seizures. His wife confirms that he has been
drinking heavily for three weeks. What treatment should be
provided to the patient if he goes into withdrawal?
A. None
B. Lorazepam
C. Pentobarbital
D. Phenytoin
A very agitated young male was brought to the emergency room
by the police. Psychiatric examination revealed that he had
snorted cocaine several times in the past few days, the last time
being ten hours previously. He was given a drug that sedated him,
and he fell asleep. The drug very likely used to counter this
patient's apparent anxiety because of cocaine withdrawal was:
A. Phenobarbital
B. Lorazepam

139
Pharmacy

64.
741.

742.
65.

743.
66.

744.
67.

745.
68.

69.
746.

747.
70.

C. Cocaine
D. Hydroxyzine
Halogenated anesthetics may produce malignant hypothermia in
A. patients with poor renal function
B. patients allergic to the anesthetic
C. pregnant women
D. patients with a genetic defect in the muscle calcium
regulation.
Children with asthma undergoing a surgical procedure are
frequently anesthetized with sevoflurane, because it:
A. is rapidly taken up
B. does not irritate the airway
C. has a low nephrotoxic potential
D. does not undergo metabolism
Which one of the following is most likely to require administration
of a muscle relaxant?
A. Ethyl ether
B. Halothane
C. Methoxyflurane
D. Nitrous oxide
Which one of the following is a potent intravenous anesthetic but a
weak analgesic?
A. Thiopental
B. Benzodiazepines
C. Ketamine
D. Etomidate
Which one of the following is a potent analgesic but a weak
anesthetic?
A. Methoxyflurane
B. Diazepam
C. Halothane
D. Nitrous oxide
55 year old man was losing interest in his work and felt worthless.
After six weeks of therapy with fluoxetine, the patient's symptoms
resolved. However the patient complaints of sexual dysfunction.
Which of the following drugs might be useful in this patient?
A. Fluvoxamine
B. Sertraline
C. Citalopram
D. Mirtazapine
A 25 year old woman had a long history of depressive symptoms
accompanied by body aches. Physical and laboratory tests were
unremarkable. Which of the following drugs might be useful in
this patient?
A. Fluoxetine
B. Sertraline
C. Phenelzine
D. Duloxetine

140
Pharmacy

71.
748.

A 51 year old woman with symptoms of major depression also has A


a narrow angle glaucoma. Which of the following antidepressants
should be avoided in this patient?
A. Amitriptyline
B. Sertraline
C. Bupropion
D. Mirtazepine

72.
749.

An adolescent male is newly diagnosed with schizophrenia.


Which of the following neuroleptic agents may improve his apathy
and blunted affect?
A. Chlorpromazine
B. Fluphenazine
C. Haloperidol
D. Risperidone

750. Which one of the following neuroleptics has been shown to be a


73.
partial agonist at the D2 receptor?
A. Aripiprazole
B. Clozapine
C. Haloperidol
D. Risperidone

751.
74.

Use of Terbutaline include all of the following EXCEPT:


A.
B.
C.
D.

752.
75.

A 28 year old woman with schizoid disorder and difficulty B


sleeping would be most benefited by which of the following
drugs?
A.
B.
C.
D.

76.
753.

Aripiprazole
Chlorpromazine
Haloperidol
Risperidone

Which are the following drugs find its use in Nocturnal enuresis:
A.
B.
C.
D.

77.
754.

Reversible airway obstruction


Premature labour
Asthma
Hypertension

Adrenaline
Atropine
Imipramine
None of the above

A heroin addict has entered a rehabilitation program that requires C


that she take methadone. Methadone is effective in this situation
because it:
A. is an antagonist at the morphine receptors

141
Pharmacy

755.
78.

756.
79.

757.
80.

81.
758.

82.
759.

83.
760.

761.
84.

B. has less potent analgesic activity than heroin


C. is longer acting than heroin; hence the withdrawal is milder
than with the latter drug.
D. Does not cause constipation
Which of the following statements about morphine is correct?
A. Its withdrawal symptoms can be relieved by naloxone.
B. It causes diarrhea.
C. It is most effective by oral administration.
D. It rapidly enters all body tissues, including the fetus of a
pregnant woman
The pain of a patient with bone cancer has been managed with a
morphine pump. However, he has become tolerant to morphine.
Which of the following might be indicated to ameliorate his pain?
A. Meperidine
B. Codeine
C. Fentanyl
D. Methadone
A nine year old boy is sent for neurologic evaluation because of
episodes of " confusion". Over the past year the child has
experienced episodes during which he develops a blank look on
his face and fails to respond to questions. However, it appears to
take several minutes before the boy recovers from the episodes.
Which one of the following describes this patient's seizures?
A. Simple partial
B. Complex partial
C. Tonic - clonic
D. Myoclonic
Which one of the following therapies would be most appropriate
in the patient described in the above question?
A. Ethosuximide
B. Carbamazepine
C. Diazepam
D. Carbamazepine plus primidone
Digitalis has profound effect on myocyte intracellular
concentrations of Na+ , K+, and Ca2+. These effects are caused by
digitalis inhibiting:
A. Ca2+ATPase of the sarcoplasmic reticulum.
B. Na+/K+ATPase of the myocyte membrane
C. Cardiac phosphodisterase
D. Cardiac 1 receptors
Compensatory increases in heart rate and rennin release that occur
in heart failure may be alleviated by which of the following drugs?
A. Milrinone
B. Digoxin
C. Dobutamine
D. Metoprolol
A 58 year old man is admitted to the hospital with acute heart
failure and pulmonary edema. Which one of the following drugs
would be most useful in treating the pulmonary edema?
A. Digoxin

142
Pharmacy

85.
762.

86.
763.

87.
764.

88.
765.

89.
766.

90.
767.

B. Dobutamine
C. Furosemide
D. Minoxidil
A 46 year old man is admitted to the emergency department. He
has taken over dose of digoxin tablets(0.25mg each),
ingesting them about three hours before admission. His pulse is
fifty to sixty beats per minute, and the electrocardiogram shows
third degree heart block. Which one of the following is the most
important therapy to initiate in this patient?
A. Digoxin immune fab
B. Potassium salts
C. Lidocaine
D. Phenytoin
A 66 year old man had a myocardial infarct. Which one of the
following would be appropriate prophylactic antiarrhythmic
therapy?
A. Lidocaine
B. Metoprolol
C. Procainamide
D. Quinidine
Suppression of arrythmias resulting from a re-entry focus is most
likely to occur if the drug :
A. has vagomimetic effects on the AV mode
B. is a Beta blocker
C. coverts a unidirectional block to a bidirectional block.
D. Slows conduction through the atria.
A 57 year old man is being treated for an atrial arrhythmia. He
complains of headache, dizziness and tinnitus. Which one of the
following antiarrhythmic drugs is the most likely cause?
A. Amiodarone
B. Procainamide
C. Propranolol
D. Quinidine
A 56 year old patient complains of chest pain following any
sustained exercise. He is diagnosed with atherosclerotic angina.
He is prescribed sublingual nitroglycerin for treatment of acute
chest pain. Which of the following adverse effects is likely to be
experienced by this patient?
A. Hypertension
B. Throbbing headache
C. Bradycardia
D. Sexual dysfunction
A 68 year old man has been successfully treated for
exercise induced angina for several years. He recently has been
complaining about being awakened at night with chest pain.
Which of the following drugs would be useful in preventing this
patient's nocturnal angina?
A. Amyl nitrate
B. Nitroglycerin (sublingual)
C. Nitroglycerin( transdermal)

143
Pharmacy

91.
768.

92.
769.

93.
770.

94.
771.

772.
95.

96.
773.

774.
97.

D. Esmolol
A 45 year old man has recently been diagnosed with hypertension
and started on monotherapy designed to reduce peripheral
resistance and prevent Na & water retention He has developed
a persistent cough. Which of the following drugs would have the
same benefits but would not cause cough?
A. Losartan
B. Nifedipine
C. Prazosin
D. Propranolol
Which one of the following drugs may cause a precipitous fall in
blood pressure and fainting on initial administration?
A. Atenolol
B. Hydrochlorothiazide
C. Nifedipine
D. Prazosin
Which one of the following antihypertensive drugs can precipitate
a hypertensive crisis following abrupt cessation of therapy?
A. Clonidine
B. Diltiazem
C. Enalapril
D. Losartan
Which one of the following is the most common side effect of
antihyperlipidemic drug therapy?
A. Elevated blood pressure
B. Gastrointestinal disturbance
C. Neurologic problems
D. Heart palpitations
Which one of the following hyperlipidemias is characterized by
elevated plasma levels of chylomicrons and has no drug therapy
available to lower the plasma lipoprotein levels?
A. Type 1
B. Type 11
C. Type 111
D. Type 1V
Which one of the following drugs decreases de novo cholesterol
synthesis by inhibiting the enzyme HMG - CoA
reductase?
A. Fenofibrate
B. Niacin
C. Cholestyramine
D. Lovastatin
Which one of the following drugs causes a decrease in liver
triacylglycerol synthesis by limiting available free fatty acids
needed as building blocks for this pathway?
A. Niacin
B. Fenofibrate
C. Cholestyramine
D. Probucol

144
Pharmacy

98.
775.

99.
776.

100.
777.

101.
778.

102.
779.

103.
780.

104.
781.

Which one of the following drugs binds bile acids in the intestine,
thus preventing their return to the liver via the enterohepatic
circulation?
A. Niacin
B. Fenofibrate
C. Cholestyramine
D. Probucol
An elderly patient with a history of heart disease and who is
having difficulty breathing is brought into the emergency room.
Examination reveals that she has pulmonary edema. Which of the
following treatments is indicated?
A. Spironolactone
B. Furosemide
C. Acetazolamide
D. Chlorthalidone
A group of college students is planning a mountain climbing trip
to the Andes. Which of the following drugs would be appropriate
for them to take to prevent mountain sickness?
A. A thiazide diuretic
B. An anticholinergic
C. A carbonic anhydrase inhibitor
D. A loop diuretic
An alcoholic male has developed hepatic cirrhosis. In order to
control the ascites and edema, he is prescribed which one of the
following?
A. Hydrochlorothiazide
B. Acetazolamide
C. Spironolactone
D. Furosemide
A55 year old male with kidney stones has been placed on a
diuretic to decrease calcium excretion. However after a few weeks
he develops an attack of gout. Which diuretic was he taking?
A. Furosemide
B. Hydrochlorothiazide
C. Spironolactone
D. Triameterene
An 75 year old woman with hypertension is being treated with a
thiazide. Her blood pressure responds and reads at 120/76mm
Hg. After several months on the medication, she complains of
being tired and weak. An analysis of the blood indicates low
values for which of the following?
A. Calcium
B. Uric acid
C. Potassium
D. Sodium
Symptoms of hyperthyroidism include all of the following EXCEPT:
A. tachycardia
B. nervousness
C. poor resistance to cold
D. body wasting

145
Pharmacy

105.
782.

Which of the following best describes the


propylthiouracil on thyroid hormone production?

effect

of

A. It blocks the release of thyrotropin releasing hormone.


B. It inhibits uptake of iodine by thyroid cells.
C. It prevents the release of thyroid hormone from
thyroglobulin.
D. It blocks iodination and coupling of tyrosines in
thyroglobulin to form thyroid hormones.
783.
106.

Hyperthyroidism can be treated by all of the following, EXCEPT?


A.
B.
C.
D.

Triiodothyronine
Surgical removal of the thyroid gland
Iodide
Propylthiouracil

784.
107.

A fifty year old woman has just diagnosed as a Type 2 diabetic D


and given a prescription for metformin. Which of the following
statements is characteristic of this medication?
A. Hypoglycemia is a common adverse effect.
B. Metformin undergoes metabolism to an active compound.
C. Many drug - drug interactions have been identified.
D. It increases peripheral glucose uptake and utilization in
target tissues

785.
108.

Which of the following statements is true for therapy with insulin D


glargine?
A. It is primarily used to control post prandial hyperglycemia
B. It should not be combined with any other insulin
C. It is now used preferentially in Type 1 diabetics who are
pregnant.
D. Pharmacokinetically there is no peak activity, and the
activity lasts about 24 hours

786.
109.

The ability to reduce insulin resistance is associated with which C


one of the following classes of hypoglycemic agents?
A. Meglitinides
B. Sulfonylureas
C. Thiazolidinediones
D. Gastrointestinal hormones

787.
110.

A 64 year old woman with a history of Type 2 diabetes is D


diagnosed with heart failure. Which of the following drugs would
be a poor choice in controlling her diabetes?
A. Metformin
B. Exenatide
C. Glyburide
D. Pioglitazone

146
Pharmacy

112.
788.

An emaciated 25 year old man with HIV is being treated with


several drugs. The rate of passive diffusion of drug molecules
through biological membranes in this patient is greater for
compounds which are:
A. insoluble in aqueous solutions
B. ionized
C. larger in molecular size
D. more lipid soluble

789.
113.

The therapeutic index is:

A.

better therapeutically, at a value of 2 than at 10

B.

determined from graded dose response


curves
the median lethal dose divided by the
median effective dose
used to determine the relative potency of
two drugs

C.
D.

114.
790.

A 40 year old man is taking oral erythromycin for the treatment


Legionnaires disease. Which factor will contribute most to
absorption variability in this patient?
A.
Daily bile output
B.
Daily urine output
C.
Drug distribution
D.
Time the drug is taken with respect to meals

115.
791.

Which characteristic is true for the active transport but not for the
facilitated diffusion of a drug across biological membranes?
A.
Exhibits
saturability
at
high
drug
concentrations
B.
Requires metabolic energy
C.
Site of drug - drug interaction
D.
Shows stereospecificity

792.
116.

Weakly acidic drugs such as barbiturates or salicylates:


A.
are solely absorbed in the stomach
B.
have a reduced rate of renal excretion
during metabolic or respiratory acidosis
C.
have increased plasma protein binding
during severe inflammation
D.
must be metabolized prior to entering the
brain

793.
117.

Which drug is probably the most safe to use in a 33 year old woman B
who is in her first trimester of pregnancy?
A.
Cyclophosphamide
B.
Folic acid
C.
Isotretinoin
D.
Lithium

147
Pharmacy

794.
119.

B
Piroxicam is a nonsteroidal antiinflammatory agent(NSAID) that
has a long shelf life and is administered as a single daily dose. It is
99% bound to plasma protein. Which renal process is most
affected by its high plasma protein binding?
A.
Active reabsorption
B.
Glomerular filtration
C.
Passive reabsorption
D.
Tubular secretion

795.
120.

Celecoxib is a selective cyclooxygenase2(COX2), NSAID


indicated to relieve the signs and symptoms of rheumatoid arthritis
and osteoarthritis. The fact that the rate of decline of this drugs
concentration in plasma follows first order kinetics means that the
drug:
A.
exhibits dose independent pharmacokinetics
B.
is cleared from plasma by a single
mechanism
C.
is eliminated by active secretion
D.
is metabolized to only one metabolite

796.
121.

Nitroglycerin is an organic nitrate used in the management of


angina pectoris. It is converted to nitric oxide (NO), a reactive
free radical, which is known to activate:
A.
adenyl cyclase
B.
beta arrestin
C.
guanylyl cyclase
D.
phospholipase C

797.
123.

The renal clearance of a drug is favored if the drug :


A.
has a high degree of binding to plasma
protein
B.
has high lipid solubility
C.
has high water solubility
D.
is acetylated

798.
124.

A 55yearold man is being treated with warfarin as an anticoagulant D


therapy when he becomes an asymptomatic carrier of Neisseria
Meningitidis for which he is treated with rifampin. His dose of
warfarin had to be increased to achieve the desired degree of
anticoagulant therapy because rifampin:
A.
decreases the absorption and, hence, the
systemic bioavailability of warfarin
B.
destroys intestinal organisms that produce
vitamin K
C.
displaces warfarin from plasma proteins
D.
induces the cytochrome P450 responsible
for the metabolism of warfarin

148
Pharmacy

799.
125.

Increased activity to Drug A is observed in a patient who has been A


on long term Drug B therapy. Drug B is an inhibitor of the process Receptor inhibition by one
drug over a period of time
stimulated by Drug A. This is an example of:
can stimulate cells to
A.
supersensitivity
generate increased numbers
B.
summation
of receptors (upregulation).
This
produces
a
C.
potentiation
CHEMICALLYINDUCED
D.
competition for receptor sites

HYPERSENSITIVITY to
other drugs which react at
the same receptor site

126.
800.

Drugs A& B both produce the same physiological effect by


different mechanisms. When given together, the observed effect is
greater than the expected sum of the combination. This is termed:
A.
supersensitivity
B.
summation
C.
potentiation
D.
synergism

801.
127.

After daily administration of a drug A for 30 days, drug B is added


which displaces drug A from the specific receptor sites. The
plasma level of drug A remains the same but the effect is
decreased. This is an example of :
A.
metabolic tolerance
B.
cellular tolerance
C.
tachyphylaxis
D.
competitive tolerance

802.
128.

Drug A has no intrinsic physiologic effect on the system in


question, but its addition to Drug B greatly increase the efficacy of
Drug B. This is an example of:
A.
supersensitivity
B.
summation
C.
potentiation
D.
competition for receptor sites

130.
803.

Hypertensive crisis in a patient taking a monoamine inhibitor for


the treatment of atypical depression:
A.
undesired expected
B.
desired excessive
C.
undesired unexpected

C
A hypertensive crisis is
NOT the DESIRED or
EXPECTED effect normally
observed when patients take
monoamine
oxidase
inhibitors. However, if the
patient
consumes
large
quantities of indirect acting
amines, such as tyramine
which is present in beers,
wines, aged cheese, etc., a
hypertensive
crisis
can
occur.

149
Pharmacy

804.
131.

Sedative effects of antihistamine in the treatment of allergies:


A.
undesired expected
B.
desired excessive
C.
undesired unexpected

805.
132.

Which of the following adverse effects could be considered as an


overextension of the pharmacological actions of the drug?
A.
Diazepam (valium) induced drowsiness
B.
Cyclophosphamide induced hemopoietic
suppression
C.
Atropine induced dry mouth
D.
All of the above

806.
133.

Which of the following adverse effects could be classified as being A & C


related to the pharmacological effects of the drug?
A.
Morphine induced respiratory depression
B.
Methotrexate induced liver toxicity
C.
Epinephrine induced arrhythmias
D.
Aminoglycoside induced nephrotoxicity

134.
807.

A 19 year old woman who is allergic to sulfa drugs was given


cotrimoxazole to treat an aerobic Gram negative infection caused
by Moraxella catarrhalis. What is the most common
manifestation of allergic response to a sulfa drug such as
sulfamethoxazole contained in cotrimoxazole?
A.
Dizziness
B.
Jaundice
C.
Loss of hair
D.
Pruritus

135.
808.

Orlistat act as a ........... to be effective in the treatment of obesity.

A.
B.
C.
D.

136.
809.

Lipase inhibitor
Appettite Suppressant
Alpha Glucosidase inhibitor
None of the above

A 45 year old man is on multiple drug therapy. Which of the


following drugs is most likely to induce microsomal enzymes?
A.
Allopurinol
B.
Cimetidine
C.
Erythromycin
D.
Phenytoin

150
Pharmacy

137.
810.

A 59 year old man with thromboem bolic disease is being treated


with warfarin. Which statement is true regarding the steadystate
concentration of drugs such as warfarin?
A.
Fluctuations are blunted by slow absorption
B.
Plateau level is directly proportional to
volume of distribution
C.
Steadystate concentrations are usually
achieved more rapidly in geriatric patients.
D.
Steadystate concentration is directly related
to clearance

138.
811.

In a controlled clinical trial of a new sedative hypnotic, what


percent of a patient population can be expected to respond to the
placebo?
A.
5 percent
B.
10 percent
C.
30 percent
D.
50 percent
The receptors for most pharmacodynamic agents (excluding
chemotherapeutic agents) are believed to reside in which cellular
structure?
A.
Cytoplasm
B.
Lysosomes
C.
Mitochondria
D.
Surface membrane
A 49 year old man with cardiovascular disease is spending one year
in malaria infested area. He is taking several drugs and develops
an immunological mediated adverse effect. Which drug is most
likely responsible for an immunological mediated adverse effect?
A.
Alphamethyldopa induced acute hemolytic
anemia
B.
Atropine induced dry mouth
C.
Diazepam induced sedation
D.
Primaquine induced acute hemolytic anemia
A new born child has moderate Crigler Najjar syndrome type 11.
The partial deficiency of hepatic microsomal enzyme glucuronyl
transferase activity in this child may cause:
A.
a decrease in serum unconjugated bilirubin
levels
B.
an increase in the excretion of unconjugated
bilirubin in bile and urine
C.
increased availability of plasma protein
binding sites for acidic drugs such as
sulfonamides
D.
increased clearance of acetaminophen
Which of the following statements is true regarding drug
metabolism?
A.
Conjugation reactions always increases the water
solubility of a drug

812.
139.

140.
813.

141.
814.

142.
815.

151
Pharmacy

B.
C.
D.
816.
143.

144.
817.

145.
818.

819.
146.

820.
147.

821.
148.

149.
822.

Ester linkages are usually more rapidly


hydrolyzed than amide linkages
Glucuronides are never eliminated by
secretion in urine or bile
Sulfates often form insoluble crystals in the
kidney

The dose response curve used to calculate LD 50 of a drug is an


example of which type of response?
A.
Allergic
B.
Efficacy
C.
Idiosyncratic
D.
Quantal
A 23 year old woman presents with a urinary infection caused by
Pseudomonas aeruginosa. Her primary care physician prescribed
an oral penicillin. Which of the penicillins below would be most
appropriate?
A.
Ampicillin
B.
Benzathine penicillin G
C.
Carbenicillin indanyl sodium
D.
Penicillin G
Procaine is a short acting local anesthetic of the ester type used for
local and regional anesthesia. What is the major mode of
biotransformation for procaine?
A.
Acetylation
B.
Hydrolysis
C.
Methylation
D.
Oxidation
Which of the following increases the halflife of a drug?
A.
Enterohepatic circulation
B.
Firstpass metabolism
C.
High water solubility
D.
Increased volume of distribution
Intracellular calcium signals are mediated by:
A.
adenylyl cyclase
B.
guanylyl cyclase
C.
polyphosphoinositide signals
D.
receptor operated calcium channels
Which of the following adverse effects is considered to be an
idiosyncratic reaction?
A.
Atropine induced dry mouth
B.
Isoniazid induced seizures
C.
Methyldopa induced hemolytic anemia
D.
Primaquine induced acute hemolytic anemia
Plasma protein binding of drugs:
A.
decreases the dose required
B.
decreases the halflife of the drug
C.
decreases volume of distribution
D.
increases in geriatric patients

A&D

C&D

152
Pharmacy

150.
823.

151.
824.

152.
825.

153.
826.

154.
827.

155.
828.

156.
829.

Which of the following is the best example of pharmacokinetic


tolerance?
A.
Decreased anticoagulant effect of warfarin
in epileptics treated with phenytoin
B.
Decreased effect of morphin after repeated
administration
C.
Reduced sedation with daily benzodiazepine
treatment
D.
Staphylococcal resistance to penicillin
Which of the following is a good indicator of safety for a
therapeutic agent?
A.
High therapeutic index
B.
Low potency
C.
Low therapeutic index
D.
Rapid metabolism
Decreased response to successive equal doses of a drug given to a
test subject during a short period of time is most accurately
defined as:
A.
anaphylaxis
B.
antagonism
C.
idiosyncrasy
D.
tachyphylaxis
Which of the following terms describes a drug that binds to
a receptor but does not produce any direct pharmacological
response?
A.
Agonist
B.
Antagonist
C.
Inverse agonist
D.
Partial agonist
Which of the following drug side effect pairing is mediated by an
allergic response?
A.
Acetaminophen induced liver toxicity
B.
Barbiturate induced porphyria
C.
Penicillin induced hemolytic anemia
D.
Primaquine induced hemolytic anemia
Hypersensitivity reactions, from a benign rash to anaphylaxis, are
among the most frequent adverse reactions to the penicillins. The
target organ of a type 1 immune response (anaphylactic response)
responsible for shock is :
A.
CNS
B.
Kidney
C.
Vasculature
D.
Lungs
An understanding of how isoniazid is metabolized is important
because:
A.
fast acetylators have a greater incidence
of aplastic anemia
B.
fast acetylators often experience
hyperbillirubinemia
C.
slow acetylators are resistant to its

153
Pharmacy

D.

830.
157.

antimicobial effects
slow acetylators have a greater incidence
of peripheral neuropathy

Which of the following is true for atropine, a competitive


antagonist at muscarinic receptors, with respect to its effect on the
response to an agonist that acts on the same receptor?
A.
B.
C.
D.

Blockade overcome by a sufficient dose of


agonist
Intrinsic activity equal to 1.0
Intrinsic activity is negative
Irreversible inhibitor of receptor

831.
159.

Which of the following antimicrobial agents exert their action by


interfering with bacterial protein synthesis?
A.
Chloramphenicol
B.
Tetracycline
C.
Erythromycin
D.
Polymyxin

A,B&C

832.
160.

Which of the following drugs act on the cell membrane of


invading organisms?
A.
Amphotericin
B.
Nystatin
C.
Polymyxin
D.
Penicillin

A,B &C

833.
161.

Which of the following is an inhibitor of cell wall synthesis in


cocci?
A.
Allopurinol
B.
Azathioprine
C.
Cefazolin
D.
None of the above

834.
162.

Which of the following might you avoid or use in lower doses in


people with reduced liver function?
A.
Chloramphenicol
B.
Aminoglycosides
C.
Erythromycin
D.
Penicillin

A&C

835.
163.

Dose must be adjusted in patients with renal failure:


A.
Cefuroxime
B.
Gentamicin
C.
Amikacin
D.
All of the above

154
Pharmacy

164.
836.

165.
837.

838.
166.

A 57 year old woman has rheumatic disease that is unresponsive


to NSAIDs. Which of the following is effective in rheumatic
disease that does not respond to treatment with cyclooxygenase
inhibitors?
A.
Allopurinol
B.
Colchicine
C.
Ibuprofen
D.
Organic gold
Which of the following anticoagulants is/are contradicted in a
27 year old woman who is 4 months pregnant?
A.
Heparin
B.
Warfarin
C.
Both A & B
D.
Neither A & B

Deficiency of cobalamin (Vitamin B12):


A.
interferes with DNA synthesis in
rapidly growing cells

B.
C.
D.

leads to degeneration of the spinal cord


causes megaloblastic anemia
All of the above

167.
839.

Which of the following is used for uriocosuric action in a


40 year old man with gout?
A.
Acetaminophen
B.
Allopurinol
C.
Aspirin
D.
Sulfinpyrazone

168.
840.

Drugs with clinically useful analgesic and antiinflammatory


actions include:
A.
acetaminophen
B.
ibuprofen
C.
propoxyphene
D.
aspirin

B&D

841.
169.

Which of the following are effective for the dissolution of


established thrombi:
A.
streptokinase
B.
urokinase
C.
alteplase
D.
All of the above

842.
171.

A 74 year rold man was given long term therapy with heparin for the A&B
prevention of venous thromboembolism. Long term therapy with
heparin requires:
A. monitoring of the partial thromboplastin
time
B. frequent platelet counts
C. monitoring of the Quick prothrombin time

155
Pharmacy

843.
172.

A 52 year old man has acute gouty arthritis. Colchicine, the preferred agent
in the treatment of acute gouty arthiritis, is effective in the treatment of
gout because it casuses:
A. xanthine oxidase inhibition
B. uricosuria
C. decreased urinary excretion of uric acid
D. decrease phagocytic action of neutrophils
in joint fluid

173.
844.

A 62 year old man is on warfarin therapy to treat and further


prevent the development of thromboembolic disease. Which of
the following drugs may decrease the effects of oral anticoagulant
agents when taken concurrently?
A. Ibuprofen
B. Cimetidine
C. Aspirin
D. Cholestyramine
Oral anticoagulant drugs e.g. warfarin, are:
A. displaced from plasma protein binding sites by salicylates
B. metabolized more rapidly when Phenobarbital is administered
simultaneously
C. both A & B
D. neither A nor B

846.
175.

A 49 year old man is given sulfinpyrazone for the treatment of


chronic gouty arthritis because it is a(n):
A. analgesic agent
B. antipyretic agent
C. antiinflammatory agent
D. uricosuric agent

847.
176.

Symptoms of folic acid deficiency may be toxic side reactions of


the following drugs:
A. pyrimethamine, primaquine, quinine
B. methotrexate, trimethoprim, pyrimethamine
C. hydralazine, isoniazid, quinacrine
D. procainamide, hydralazine, trimethoprim
The following are effective anticoagulant in vitro EXCEPT:
A. warfarin sodium
B. heparin
C. EDTA
D. Oxalate
Intrinsic factor is:
A. a glycoprotein secreted by the parietal
cells of the stomach in humans
B. a polypeptide secreted by the pancreas and
necessary for the absorption of vitamin B12
C. a glycoprotein secreted into the bile that
aids in the absorption of vitamin B12 in the colon
D. necessary for the absorption of both folic
acid and vitamin B12

845.
174.

848.
177.

178.
849.

156
Pharmacy

850.
180.

851.
181.

Warfarin is most likely to cause:


A. damage to intestines, liver and heart
B. massive hemorrhage
C. photosensitive keratitis
D. pulmonary embolism
Tetracycline:
A. binds to bacterial 30S ribosomal subunit
B. binds to bacterial 50Sribosomal subunit
C. inhibits bacterial peptidoglycan synthetase
D. inhibits bacterial protein synthesis

A&D

852.
182.

Tetracyclines should not be administered to pregnant women(in 3rd


semester because tetracyclines may produce discolored teeth in the
offspring of pregnant women.
A. True
B. False

183.
853.

Retards bone growth:


A. chloramphenicol
B. tetracyclines
C. both A & B
D. neither A nor B

854.
184.

One of the following drugs is sometimes used for the prophylaxis


of migraine

A.
B.
C.
D.
855.
186.

Aspirin
Propranolol
Carbamazepine
Domperidone

Which of the following factors contribute to the gastrointestinal


side effects of the tetracyclines?
A. Superinfection, the destruction of the
normal bacterial flora, permitting the
overgrowth of pathogens
B. The poor absorption of these antibiotics
from the gastrointestinal tract
C. Their irritant effects on the gastrointestinal
mucosa
D. All of the above

157
Pharmacy

856.
187.

Which of these tetracyclines is most frequently associated with severe B


phototoxic reactions?
A. Chlortetracycline
B. Demeclocycline
C. Doxycycline
D. Oxytetracycline

857.
188.

The drug of choice for the treatment of Rocky Mountain spotter


fever in a 31 year old man is:
A. ampicillin
B. chloroamphenicol
C. doxycycline
D. gentamicin

858.
189.

Tetracyclines are useful for the treatment of which of the


following
A. anaerobic infections
B. gonococcal infections
C. mycoplasmal pneumonia
D. Rickettsial diseases

B,C&D

859.
190.

Which of the following are common side effects of tetracyclines?


A. Nausea and vomiting
B. Hearing loss
C. Superinfection with Candida albicans
D. Hypersensitivity

A&C

860.
191.

Which of the following drugs is usually given intravenously for


the treatment of systemic fungal infections?
A. Amphotericin B
B. Bacitracin
C. Flucytosine
D. Griseofulvin

861.
193.

As used, frequently produces decreased renal function:


A. amphotericin B
B. nystatin
C. both A & B
D. neither A nor B

862.
194.

Which of the following drugs is used clinically as a systemic


broad spectrum antifungal agent?
A. Amphotericin B
B. Itraconazole
C. Both A & B
D. Neither A nor B

158
Pharmacy

203.
863.

Iron is stored in the body :


A.
mainly as transferrin
B.
mainly in the parenchymal cells of the liver
C.
tightly bound to ferritin from which it can
be released only by oxidixing agents
D.
in significant amounts in hemoglobin
The most common side effect of oral iron therapy, G.I.symptoms,
depends on the amount of:
A.
iron absorbed
B.
soluble, elemental iron per dose
C.
ironphytate formed
D.
iron already in the bodys tissue stores
Which of the following symptoms may occur in a 77 year old with
vitamin B12 deficiency?
A.
Ineffective hematopoiesis
B.
Inadequate maintenance of epithelial cells
of gastrointestinal tract
C.
Numbness and tingling in the hands and feet
D.
All of the above

866.
206.

Which type of anemia is most likely to occur in a 27 year old man


with major acute blood loss from a gunshot wound?
A.
Aplastic anemia
B.
Macrocytic anemia
C.
Microcytic anemia
D.
Normocytic anemia

867.
207.

A 4 year old child swallowed twenty five 325mg aspirin tablets.


Poisoning with aspirin is associated with which of the following
symptoms:
A.
dehydration
B.
hypothermia
C.
oliguria
D.
All of the above

868.
208.

Which of the following agents prevent platelet aggregation:


A.
aspirin
B.
indomethacin
C.
naproxen
D.
All of the above
Megaloblastic anemia can develop as a consequence of prolonged
therapy with which of the following:
A.
phenytoin
B.
primidone
C.
phenobarbital
D.
All of the above
Transferrin:
A.
is one of the storage forms of iron
B.
is a betaglobulin in the plasma
C.
transports iron in the ferrous form
D.
is a coenzyme of the flavoprotein series

204.
864.

205.
865.

209.
869.

210.
870.

159
Pharmacy

871.
195.

196.
872.

873.
197.

874.
198.

875.
199.

876.
200.

201.
877.

202.
878.

(A) absorption of amphotericin B from the GI tract


(B) absorption of flucytosine from the GI tract
A.
A>B
B.
B>A
C.
A=B
Which of the following antifungal agents would be only given
orally for the treatment of dermatomycosis in a 31 year old man?
A. Amphotericin B
B. Flucytosine
C. Griseofulvin
D. Miconazole
A drug used for the oral treatment of dermatophytosis of toenails,
fingernails and hair bulb is:
A. fluconazole
B. griseofulvin
C. ketoconazole
D. nystatin
Which of the following oral medications could be used in a
31yearold woman to treat vulvovaginal candidiasis?
A. Butoconazole
B. Fluconazole
C. Nystatin
D. Terconazole
Which of the following are clinical uses of metronidazole?
A.Trichomoniasis
B. Giardia lamblia
C. Intestinal amebiasis
D. All of the above
What are the side effects of metronidazole?
A.Disulfiram like effect
B. Metallic taste
C. Mutagenic in rodents and the Ames test
D.All of the above
Ferrous sulfate is valuable for the treatmentof Which of the
following disorders in a 26 year old pregnant woman?
A.Pernicious anemia
B. Macrocytic anemia
C. Hypochromic anemia
D.Trifacial neuralgia
When iron is ingested:
A.only that portion in the form of hemoglobin is
absorbed
B. as inorganic iron salts, it must be reduced to the
divalent (ferrous ) form before absorption
C. about 50 percent of the amount ingested is
absorbed
D.a smaller percentage of the dose is absorbed by
the anemic individual

160
Pharmacy

211.
879.

880.
212.

881.
213.

214.
882.

883.
215.

884.
216.

217.
885.

Macrocytic hypochromic anemia usually responds to :


A.
cyanocobalamin
B.
folic acid
C.
deferoxamine
D.
ferrous sulfate
Which of the following substances will increase the absorption of
inorganic iron?
A.
Nicotinic acid
B.
Pyridoxine
C.
Paraamino benzoic acid (PABA)
D.
Ascorbic acid
Which of the following drugs would enhance the production of
both granulocytes and monocytes in a 46 year old woman with
Hodgkins disease who is undergoing a bone marrow transplant?
A.
Anakinra
B.
Epoetin alfa
C.
Filgrastim
D.
Sargramostim
A 25 year old women who was given an injection of a drug to a
correct anemia almost immediately went into anaphylactic shock.
Which of the following drugs would be most likely responsible for
producing anaphylactic shock?
A. Epoetin alfa
B. Filgrastim
C. Iron dextran
D. Oprelvekin
Mode of antithrombotic action is to inhibit the platelet release
reaction:
A. aspirin
B. tissue plasminogen activator (tPA)
C. both A & B
D. neither A nor B
A 49 year old woman has been suffering from morning stiffness
that persists for several hours, anorexia, fatigue and generalized
muscle and joint pain for the past 6month. She is no longer
capable of wearing rings because of finger swelling. Physical
examination and laboratory findings are consistent with a
diagnosis of rheumatoid arthritis. A drug of first choice in the
treatment of rheumatoid arthritis is :
A. acetaminophen
B. allopurinol
C. aspirin
D. colchicine
A 45year old man took several tablets of an OTC analgesic prior
to going to bed after a heavy bout of drinking. The next morning
he had severe nausea and abdominal tenderness. He went to the
ER and laboratory analysis reveled marked elevations of liver
enzymes and liver biopsy showed hepatocellular swelling and
necrosis. Overdose of which analgesic is most likely to produce

D
Sargramostim
is
a
recombinant
human
granulocytemacrophage
colonystimulating factor. It is
used to reduce the duration of
neutropenia and incidence of
infection in patients receiving
myelosuppressive
chemotherapy or bone marrow
transplantation
C

Iron dextran can cause a


sudden onset of dyspnea and /or
anaphylactic shock.

161
Pharmacy

218.
886.

887.
219.

888.
220.

221.
889.

890.
222.

891.
223.

224.
892.

centrionular necrosis in the liver:


A. acetaminophen
B. aspirin
C. naproxen
D. piroxicam
A 36 year old woman with a history of peptic ulcer disease came
home from work with high fever. Which of the following is
most appropriate for the relief of fever in this patient with a history
of peptic ulcer disease?
A. Acetaminophen
B. Acetylsalicylic acid
C. Codein
D. Rofecoxib
Which of the following is an appropriate antidote for a 3year old
child who has consumed several pills of his mothers medication
to treat hypochromic microcytic anemia?
A. Ascorbic acid
B. Deferoxamine
C. Ferritin
D. Protamine sulfate
Administered orally:
A. heparin
B. warfarin
C. both A & B
D. neither A nor B
Agent(s) which decreases(S) the host defense mechanism against
microbial organisms:
A. Vinblastine
B. Prednisone
C. Azathioprine (Imuran)
D. All of the above
Therapeutically valid reason(s) for using combinations of
antibiotic agents is (are):
A. to enhance therapeutic activity (synergistic effect)
B. to delay the emergence of resistant organisms
C. to treat a mixed infection which does not respond to
a single agent
D. All of the above
Dental mottling is an adverse reaction that has been reported in the
offspring of pregnant women treated during the last timester with:
A. ampicillin
B. azithromycin
C. gentamicin
D. minocycline
Which of the following statements is true for
phenoxymethylpenicillin (Penicillin V) but is not true for
benzathine penicillin G?
A. Broad chemotherapy spectrum
B. Excretion antagonized by probenecid
C. Given orally

162
Pharmacy

225.
893.

D. Penicillinase resistant
A 43 year old man with staphlyococcal septicemia who is allergic
to ampicillin and cephalothin and whose organisms are resistant to
doxycycline and erythromycin could be treated with:
A. Cefixime
B. Clarithromycin
C. Piperacillin
D. Vancomycin

894.
226.

Which of the following drugs has the best chemotherapeutic


index?
A. Amikacin
B. Ampicillin
C. Ciprofloxacin
D. Cyclophosphamide

227.
895.

Which of the following is most appropriate for a radical cure of


vivax malaria?
A. Chloroquine
B. Chloroquine plus primaquine
C. Fansidar
D. Mefloquine plus chloroquine

896.
229.

Penetration into cerebrospinal fluid when the meninges are


inflamed:
A. penicillins
B. sulfonamides
C. both A & B
D. neither A nor B

230.
897.

Sulfamethoxazole acts by:


A. alteration of function of plasma membrane
B. competitive antagonism of dihydrofolic acid
synthesis
C. inhibition of DNA synthesis at the replication stage
D. inhibition of protein synthesis at translation stage

898.
231.

Competitive inhibition is thought to be the mechanism by which


procaine interferes with the chemotherapeutic effect of:
A. ampicillin
B. chloramphenicol
C. chlortetracycline
D. sulfamethoxazole

232.
899.

The frequency of development of bacterial resistance to the


trimethoprim sulfamethoxazole combination is lower that it is to
either of the agents alone because microorganisms which acquire
resistance to one agent in a combination may still be susceptible to
the effects of the other agent.
A. True
B. False

163
Pharmacy

900.
233.

Crystalluria in an acid urine is a probable toxic effect of :


A. para aminosaliccylic acid
B. gentamicin
C. penicillin G, Sodium
D. sulfadiazine

A&B

901.
234.

Possible side effects of sulfonamide therapy:


A. Hemolytic anemia
B. Hepatitis
C. Stevens Johnson syndrome
D. All of the above

235.
902.

A 36 year old African/American man who is deficient in


glucose 6 phosphate dehydrogenase may develop acute hemolytic
anemia when given:
A. primaquine
B. sulfisoxazole
C. tetracycline
D. trimethoprim

A&B

903.
236.

A 31 year old woman with complaints of dysuria. Frequency and


suprapubic pain was diagnosed with cystitis (lower urinary tract
infection) caused by E.Coli. Which of the following drugs would
be most appropriate for the treatment of this condition?
A. Mafenide
B. Sulfacetamide
C. Sulfadiazine
D. Sulfisoxazole

904.
237.

Which of the following agents would be used in combination with


a sulfonamide to treat a sinus infection caused by Moraxella
catarrhalis in a 52 year old man?
A. Methotrexate
B. Phenazopyridine
C. Pyrimethamine
D. Trimethoprim
A 26 year old man is infected with HIV and his CD4+cells/mm3 is
less than 200. Which drug is appropriate to use for primary
pneumocystis carini prophylaxis in HIVinfected individuals?
A. Aztrenam
B. Ciprofloxacin
C. Cotrimoxazole
D. Sulafacetamide
Which of the following DNA gyrase inhibitors could be given
once a day for the treatment of acute bacterial exacerbations of
chronic bronchitis in 15 year old boy?
A. Cinoxacin
B. Ciprofloxacin
C. Nalidixic acid
D. Sparofloxacin

905.
238.

906.
239.

164
Pharmacy

240.
907.

908.
241.

242.
909.

910.
243.

911.
244.

912.
245.

246.
913.

247.
914.

915.
248.

Renal excretion is slowed by concomitant administration of


probenecid:
A. pencillins
B. aminoglycosides
C. both A & B
D. neither A nor B
Renal damage is a frequent side effect in the parenteral use of:
A. amphotericin B
B. erythromycin
C. nalidixic acid
D. tetracyclines
Many antibiotics appear to have their mechanism of action as capability to inhibit
bacterial cell wall synthesis. This does NOT appear to be a mechanism of:
A. Gentamicin
B. Penicillin
C. Bacitracin
D. Cephalexin
A favorable chemotherapeutic index could arise as a result of :
A. proportionately increased toxicity to parasite cells
B. proportionately increased toxicity to host cells
C. proportionately diminished toxicity to host cells
D. proportionately diminished toxicity to parasite cells
Which of the following drugs can be given to patients with renal
dysfunction at the same maintenance dose that is used in patients
with normal renal function?
A. Streptomycin
B. Erythromycin
C. Gentamicin
D. All of the above
Which one of the following class of antimicrobial agents is
classified as being bacteriostatic?
A. Aminoglycosides
B. Cephalpsporins
C. Fluroquinolones
D. Tetracycline
Which of the following drugs will readily penetrate the blood brain
barrier making them useful in treating infections of the central
nervous system?
A. Ampicillin
B. Sulfadiazine
C. Gentamicin
D. Chloramphenicol
Of the drugs listed below, which has the broadest antibiotic
spectrum?
A. Amikacin
B. Cefazolin
C. Doxycycline
D. Erythromycin
Which of the following agents is administered by injection for
treatment of systemic infections?

A&C

B&D

165
Pharmacy

249.
916.

250.
917.

918.
251.

252.
919.

253.
920.

254.
921.

255.
922.

256.
923.

A. Doxycycline
B. Gentamicin
C. Isoniazid
D. Linezolid
Which of the following produces its therapeutic effect by lowering
intracellular levels of folinic acid?
A. Sulfadiazine
B. Methotrexate
C. Paraaminosalicylic acid(PSA)
D. All of the above
(A) Gastrointestinal absorption of phenoxymethyl penicillin
(penicillin V)
(B) Gastrointestinal absorption of bacitracin
A. A>B
B. B>A
C. A=B
A 9 year old boy has been admitted to the hospital with a 7 day
history of localized pain and swelling in his lower right leg and a 5
day history of fever. Laboratory results indicate that he has
osteomyelitis caused by staphylococcus aureus. Which of the
antibiotics listed below would be appropriate?
A. ampicillin
B. ceftriaxone
C. clindamycin
D. gentamicin
Which of the following antibiotics is active against gram positive
organism and would be safe to be administered at full doses to a
69 year old woman with renal insufficiency?
A. Cefazolin
B. Erythromycin
C. Gentamicin
D. Nalidixic acid
Vanomycin inhibits:
A. cell wall synthesis
B. folic acid synthesis
C. neuromuscular transmission
D. nucleic acid synthesis
Penicillinase resistant and effective following oral administration:
A. ampicillin
B. oxacillin
C. penicillin G
D. ticarcillin
Which of the following drugs is similar to penicillin in spectrum
and is also destroyed by gastric acid?
A. Clindamycin
B. Chloramphenicol
C. Cycloserine
D. Erythromycin base
Which of the following microorganisms is most sensitive to
penicillin G?

166
Pharmacy

924.
257.

925.
258.

926.
259.

927.
260.

928.
261.

262.
929.

930.
263.

A. betahemolytic streptococci
B. klebsiella penumoniae
C. proteus mirabilis
D. staphylococcus aureus
A 67 year old woman with a drug resistant hospital acquired
infection was given a drug that is given in combination with
cilastatin. Which of the following drugs was given to the patient?
A. Aztreonam
B. Fosfomycin
C. Ertapenem
D. Imipenem
Penicillin has little or no bactericidal action against:
A. actively growing bacterial cells
B. meningococci
C. resting bacterial cells
D. treponema pallidum
A 22 year old woman presented with complaints of vaginal
discharge, dysuria, abdominal pain and fever. She was diagnosed
to have an acute gonococcal infection and was treated with a
third generation cephalosporin which was most likely:
A. aztreonam
B. cefepime
C. ceftriaxone
D. lomefloxacin
A 14 year old girl with mononucleosis was given an oral antibiotic
for a respiratory infection. Shortly after taking the antibiotic she
developed a severe skin rash. Which one of the following
antibiotics has produced a higher incidence of skin rash than any
of the others listed?
A. Ampicillin
B. Azithromycin
C. Cephalexin
D. Ciprofloxacin
A 21 year old man was diagnosed with meningitis caused by
gram negative bacteria. Which of the following cephalosporins are
most appropriate for this patient?
A. Cefotaxime
B. Ceftazidime
C. Ceftizoxime
D. All of the above
A 39 year old woman was diagnosed with a penicillin sensitive
enterococcal infection, however , she has a past history of
penicillin allergy. Which of the following statements are true
regarding hypersensitivity reactions to penicillin G?
A. crossreaction occurs towards all penicillins
B. have been fatal following oral administration
C. occur least frequently with oral administration
D. All of the above
A 37 year old woman with cystic fibrosis developed an infection
caused by pseudomonas aeruginosa. Which of the following

B&D

167
Pharmacy

931.
264.

932.
265.

266.
933.

267.
934.

935.
268.

936.
269.

937.
270.

cephalosporins would be appropriate for this patient?


A. Cefazolin
B. Cefoperazone
C. Cefoxitin
D. Ceftazidime
A 69 year old woman has a modest reduction in renal function.
Which of the following antibiotics would have the lowest
incidence of toxicity when used to treat a severe urinary tract
infection caused by susceptible microorganisms in this patient?
A. Bacitracin
B. Carbenicillin, indanyl sodium
C. Gentamicin
D. Polymixin
Which of the following antibiotics are more active against gram
negative bacteria than cephalexin?
A. Cefaclor
B. Cefepime
C. Cefoxitin
D. All of the above
Which of the following penicillins are sensitive to betalactamase
and active against Gram negative organisms?
A. Ampicillin
B. Nafcillin
C. Penicillin G
D. Ticarcillin
Which of the following cephalosporins would provide good
coverage against B. fragillis in a 49 year old woman who recently
had a surgical procedure?
A. Cefadroxil
B. Cefazolin
C. Cefoxitin
D. Ceftizoxime
Penicillins which are resistant to bacterial penicillinases include:
A. ampicillin
B. benzathine penicillin G
C. methicillin
D. oxacillin
Cephalosporinase resistant:
A. cefaclor
B. cefazolin
C. cefepime
D. cefoxitin
A 25 year old man with AIDS is given penicillin G for the
treatment of a respiratory infection caused by streptococcus
pneumoniae. It is important to use a bactericidal agent in this
patient because of his reduced immune competence. Penicillin s
are bactericidal because they affect the bacteria by:
A. altering nucleic acid metabolism
B. competitive antagonism of paraaminobenzoic acid
(PABA)

A&D

C&D

C&D

C&D

168
Pharmacy

271.
938.

272.
939.

273.
940.

274.
941.

275.
942.

276.
943.

277.
944.

C. increasing cell membrane permeability


D. preventing formation of or destroying the bacterial
cell wall
Which of the following agents is least likely to produce an allergic
response in a 35 year old woman with documented anaphylaxis to
benzyl penicillin G?
A. ampicillin
B. benzathine penicillin G
C. carbenicillin indanyl
D. cefaclor
A 27 year old man presented to the free medical clinic and was
diagnosed with primary syphilis. He was given IM injection of a
repository form penicillin. Which of the following penicillin
preparations has the longest duration of action?
A. Amoxicillin
B. Benzathine penicillin G
C. Carbenicillin indanyl sodium
D. Oxacillin
Which of the following antibiotics most closely resembles the
inhibition of cell wall synthesis by penicillin?
A. Amikacin
B. Bacitracin
C. Cefazolin
D. Clindamycin
A 45 year old man is given ampicillin and probenecid, as a
combined oral preparation, to treat an uncomplicated Neisseria
gonorrhoeae infection. He also has occasional bouts of gouty
arthiritis and frequently takes NSAIDS for arthritic pain.
Probenecid may significantly alter the renal excretion of :
A. ampicillin
B. ketoprofen
C. uric acid
D. All of the above
Which of the following cephalosporins is effective when
administered orally?
A. Cefaclor
B. Cefepime
C. Ceftriaxone
D. Cephalexin
A 75 year old man with reduced renal function is being treated for
an enterococcal infection with penicillin G. Penicillins action is
mainly terminated by :
A. enzymatic hydrolysis in the serum
B. glomerular filtration
C. metabolism in the liver
D. tubular secretion in the kidney
A 59 year old , 90kg, man with complaints of fatigue, a persistent
low grade fever, night sweats, arthralgias and a recent
unintentional 8 kg weight loss was admitted to the hospital.
Physical examination indicated a grade IV diastolic murmur with

A&D

169
Pharmacy

278.
945.

279.
946.

280.
947.

281.
948.

282.
949.

950.
284.

285.
951.

286.
952.

mitral regurgitation(insufficiency) and a temperature of 101F.


After appropriate workup a diagnosis of bacterial endocarditis
caused by streptococcus viridans was made. Which of the
following is most appropriate treatment for this patient?
A. Ampicillin (IV)
B. Cefazolin(IV)
C. Penicillin G(IV)
D. Ticarcillin (IV)
C
Which penicillin is least inactivated when given orally?
A. Methicillin
B. Nafcillin
C. Oxacillin
D. Penicillin G
The limitations of penicillin G include:
D
A. hypersensitization
B. instability in normal gastric juices
C. rapid renal excretion
D. All of the above
What is the effect of Atropine on respiratory tract secretion?
B
A. Increase
B. Decrease
C. No change
Which one of the following agents are used in some cough syrups? B
A. Amphetamine
B. Ephedrine
C. Methoxamine
D. All of the above
All of the following drugs are used the management of generalised D
seizures EXCEPT:
A. Sodium Valproate
B. Lamotrigene
C. Carbamazepne
D. Ethosuximide
Primary objective in the treatment of glaucoma is :
D
A. Increasing the convexity of lens
B. Dilation of vessels in the eye
C. Dilation of pupil
D. Reducing intraoccular pressure.
Which of the following is not a substrate of ACh esterase but
C
binds only to the anionic site and not the esteratic site of the enzyme:
A. Physostigmine
B. Neostigmine
C. Edrophonium
D. None of the above
Which of the following is not a characteristic of the sympathetic
system?
A. The same ganglionic transmitter as the parasympathetic
B. ganglia located close to the CNS
C. blocked by the same drugs as the parasympathetic
D. Thoracolumbar origin

170
Pharmacy

953.
287.

What class of cholinergics does neostigmine belong to:


A. Cholinomimetic Alkaloid
B. Alkaloid
C. Cholinergic blocker
D. Anticholinesterase Inhibitor

954.
288.

Most sweat glands are innervated by :


A. Adrenergic parasympathetic nerves
B. Adrenergic sympathetic nerves
C. Cholinergic parasympathetic nerves
D. Cholinergic sympathetic nerves

289.
955.

Muscarinic agonists would be contraindicated in patients with


A. Peptic ulcer
B. Bronchial Asthma
C. Pulmonary edema
D. All of the above

956.
290.

Effective brain - dopamine concentrations can be achieved with


lower doses of Levodopa by addition of :
A Carbidopa
B. Benserazide
C. Both A & B
D. Neither A nor B

957.
291.

Which of the following would be classified as miotics?


A. Pilocarine
B. Physostigmine
C. Both A & B
D. Neither A nor B

958.
292.

Which compound is depolarizing blocker at nicotinic sites?


A. Nicotine
B. Physostigmine
C. Neostigmine
D. DFP

293.
959.

Edrophonium, a very short acting cholinesterase inhibitor


administered in cholinergic crisis should:
A. improve patient condition
B. worsen patient condition
C. has no effect

960.
294.

Which of the following agents might precipitate an attack in a


patient who has Asthma?
A. Propranolol
B. Neostigmine
C. Both A & B
D. Neither A nor B

171
Pharmacy

961.
295.

Antimuscarinic drugs used in Parkinsonism include all of the


following EXCEPT:
A. Orphenadrine
B. Procyclidine
C. Trihexyphenidyl
D. Homatropine
What is the effect of sympathomimetic agents on cardiac rate and
force of contraction?
A. Inhibition
B. Excitation
C. Both
D. Neither

963.
297.

Which of the following has greatest CNS effects?


A. Hexamethonium
B. Neostigmine.
C. Nicotine
D. Succinylcholine

964.
298.

Which of the following is true about ACh?


A. Stimulates muscarinic receptors
B. Stimulates adrenal medulla.
C. is released from parasympathetic pre and post ganglionic
nerve endings.
D. all of the above

965.
299.

Atropine :
A. is present in the leaves of the nightshade plant
B. Blocks cholinergic muscarinic receptors.
C. Blocks cholinergic Nicotinic receptors
D. A and B

966.
300.

Acetyl choline is the mediator of nerve activity at:


A. All autonomic ganglia.
B. The parasympathetic nerve endings.
C. The neuromuscular junction of skeletal muscle
D. All of the above

Amikacin belong to the group of drugs which:


A. are Aminoglycosides
B. Used treat gram negative infections
C. may produce ototoxicity
D. All of the above

Properties of H1 antihistamines apart from their ability to


antagonize histamine:
A. Sedative effect
B. Anticholinergic effect
C. Antimotion sickness effect
D. All of the above

962.
296.

967.
301.

968.
303.

172
Pharmacy

969.
304.

The major direct effect(s) of histamines on the cardiovascular


system.
A. Capillary dilation
B. Increased capillary permeability
C. Both A & B
D. Neither A nor B

970.
305.

Co-trimoxazole is contraindicated in:

A. Blood dyscrasias
B. Pregnancy
C. Predisposition to folate deficiency
D. All of the above
971.
306.

972.
307.

308.
973.

309.
974.

310.
975.

311.
976.

Which of the following drugs with anti inflammatory effects is/are


used in anti-asthma treatment?
A. Prednisone
B. Cromolyn sodium
C. Dexamethasone
D. All of the above
For which of the following anti-asthma drugs is it recommended
that plasma concentrations be determined as a guide for safety?
A. Ipratropium bromide
B. Salmeterol
C. Terbutaline
D. Theophylline
Which of the following drugs are beta agonists used in asthma
treatment?
A. Beclomethasone
B. Salmeterol
C. Ipratropium bromide
D. Theophylline
Salmeterol is a beta adrenergic agonist evolved from adrenaline.
Its therapeutic advantages include:
A. Selectivity for bronchodilation
B. Minimal cardiac side effects
C. Extended duration of action
D. All of the above
An individual who is to undergo abdominal surgery is to receive a
neuromuscular blocking agent as an adjunct to anesthesia to
enhance skeletal muscle relaxation. A commonly used agent for
this purpose?
A. Neostigmine
B. Succinylcholine
C. Neomycin
D. None of the above
Post operative urinary retention may be treated with:
A. DFP
B. Carbachol
C. Bethanechol
D. Ephedrine

173
Pharmacy

977.
312.

Which of the following may be used for treatment of Bronchial


asthma?
A. Salbutamol
B. Bitolterol
C. Terbutaline
D. All of the above

978.
313.

Which of the following is produced by alpha adrenergic receptor


stimulation?
A. Intestinal relaxation
B. Vasodilation
C. Positive chronotropic effect
D. All of the above

314.
979.

Which of these drugs is used in the treatment of bronchial asthma


by inhalation?
A. Salbutamol
B. Theophylline
C. Both A & B
D. Neither A nor B
Beta adrenergic agonistic properties are characteristic of :
A. Isoprenaline
B. Tolazoline
C. Both A & B
D. Neither A nor B
Blockade of the direct cardiac effect of Adrenaline is a property of:
A. Phenoxybenzamine
B. Propranolol
C. Both A & B
D. Neither A nor B
Noradrenaline released by brief stimulation of post ganglionic
nerves has short lived effects because it is rapidly:
A. Taken back into nerve endings.
B. Methylated by COMT
C. Deaminated by MAO
D. Inactivated by receptors on effector cells.
Blockade of the alpha 1 adrenoceptors produces:
A. Decreased sympathetic tone in peripheral vasculature
B. Reversal of effects of Adrenaline on BP
C. Reversal of effects of Adrenaline on heart
D. A&B
Acetylcholine would directly produce which of the following?:
A. Mydriasis
B. Bronchoconstriction
C. Decreased GI motility
D. All of the above
Untoward reactions to Dapsone are common and include:
A. Hemolysis
B. Methemoglobinemia
C. Both A & B
D. Neither A nor B

315.
980.

316.
981.

317.
982.

318.
983.

319.
984.

320.
985.

174
Pharmacy

986.
321.

Which of the following drugs are used for treatment of Leprosy?


A. Dapsone
B. Rifampicin
C. Clofazimine
D. All of the above

987.
322.

Today, Streptomycin is primarily limited to use in patients with


severe possibly life threatening tuberculosis, due to its side effects
which include:
A. Arthritis
B. Hepatotoxicity
C. Both
D. Neither
Following are toxicities of Streptomycin which limits its long term use:
A. Ototoxicity
B. Nephrotoxicity
C. Both
D. Neither
Important drug interactions with Rifampin include:
A. Increased estrogen metabolism
B. Decreased elimination of warfarin
C. Both
D. Neither
Match the drug with the mechanism of action: Rifampicin:
A. Competes with PABA for enzyme active centre
B. Inhibits mycolic acid synthesis.
C. Inhibits RNA synthesis by binding to DNA dependent RNA
polymerase
D. None of the above.
Match the drug with the mechanism of action: Cotrimoxazole:
A. Competes with PABA for enzyme active centre
B. Inhibits mycolic acid synthesis.
C. Inhibits RNA synthesis by binding to DNA dependent RNA
polymerase
D. None of the above.
Gastric acid secretion follows a circadian pattern. Which of the
following are true?
A. Secretion decreases in evening
B. Secretion increases during the day
C. secretion nearly ceases at night
D. All of the above

988.
323.

324.
989.

990.
325.

991.
326.

992.
327.

328.
993.

A ( Most peptic ulcer

patients are instructed to eat


a snack before retiring
however when gastric
emptying occurs 2 hrs later
the gastric acidity increases.
Therefore, the evening
snack interrupts the
circadian rhythm) .

Which of the following are true?


D
A. Mg toxicity is manifested by neurological, neuromuscular, and
cardiovascular impairment.
B. The maximum intragastric pH obtained with MgAl mixtures is
about 4.5
C. Mg(OH)2 is used as a catharitic.
D. All of the above

175
Pharmacy

994.
329.

Which of the following properties are typical of the


aminoglycoside group?
A. act by inhibiting the 30S Bacterial ribosomal subunit
B. eliminated by glomerular filtration
C. poorly absorbed from GI tract
D. All of the above

330.
995.

67 year old catheterised woman developed an infection caused by


E fecalis. She was treated with gentamicin and drug X. When X
was administered, it caused release of histamine, which in turn
produced, fever, chills, pruritis, redness in face, neck and upper
body. The drug was most likely:
A. Azithromycin
B. Clindamycin
C. Linezolid
D. Vancomycin

D
This is Redman
syndrome
associated with
Vancomycin

331.
996.

Which of the following antimycobacterial agents are absorbed


orally?
A. Ethambutol
B. Isoniazid
C. Rifampin
D. All of the above
332.
Which of the following is affected by an individual patients
997.
acetylation
.
staus?
A. Acetaminophen
B. Aspirin
C. Isoniazid
D. All of the above
333.
Gentamicin
is usually administered :
998.
A. Orally
B. Parentrally
C. Both
D. Neither
334.
Which
of the following antibiotics are employed principally as
999.
topical therapy:
A. Amikacin
B. Bacitracin
C. Gentamicin
D. Neomycin
335.
Hepatotoxicity is associated with the use of:
1000.
A. Cycloserine
B. Isoniazid
C. Rifampin
D. Streptomycin
336.
Which of the following antimicrobials may result in dangerously
1001.
high concentrations in blood and tissues in a 81 year old woman
with severely impaired renal function unless intervals of doses are
adjusted:

B,D

B,C

176
Pharmacy

A. Amikacin
B. Gentamicin
C. Streptomycin
D. All of the above
338.
1002.

May produce ototoxicity:


A. Cycloserine
B. Ethambutol
C. Isoniazid
D. Streptomycin

1003.
339.

Which of the following TB drugs may produce impairment of liver D


function at therapeutic doses?
A Isoniazid
B. Rifampicin
C. Pyrazinamide
D. All of the above

1004.
340.

Frequently used as sole agent for chemoprophylaxis of TB:


A. Streptomycin
B. Cycloserine
C. Ethambutol
D. Isoniazid

1005.
341.

Which drugs when given orally can be used to treat TB meningitis


(cross blood brain barrier)
A. Ethambutol
B. Isoniazid
C. Rifampin
D. All of the above
A 26 year old man with TB was given ethambutol as a single
dose in combination with isoniazid and rifampicin. Which organ
should be monitored for ADR of Ethambutol therapy?
A. Bone marrow
B. Eyes
C. Kidney
D. Liver

1007.
343.

Match the drug with the mechanism of action: Isoniazid


A. Competes with PABA for enzyme active center
B. inhibits mycolic acid synthesis
C. inhibits RNA synthesis by binding to DNA dependent RNA
polymerase
D. interferes with envelope formation by poxviruses

1008.
344.

Match the drug with the Mechanism of action: Streptomycin


A. Competes with PABA for enzyme active center
B. inhibits mycolic acid synthesis
C. inhibits RNA synthesis by binding to DNA dependent RNA
polymerase
D. irreversible inhibition of bacterial protein synthesis

342.
1006.

177
Pharmacy

1009.
345.

A 22 year old man has returned from a vaccation from Mexico


suffering from travelers diarrhea; a gram negative bacillus is
suspected as the caustitive pathogen. Which one of the following
drugs would most likely to be effective in this patient?
A. Ampicillin
B. Ciprofloxacin
C. Erythromycin
D. Sulfisoxazole

1010.
346.

Minimum duration of therapy for active TB should be:


A. 3 to 6 months
B. 6 to 9 months
C. 1 to 2 years
D. 4 years

1011.
347.

Which of the following are side effects of Isoniazid?


A. Allergic reactions
B. Hepatotoxicity.
C. Peripheral Neuritis
D. All of the above
Nephrotoxicity and/or ototoxicity may result from the
administration of :
A. Amikacin
B. Doxycycline
C. Gentamicin
D. Erythromycin

349.
1012.

1013.
350.

1014.
351.

352.
1015.

353.
1016.

Fluoroquinolones are
effective in diarrhoea caused
by E coli, shigella and
salmonella

A,C

Known to be Nephrotoxic:
B,C
A. Amoxycillin
B. Gentamicin
C. Tobramycin
D. All of the above
Which of the following antibiotics produce curare like neuromuscular B
blockade?
A.Amoxycillin
B. Gentamicin
C. Erythromycin
D. All of the above
Tobramycin inhibits:
D
A. Bacterial cell membrane function
B. Cell wall synthesis
C. Folic acid synthesis
D. Bacterial Protein synthesis
A 22 year old woman with a lower urinary tract infection (UTI)
was given an antimicrobial agent that is only used for UTIs. This
drug was most likely:
A. Ampicillin
B. Ciprofloxacin
C. Cotrimoxazole
D. Nalidixic acid

178
Pharmacy

1017.
354.

Hepatic function need to be monitored which of the following


therapy?
A. Antituberculosis therapy
B. Antiepilleptic therapy
C. Hyperlipidemia therapy
D. All of the above

1018.
355. Which of the contrast media can be given orally to treat
hyperthyroidism?
A. Diatrizoate
B. Iohexol
C. Radioactive iodine
D. Thyrotropin

1019.
356.

Which of the following is an insulin secretagogue?


A. Acarbose
B. Glibenclamide
C. Metformin
D. Rosiglitazone

1020.
357.

Which of the following medications produce symptomatic changes


within 24 hours, although maximal effects may not be seen until 2
weeks later when given to a hyperthyroid patient?
A. Atenolol
B. Methimazole
C. Potassium Iodide
D. Propranolol

1021.
358.

Which of the following insulin secretagogues are second


generation sulphonylureas?
A. Acarbose
B. Chlorpropamide
C. Glipizide
D. Repaglinide

1022.
359.

Which of the following medications inhibits the release of thyroid


hormones by inhibiting thyroglobin proteolysis?
A. Atenolol
B. Methimazole
C. Potassium Iodide
D. Propranolol

1023.
360.

Which of the following insulin secretagogues are first generation


sulphonylureas?

A. Chlorpropamide
B. Glibenclamide
C. Glipizide
D. Repaglinide

179
Pharmacy

1024.
361.

Which of the following medications inhibits the thyroid hormones


synthesis and release and reduces thyroid gland vascularity?
A. Atenolol
B. Methimazole
C. Potassium Iodide
D. Propranolol

363.
1025.

Which of the following antithyroid drugs usually require 2 to 4


months of treatment to achieve initial euthyroid status?
A. Methimazole
B. Propylthiouracil
C. Thyrotropin
D. None of the above
Which of the following medications is preferred antithyroid agent
for a 27 year old pregnant woman with hyperthyroidism?
A. Methimazole
B. Potassium iodide
C. Propyl thiouracil
D. All of the above
Which of the following is classified as a long acting insulin?
A. Insulin aspart
B. Insulin Glargine
C. Insulin Glulisine
D. All of the above
Which of the following drugs directly interfere with first step
(organification) in thyroid hormone biosynthesis in the thyroid
gland?
A. Methimazole
B. Propyl thiouracil
C. Both A & B
D. Neither A nor B
Which of the following is classified as an intermediate acting
insulin?
A. Insulin aspart
B. Isophane Insulin
C. Both A & B
D. Neither A nor B
Which of the following are beta 1 selective blockers used for short
term treatment of tachycardia associated with hyperthyroidism.?
A. Atenolol
B. Metoprolol
C. Both A & B
D. Neither A nor B

A,B

Which of the following are beta 1 selective blockers used for short
term treatment of tachycardia associated with hyperthyroidism.?
A. Labetalol
B. Propranolol
C. Both A & B
D. Neither A nor B

1026.
364.

365.
1027.

366.
1028.

367.
1029.

1030.
368.

1031.
369.

180
Pharmacy

1032.
370.

Which of the following is recommended for IV administration to


treat a 21 year old man with type1 diabetes mellitus who is in a
state of severe ketoacidosis?
A. Regular insulin
B. NPH insulin
C. Both
D. Neither

371.
1033.

Which of the following are classified as rapid acting insulin?


A. Insulin Aspart
B. Insulin Lispro
C. Both A & B
D. Neither A nor B

372.
1034.

What type of receptors are predominantly found in cardiac muscle? B


A. alpha1 receptors
B. beta 1 receptors
C. Both A & B
D. Neither A nor B

373.
1035.

Catecholamines released into the synaptic cleft interact with


adrenergic receptors until inactivated by one of 3 basic
mechanisms.
What is the major mechanism responsible for inactivation of
Catecholamines in the nervous system?
A. MAO (Mono Amine oxidase)
B. COMT(Catechol, O methyl transferase)
C. Neuronal uptake

374.
1036.

Which one inactivates the intraneuronal Catecholamines?


A. MAO (Mono amine oxidase)
B. COMT(Catechol-O-methyl transferase)
C. Neuronal uptake

375.
1037.

Which of the following are not catecholamines?


A. Adrenaline
B. Amphetamine
C. Dopamine
D. Noradrenaline

376.
1038.

Which one of the following is the best treatment for muscarine


poisoning?
A. Adrenaline
B. Atropine
C. Nicotine
D. Noradrenaline

1039.
377.

What is the effect of Isoprenaline on heart rate?


A. Increase
B. Decrease
C. No effect

181
Pharmacy

1040.
378.

What is the effect of Atropine on GI motility?


A. Increase
B. Decrease
C. No effect

1041.
379.

Which one of the following agents produce CNS stimulation?


A. Amphetamine
B. Ephedrine
C. Both A & B
D. Neither A nor B

1042.
380.

Which of the following are true regarding Magnesium trisilicate?


A. More potent than Mg hydroxide
B. Can cause renal stones
C. Causes constipation
D. None of the above

1043.
381.

Which one of the following statements is/are true regarding


Aluminium containing antacids?
A. Only a small amount of Aluminium is absorbed systemically
B. Al hydroxide forms insoluble complexes which prevent
absorption of dietary phosphate.
C. Aluminium hydroxide causes constipation
D. All of the above

1044.
382.

Local treatment of vaginal candidiasis can be achieved with:

A. Metronidazole
B. Tinidazole
C. Clotrimazole
D. None of the above
383.
1045.

Calcium carbonate is a potent antacid with several drawbacks.


Which of the following are hazards of calcium carbonate use?
A. Hypercalcemia
B. Nephrolithiasis
C. Diarrhea
D. Hyperuricemia

A,B

384.
1046.

The milk alkali syndrome is associated with chronic ingesion of


excess Calcium. Features of milk alkali syndrome are?
A. Hypercalcemia
B. Renal impairment
C. Metabolic alkalosis
D. All of the above

385.
1047.

Which of the following effects of antimuscarinic agents employed


in the treatment of peptic ulcer disease?
A. Suppress acid secretion
B. Increase gastric emptying rate
C. Both A & B
D. Neither A nor B

182
Pharmacy

1048.
386.

Which of the following conditions increase the likelihood of


complications with repeated use of sodium bicarbonate ?
A. Hypertension
B. Cardiac failure
C. Renal insufficency
D. All of the above

D The high Na load

387.
1049.

Which of the following conditions are contraindications to the use


of anticholinergics ?
A. Hepatitis
B. Prostatism
C. Both A & B
D. Neither A nor B

388.
1050.

Dosages vary widely for individual patients and require titration


using side effects as indicators of adequate dosage. Which of the
following are side effects of anticholinergics?
A. Dry mouth
B. Photophobia
C. Blurred vision
D. All of the above

389.
1051.

Which of the following is/are specific side effects of cimetidine?


A. Gynecomastia
B. Photophobia
C. Blurred vision
D. All of the above
Which of the following non-systemic antacids may depress the
CNS in patients with renal insufficiency?
A. Sodium bicarbonate
B. Calcium carbonate
C. Aluminium hydroxide
D. Magnesium hydroxide

391.
1053.

Soluble insuline can be given by:


A. Intravenous route
B. Intramuscular route
C. Subcutaneous route
D. All of the above

392.
1054.

Which combinations if given in correct proportions would have


the least GI problems with constipation or diarrhea?
A. Sodium bicarbonate+ Calcium carbonate
B. Calcium carbonate +. Aluminium hydroxide
C. Aluminium hydroxide+ Magnesium hydroxide
D. Magnesium hydroxide+ Sodium bicarbonate

393.
1055.

Which antacid combines on a mole basis with HCl ?


A. Sodium bicarbonate
B. Calcium carbonate
C. Aluminium hydroxide
D. Magnesium hydroxide

390.
1052.

adversely affect patients


hypertension, cardiac failure
and edema by increasing
intravascular fluid volume.

183
Pharmacy

1056.
394.
Which antacid may be used to decrease phosphate stores in a
patient with chronic renal failure or bone disease?
A. Sodium bicarbonate
B. Calcium carbonate
C. Aluminium hydroxide
D. Magnesium hydroxide

1057.
395.
Which antacid may induce systemic alkalosis?
A. Sodium bicarbonate
B. Calcium carbonate
C. Aluminium hydroxide
D. Magnesium hydroxide
1058.
396.
Which antacid may cause milk alkali syndrome?
A. Sodium bicarbonate
B. Calcium carbonate
C. Aluminium hydroxide
D. Magnesium hydroxide
1059.
397.
Potassium supplements must NOT be given along with:
A. Amiloride
B. Trianterene
C. Spironolactone
D. All of the above
1060.
398.
Which of the following antihistamines blocks gastric secretion?
A. Terfenadine
B. Loratidine
C. Astemizole
D. None of the above
1061.
399.
Spironolactone is usually given in combination with :
A. Acetazolamide
B. A thiazide diuretic
C. Furosemide
D. None of the above
1062.
400.
Which of the following statements is INCORRECT?
A. Acetazolamide acts mainly on the distal tubules and produces
metabolic alkalosis.
B. High ceiling diuretics act on theascending loop of Henle, and
plasma electrolytes should be checked at frequent intervals.
C. Trimeterene is a potassium sparing diuretic, and the most
common side effects are nausea, vomiting, dizziness, and leg
cramps.
D. Thiazides act mainly on the distal tubules, they do not modify
the acid base balance and are effective in either the acid nor
alkaline urine.

1063.
401.
This drug is a potassium sparing diuretic
A. Acetazolamide
B. Hydrochlorothiazide
C. Furosemide
D. Triameterene

184
Pharmacy

402. Lack of absorption after oral administration limits its diuretic effect
1064.
by this dosing route but makes it useful in treatment of ingested toxic substances.
A. Hydrochlorothiazide
B. Mannitol
C. Both
D. Neither
1065.
403.
This drug produces a rapid, profound diuresis due to the inhibition
of salt transport in the ascending limb of the loop of Henle
A. Chlorthalidone
B. Furosemide
C. Acetazolamide
D. Mannitol
1066.
404.
Hydrochlorothiazide
A. Is often combined with a potassium sparing diuretic for
chronic treatment programs
B. May induce an abnormal glucose tolerance test
C. Is effective in both alkalosis and acidosis
D. All of the above
405.
Hypokalemia,
gout and aggravation of diabetes mellitus are seen
1067.
as side effects of
A. Mannitol
B. Hydrochlorothiazide
C. Acetazolamide
D. Triamterene
406. Which of the following drugs is an appropriate choice for initial antihy1068.
pertensive therapy in an otherwise healthy adult with mild hypertension?
A. Bumetanide
B. Triamterene
C. Hydrochlorothiazide
D. Aldactone
407.
Exerts its diuretic effect by inhibiting the action of aldosterone on
1069.
the distal tubule
A. Spironolactone
B. Urea
C. Triameterene
D. Ammonium chloride
1070.
408.
The primary disadvantage of the thiazides is that
A. They are very toxic
B. They cannot be used over long periods of time since they
alter plasma pH
C. They increase potassium loss from the body
D. They must be given parenterally
1071.
409.
Which of the following produces a major increase in the renal
excretion of NaHCO 3 ?
A. Furosemide
B. Acetazolamide
C. Ethacrynic acid
D. Probenecid
410.
An
osmotic
diuretic which is not reabsorbed to any degree by the
1072.
renal tubules
A. Acetazolamide

185
Pharmacy

411.
1073.

412.
1074.

413.
1075.

414.
1076.

415.
1077.

416.
1078.

417.
1079.

1080.
418.

B. Aminophylline
C. Furosemide
D. Mannitol
The pharmacologic actions of this agent make it useful as a
diuretic and as bronchodilator
A. Acetamolamide
B. Aminophylline
C. Ammonium chloride
D. Spironolactone
Furosemide inhibits the effectiveness of the countercurrent
mechanisms because it
A. increases permeability of ascending limb of loop of Henle to
water
B. decreases permeability of collecting tubule of water
C. inhibits urea absorption
D. inhibits chloride and sodium reabsorption in the loop of
Henle
The androgen like effects of spironolactone are probably due to
A. Antagonism of aldosterone
B. Its steroid structure
C. Inhibition of pituitary hormones
All the following are thiazide diuretics except
A. Chlorothiazide
B. Hydroflumethiazide
C. Diazoxide
D. Polythiazide
Frusemide increases excretion of all of the following EXCEPT:
A. Sodium
B. Potassium
C. Calcium and magnesium
D. Uric acid
Antichloinergic agents are more effective and have a longer
duration of action when administered in combination with
A. Beta adrenergic agonists
B. Theophylline
C. Glucocorticoids
D. Cromolyn Sodium
Which of the following drugs are anticholinergic agents used in
asthma treatment?
A. Dexamethasone
B. Hydrocortisone
C. Ipratropium Bromide
D. Nedocromil Sodium
Which of the following is characteristic of duodenal ulcer?
A. a chronic disease with exacerbations and remissions
B. 80% of peptic ulcers are duodenal
C. typical symptoms are gnawing epigastic pain which begins
2 to 2 hours after meals relieved
D. all of the above

186
Pharmacy

419.
1081.

Which of the following are used for the treatment of Peptic ulcer
disease?
A. Diet therapy
B. Combination therapy with omeprazole if H Pylori infection
is confirmed.
C. H 2 blockers
D. All of the above

421.
1082.

An antacid given in the fasting state is effective for about 30


minutes. At which time in relation to meals should antacids be
given for maximum efficacy and duration of effect?
A. Before eating
B. Immediately after eating
C. One hour after eating
D. two hours after eating

C A single dose given one


hour after eating prolongs
buffering capacity to about 2
hrs. A second dose given 3
hrs after eating lasts for
about 1 hour

422.
1083.

Agents implicated as chemical mediators in immediate


hypersensitivity reactions include:
A. Histamine
B. Bradykinin
C. Both A & B
D. Neither A nor B

1084.
423.

Antihistamine compounds can produce:


A. Local anesthesia
B. Drying of mucous secretions
C. Sedation
D. All of the above
Parenteral administration of histamine in humans should cause:
A. an elevation of BP
B. a fall in BP
C. Contraction of uterus
D. Rise in pulmonary BP

1085.
424.

425.
1086.

1087.
426.

427.
1088.

Which of the following agents inhibits release of histamine from mast cells? A
A. Cromolyn Sodium
B. Cimetidine
C. Chlorpheniramine
D. All of the above
A drug useful in prevention of motion sickness:
B
A. Meperidine
B. Meclizine
C. Meprobamate
D. Metronidazole
The mechanism by which antihistaminic compounds relieve
allergic conditions involves:
A. Acceleration of excretion of histamine
B. Neutralization of the effects of histamine by producing
opposite reactions
C. Competition with histamine in attachment to cell receptors
D. None of the above

187
Pharmacy

1089.
428.

When given before asthmatic attacks, it can prevent


bronchoconstriction for several hours, however, it is ineffective if
given after the asthmatic attack has started:
A. Diphenhydramine
B. Betazole
C. Cromolyn sodium
D. Chlorpheniramine

1090.
429.

A H2 receptor antagonist which can be used in the treatment of


gastric and duodenal ulcers:
A. Diphenhydramine
B. Ranitidine
C. Promethazine
D. 4 methyl histamine

430.
1091.

Which of the following agents would be contraindicated in


patients concurrently treated for fungal infections with
ketoconazole because of possible cardiac toxicity?
A. Astemizole
B. Diphenhydramine
C. Terfenadine
D. A&C
Which of the following agents used to treat angina gets increases
cyclic GMP?
A. Nitroglycerine
B. Nifedipine
C. Terfenadine
D. Metoprolol
Which of the following agents used to treat angina gets converted to
nitric oxide
A. Nitroglycerine
B. Nifedipine
C. Terfenadine
D. Metoprolol
Which of the following lipid lowering agents is most associated
with increasing serum HDL levels?
A. Atorvastatin
B. Niacin
C. Cholestyramine
D. Fenofibrate

1095.
434.

Which of the following lipid lowering agents mediates its action


via activation of peroxisome proliferator?
A. Atorvastatin
B. Niacin
C. Cholestyramine
D. Fenofibrate

1096.
435.

Which of the following is a competitive HMG - CoA reductase


inhibitor?
A. Atorvastatin

431.
1092.

432.
1093.

433.
1094.

188
Pharmacy

436.
1097.

1098.
437.

438.
1099.

439.
1100.

1101.
440.

1102.
441.

442.
1103.

B. Niacin
C. Cholestyramine
D. Fenofibrate
Which of the following is most noted for elevating both liver and
muscle enzymes?
A. Atorvastatin
B. Niacin
C. Cholestyramine
D. Fenofibrate
Which of the following lipid lowering agents affects the
absorption of bile acids from the GI tract?
A. Atorvastatin
B. Niacin
C. Cholestyramine
D. Fenofibrate
Which of the following lipid lowering agents lowers serum
cholesterol concentrations by selectively inhibiting the absorption
of cholesterol and related phytosterols from the intestine?
A. Atorvastatin
B. Niacin
C. Ezetimibe
D. Fenofibrate
Digoxin was added to the therapeutic regimen of ramipril and
frusemide for a 68 year old man with progressive CCF. Which of
the following is the molecular target for digoxins actions in this
patient ?
A. Aldosterone receptor
B. Angiotensin converting enzyme
C. Angiotensin receptor
D. Na+/ K+ ATPase
Which of the following diuretics would be preferred to reduce
edema in a 68 year old woman with moderate congestive heart
failure?
A. Ethacrinic acid
B. Furosemide
C. Hydrochlorthiazide
D. Spironolactone
63 year old man with a history of cardiac disease taking
Atorvastatin for lowering cholesterol he develops muscle pain and
weakness with elevated CPK. Which of the following life
threatening conditions in this patient is observed?
A. Myocardial infarction
B. Pulmonary embolism
C. Rhabdomyolysis
D. Seizures
Which of following is not a side effect of therapeutic doses of
nitroglycerin?
A. Throbbing headache
B. Drug rash
C. Syncope

189
Pharmacy

443.
1104.

444.
1105.

445.
1106.

1107.
446.

447.
1108.

448.
1109.

449.
1110.

450.
1111.

D. Methamoglobinemia
Tolerance to nitroglycerin may be overcome by
A. initially using the largest safe dose of the drug
B. using other nitrates
C. temporary discontinuing the drug for one or two weeks.
D. use of higher doses
Xanthine alkaloid which may produce coronary vasodilation
A. papaverine
B. quinidine
C. dipyridamole
D. theophylline
Can relieve angina pectoris by decreasing myocardial work, but
may precipitate congestive heart failure
A. phentolamine
B. phenoxybenezamine
C. propranolol
D. atropine
Of the vasodilators listed below, the drug of choice for treatment
of acute anginal attacks is
A. nitroglycerin
B. acetylcholine
C. papaverine
D. propranolol
Which of the following medications used to maintain sinus rhythm
in a 54 year old man is classified as a Class IA antiarrhythmic
agent?
A. Amiodarone
B. Lidocaine
C. Carvedilol
D. Quinidine
A 57 year old man who is being treated with a medication to
maintain atrial sinus rhythm develops conditions referred to a
cinchonism by his physician. Which of the following medications
is most likely responsible for this adverse effect in this patient?
A. Adenosine
B. Amiodarone
C. Lidocaine
D. Quinidine
A 56 year old African American man has been diagnosed with
moderate hypertension. Which of the following medications would
be most appropriate to manage hypertension in this patient?
A. Captopril
B. Furosemide
C. Hydrochlorothiazide
D. Losartan
A 64 year old man whose hypertension is being managed by
hydrochlorithiazide develops severe hypokalemia. Addition of
which of the following medications will minimize thiazide induced
hypokalemia by competing with aldosterone in the distal renal
tubule?

190
Pharmacy

1112.
451.

1113.
452.

1114.
453.

1115.
454.

1116.
455.

456.
1117.

457.
1118.

A. Amiloride
B. Captopril
C. Losartan
D. Spironolactone
A 45 year old man is being treated for hypertension with a
nonselective beta blocker. Which of the following medications is
most likely being used to treat this patient?
A. Atenolol
B. Carvedilol
C. Labetalol
D. Propranolol
Which of the following antihypertensive medications would be
preferred for the treatment of chronic hypertension in a 33 year old
pregnant woman?
A. Methyldopa
B. Carvedilol
C. Clonidine
D. None of the above
Which of the following lipid lowering medications could reduce
the effectiveness of digoxin when used to control ventricular rate
in a 55 year old woman with atrial fibrillation ?
A. Cholestyramine
B. Ezetimibe
C. Fenofibrate
D. Lovastatin
Which of the following would be most contraindicated for the
treatment of angina in 55 year old man with severe bronchial
asthma ?
A. Amlodipine
B. Atenolol
C. Diltiazem
D. Propranolol
Which of the following agents would be most effective for the
treatment of variant angina (Prinzmetals angina)
A. Atenolol
B. Metoprolol
C. Nifedipine
D. Nitroglycerine
Which of the following medications used in the treatment of
angina pectoris in a 55 year old man exerts its beneficial actions by
increasing vascular cGMP levels?
A. Diltiazem
B. Metoprolol
C. Nifedipine
D. Nitroglycerine
Which of the following 2 drugs are most noted for causing a
throbbing headache after first exposure?
A. Atorvastatin
B. Bisoprolol
C. Isosorbide dinitrate
D. Nitroglycerin

C&D

191
Pharmacy

1119.
458.

Which of the following 2 drugs are most noted for causing rapid
tolerance on continuous exposure?
A. Carvedilol
B. Bisoprolol
C. Isosorbide dinitrate
D. Nitroglycerin

C&D

459.
1120.

Which of the following drugs produce greater venodilation than


arterial dilation, and thus, reduce preload?
A. Isosorbide dinitrate
B. Spironolactone
C. Nitroglycerin
D. Enalapril
Which of the following medications would be preferred as
monotherapy for the treatment of a 53 year old man with moderate
hypertension and mild benign prostatic hyperplasia (BPH)?
A. Carvedilol
B. Labetalol
C. Propranolol
D. Terazosin
A 56 year old diabetic, hypertensiveman is being managed with a
medication that reduces both preload and afterload while reducing
left ventricular hypertrophy (LVH) without worsening insulin
resistance, hyperlipidemia or sexual function. This medication,
however, did cause an annoying, nonproductive cough. Which of
the following medications was most likely being used to treat this
patient?
A. Atenolol
B. Captopril
C. Hydrochlorothiazide
D. Losartan
A 64 year old woman with an acute hypertensive crisis was treated
intravenously with a vasodilator drug that reduces both preload
and afterload. However, prolonged infusion of this drug may lead
to metabolic acidosis and an increase in blood lactate.
Which of the following drugs was most likely used to treat this
patient?
A. Fenoldopam
B. Hydralazine
C. Sodium Nitroprussside
D. Nitroglycerine
Which of the following drugs is recommended to take sublingually
to be dissolved under the tongue or in buccal pouch immediately
following indication of an anginal attack in a 58 year old man?
A. Amyl nitrate
B. Atenolol
C. Diltiazem
D. Nitroglycerin
Maintenance of antihypertensive therapy:
A. is really needed

A&C

1121.
460.

1122.
461.

1123.
462.

463.
1124.

464.
1125.

A and C

192
Pharmacy

465.
1126.

1127.
466.

1128.
467.

1129.
468.

469.
1130.

B. will rarely have to be altered once the patient is discharged


from the hospital
C. will usually necessitate the use of combination therapy
D. all of the above
Orthostatic hypotension is a side effect accompanying therapeutic
doses of:
A. Reserpine
B. Guanethidine
C. Alpha methyldopa
D. All of the above
Drugs may lower blood pressure by:
A. blocking peripheral sympathetic nerve endings
B. decreasing central sympathetic outflow
C. inhibiting the rennin angiotensin system
D. all of the above
The rationale for administering propranolol along with minoxidil
in treating hypertension is to:
A. augment the hypotensive action by blockade of
beta adrenergic receptors in peripheral vessels
B. suppress rennin release
C. prevent reflex tachycardia in response to vasodilation
D. B and C
Which of the following would be your first choice for treating
moderate hypertension?
A. nifedipine
B. minoxidil
C. clonidine
D. all of the above
Which of the following statements are correct?
A. hydralazine may cause development of a collagen disease
resembling disseminated lupus erythematosus
B. reserpine and guanethidine both impair sympathetic nerve
discharge by depleting catecholamine stores
C. the most serious complication of reserpine therapy is mental
depression which may occasionally lead to suicide
D. all of the above

470.
1131.

Side effects of amrinone include:


A. nausea
B. liver enzyme changes
C. vomiting
D. all of the above

1132.
471.

Which of the following is the cellular mechanism by which


digoxin improves the force of myocardial contraction?
A. Releasing catecholamines from nerve endings
B. Increasing sensitivity of postsynaptic catecholamine
receptors
C. Inhibiting the Na K transport enzyme in the cardiac cell
membrane

193
Pharmacy

1133.
473.

Which of the following mechanisms would cause tachycardia?


A. Increased sympathetic tone to the heart
B. Increased parasympathetic tone to the heart

1134.
474.

Which structure has the highest rate of automaticity and is


normally the primary pacemaker of the heart?\
A. SA node
B. AV node
C. Bundle of His
D. Vagus nerve

1135.
475.

Digoxin promotes diuresis in persons with:


A. Heart failure but no edema
B. Edema and heart failure refractory to digoxin therapy
C. Congestive heart failure with edema
D. Edema secondary to renal disease

1136.
476.

In congestive heart failure, edema results from a decreased cardiac


output because:
A. Venous return is impaired and capillary hydrostatic pressure
increases
B. Renal blood flow is decreased
C. Both A & B
D. Neither A nor B

477.
1137.

Digoxin promotes diuresis in congestive heart failure by:


A. Inhibiting aldosterone activity
B. Direct renal vasodilation
C. Inhibiting ADH
D. Improving cardiac function and increasing cardiac output

478.
1138.

Digoxin is clinically used for treatment of:


D
A. congestive heart failure
B. atrial fibrillation
C. atrial flutter
D. all of the above
The first and most characteristic change that digoxin may produce D
in the ECG is:
A. broad notched P waves
B. slurring of the QRS complex
C. tall peaked T waves
D. flattening or inversion of the T waves
The current hypothesis for the positive inotropic action of digoxin C
is that it is due to inhibition of sodium potassium ATPase by
which:
A. Increased intraceullalar K+ causes a decrease in Ca++ influx
B. Increased intracellular K+ causes an increased release of
bound Ca++
C. Increased intracellular Na+ causes an increased release of
bound Ca++
D. Decreased intracellular Na+ causes a decrease in Ca++ influx

479.
1139.

480.
1140.

194
Pharmacy

1141.
481.

Which of the following statements is correct?


A. bigeminy is an important sign of digoxin toxicity
B. digoxin has a low margin of safety
C. central nervous system manifestations of digoxin include
visual disturbance, bizarre neurological manifestations, and
occasionally, excitement and psychosis
D. all of the above

482.
1142.

Agents which can antagonize cardiac arrhythmias produced by


digoxin include:
A. propranolol
B. potassium
C. phenytoin
D. all of the above
Signs or symptoms of digoxin intoxication may include:
A. yellow or green vision
B. nausea and vomiting
C. gynecomastia
D. all of the above
Which of the following are true in regard to digoxin?
A. may produce nausea and vomiting due to stimulation of the
chemoreceptor trigger zone in the medulla oblongata
B. nausea and vomiting are among the earliest signs of
overdosage
C. the diuresis produced in patients with congestive failure is
due to improvement of cardiac function
D. all of the above
Which of the following cation levels decrease patient resistance to
the toxic effects of digoxin?
A. low serum Ca++
B. high serum Ca++
C. low serum K+
D. B and C
Which of the following belong to the newer class of positive
inotropic drugs termed bipyridine compounds that can be used to
treat congestive heart failure?
A. amrinone
B. salbutamol
C. milrinone
D. A & C
Which of the following medications has the highest propensity to
produce a profound hypokalemia when used to manage
hypertension in a 55yearold man?
A. Amiloride
B. Eplerenone
C. Furosemide
D. Spironolactone
Which of the following drugs is preferred to be give intravenously to
a 23 year old man to reduce intracranial pressure associated with
cerebral edema from an automobile accident?

1143.
483.

484.
1144.

485.
1145.

486.
1146.

487.
1147.

488.
1148.

195
Pharmacy

489.
1149.

A. Amiloride
B. Eplerenone
C. Mannitol
D. Spironolactone
Which of the following medications is most appropriate for the
management of a 29 year old man with neurogenic diabetes
insipidus (neurohypophyseal diabetes insipidus) from severe head
trauma?
A. Desmopressin
B. Demeclocycline
C. Hydrochlorothiazide
D. Lithium

490.
1150.

Which of the following statements applies to acetazolamide?


A. inhibits carbonic anhydrase
B. may produce metabolic acidosis
C. depresses tubular bicarbonate reabsorption
D. all of the above

1151.
491.

Which of the following diuretics is LEAST likely to produce


hypokalemia?
A. bumetanide
B. furosemide
C. spironolactone
D. hydrochlorothiazide

1152.
492.

Which of the following diuretics is LEAST likely to produce


hypokalemia?
A. ethacrynic acid
B. furosemide
C. triamterine
D. hydrochlorothiazide

1153.
493.

Dr. Mohammed is considering adding phenobarbitone to Ms. Fathims A


regimen for she is not getting controlled on the regimen she is
already on for her seizures. Which of the following best describe the
mechanism of action of barbiturates in controlling seizures?
A. Inhibit monosynaptic and polysynaptic transmission and
raise seizure threshold in cortex
B. Stabilize neuronal membranes
C. Affect limbic system and thalamus
D. Limit carbonic anhydrase in CNS

1154.
494.

J.G. A 62 year old male has a 10 year history of heart disease


including three myocardial infarctions and, accompanying the last
attack, mitral valve murmur. Following the last attack he was
placed on a reduced sodium diet with which he usually complied.
However, over time, J.G. began to exhibit peripheral edema about
the ankles. He was also being maintained on probenecid for

196
Pharmacy

1155.
495.

1156.
496.

497.
1157.

1158.
498.

499.
1159.

treatment of hyperuricemia which had manifested as a serious


attacks of acute gout. J.Gs physician decided to treat the
peripheral edema with furosemide, using a standard dosage, one
tablet at bedtime. J.G. had been prepared to expect a significant
increase in urine volume and frequently after starting his new
medication. When this did not happened he called his physician.
Why was the expected pharmacological effect of furosemide was
not seen?
A. Furosemide is not effective when the edema is peripherally
localized.
B. Furosemide is not effective in individuals who are on a low
sodium diet.
C. Probenecid induces significant plasma protein binding of
furosemide
D. Probenecid prevents secretion of furosemide into the renal
tubule.
Choose the osmotic diuretic
A. Furosemide
B. Bumetanide
C. Mannitol
D. Thiazide agents
Which of the following factors may precipitate an acute asthma
attack?
A. exercise
B. aspirin
C. respiratory infection
D. All of the above
Some asthmatic patients exhibit aspirin sensitivity. These patients
are also sensitive to other nonsteroidal agents such as
indomethacin, fenoprofen, naproxen, ibuprofen and
phenylbutazone. Daily administration of aspirin may result in
desensitization not only to aspirin, but to other nonsteroidal
antiinflammatory drugs. This phenomenon is called:
A. placebo effects
B. cross tolerance
C. systemic bioavailability
D. potency
The mechanisms of action of which of the following agents used
in the treatment of asthma involve activation of beta receptors and
increased cAMP levels?
A. isoproterenol
B. albuterol
C. terbutaline
D. All of the above
Side effects of cimetidine include
A. male sexual dysfunction
B. gynecomastia
C. decrease metabolism of warfarin and theophylline
D. All of the above

197
Pharmacy

500.
1160.

1161.
501.

1162.
502.

503.
1163.

504.
1164.

505.
1165.

506.
1166.

507.
1167.

508.
1168.

509.
1169.

Omeprazole is most efficacious in the treatment of


A. duodenal ulcers
B. gastric ulcers
C. ZollingerEllison syndrome
D. all of the above
Disopyramide (Norpace) :
A. is a myocardial depressant
B. decreases myocardial conduction velocity
C. decreases myocardial automaticity
D. all of the above
Antiarrhythmic which causes hypotension and bone marrow
depression:
A. Quinidine
B. Lidocaine
C. Procainamide
D. Potassium ions
Cardiac depressant effects of quinidine are diminished by:
A. Hypocalcemia
B. Hypokalemia
C. Hyperkalemia
D. Hyper chloremia
Propranolol is contraindicated in presence of:
A. Complete heart block
B. Atrial flutter
C. Atrial fibrillationl automaticity
D. all of the above
Tinnitus may occur with:
A. Quinidine
B. Lidocaine
C. Procainamide
D. Potassium ions
Quinidine cardiotoxicity can be potentiated by:
A. Digoxin
B. Potassium chloride
C. Amiodarone
D. all of the above
Quinidine has which of the following properties?
A. Causes cinchonism
B. Obtained from plants
C. Effective on oral administration
D. all of the above
Cardiac actions of Quinidine include:
A. depressed electrical excitability
B. prolonged refractory period
C. reduced conduction velocity
D. All of the above
Toxic effects of quinidine include:
A. Severe hypotension
B. Severe hypertension
C. Neither A & B
D. Both A nor B

198
Pharmacy

510.
1170.

511.
1171.

512.
1172.

513.
1173.

514.
1174.

515.
1175.

516.
1176.

1177.

A 59 year old woman with CCF , hyperlipidemia, develops muscle


weakness and raised creatine kinase. Which of the following
drugs is most likely for this ADR?
A. Simvastatin
B. Pravastatin
C. Atorvastatin
D. All of the above
A 61 year old woman with CCF which of the following drugs
would prolong life inspite of its negative inotropic effect on
cardiac contractility?
A. Captopril
B. Digoxin
C. Dobutamine
D. Metoprolol
A 55 year old man with elevated plasma lipids, hypertension, is
given a drug regimen of Atorvastatin, Hydrochlorthiazide and a
cardioselective beta blocker. Which is this beta blocker?
A. Metoprolol
B. Carvedilol
C. Labetalol
D. None of the above
Two months after 50year old man begins diuretic therapy for
hypertension, his lab results show hypokalemia, hypercalcemia,
and hyperglycemia. The man was most likely treated with?
A. Bumetanide
B. Chlorthalidone
C. Mannitol
D. None of the above
In a 59 year old man with hypertension and elevated plasma
rennin activity, treatment with which of the drugs would reduce
BP but further increase plasma rennin activity ?
A. Simvastatin
B. Enalapril
C. Atenolol
D. All of the above
A 69year old woman who complains of severe vomiting and
diarrhoea, was found to be severely dehydrated and
hypercalcemic. After rehydration, which diuretic is most
appropriate to lower calcium levels?
A. Acetazolamide
B. Frusemide
C. Hydrochlorthiazide
D. Mannitol
A 63 year old obese woman with renal disease and type II DM, is
treated for hypertension with a drug that blocks both alpha and
beta receptors. The drug is:
A. Carvedilol
B. Bumetanide
C. Doxazosin
D. None of the above
A Skin reaction usually assosiated with Vancomycin is
A. Red man syndrome
B. Steven Johnsons syndrome
C. Fixed drug eruption
D. None of the above

199
Pharmacy

517.
1178.

518.
1179.

519.
1180.

1181.
520.

521.
1182.

1183.
522.

1184.
523.

1185.
524.

A 56 year old man with chronic stable angina, and previous MI, is
treated with a calcium channel blocker. This patient is most likely
to be treated with:
A. Carvedilol
B. Diltiazem
C. Doxazosin
D. Hydralazine
A 62 year old man brought to emergency Dept, with hypertensive
crisis, was given an IV infusion of nitroprusside, which reduces
both after load and pre load, by:
A. blocking adrenergic neurones
B. decreasing rennin secretion
C. releasing nitric oxide
D. Inhibiting sympathetic outflow
A 65 year old man with chronic hypertension being treated with a
drug that would reduce BP but does not increase bradykinin levels,
serum glucose level or total cholesterol or triglycerides. The drug
is most likely:
A. Amlodipine
B. Carvediliol
C. Losartan
D. Verapamil
Which of the following potassium sparing diuretics is
recommended for patients with severe heart failure?
A. Furosemide
B. Neseritide
C. Ezetimibe
D. Spironolactone
Which of the following drugs is best recommended for rapid
diuresis in a patient with CCF and extensive edema?
A. Furosemide
B. Neseritide
C. Ezetimibe
D. Spironolactone
Verapamil:
A. Is a calcium channel blocker
B. Has antianginal and antihypertensive properties
C. Slows AV nodal conduction
D. All of the above
Which of the following promote diuresis by acting on the kidney?
A. Furosemide
B. Neseritide
C. Spironolactone
D. All of the above

Which of the following has a positive inotropic effect?


A. Bisoprolol
B. Carvedilol
C. Digoxin
D. All of the above

200
Pharmacy

1186.
525.

526.
1187.

527.
1188.

1189.
528.

529.
1190.

530.
1191.

1192.
531.

532.
1193.

533.
1194.

Which of the following produces balanced arterial and venous


B
dilation and promotes diuresis?
A. Furosemide
B. Neseritide
C. Spironolactone
D. All of the above
Toxic effects of Verapamil include:
D
A. Bradycardia
B. Hypotension
C. Asystole
D. All of the above
Glyceryl trinitrate in therapeutic doses may produce:
C
A. Dilation of cerebral blood vessels
B. Reflex tachycardia
C. Both A & B
D. Neither A nor B
A chronic hypertensive man, during his recent visit to the hospital A
had his antihypertensive medication changed from Tenormin to
Zestril. He develops a dry cough after a period of 2 weeks. His
cough could be due to:
A. Accumulation of bradykinin
B. Chest congestion
C. Increase level of nitric oxide
D. None of the above
Which of the following has a significant direct vasodilator action? C
A. Nitroglycerin
B. Hydralazine
C. Both
D. Neither
When nitroglycerin is used for the treatment of angina:
A
A. Headaches occur frequently
B. Exercice tolerance decreases
C. Methemoglobinemia always occur
D. None of the above
Lowers BP by inhibiting angiotensin converting enzyme:
B
A. Doxazosin
B. Enalapril
C. Metoprolol
D. None of the above
A male patient being treated for essential hypertension, is
C
receiving drug X orally. The patient complains of side effects like
sedation, dizziness, dry mouth. His skin is warm and dry. Which
of the antihypertensive drugs would most likely produce these
symptoms?
A. Lisinopril
B. Hydralazine
C. Clonidine
D. Chlorothiazide
Antihypertensive agents that relax vascular smooth muscle such as A
minoxidil produce tachycardia and occasional arrhythmias. Which
of the following agents would best protect against this effect?

201
Pharmacy

1195.
534.

535.
1196.

1197.
536.

1198.
537.

1199.
538.

539.
1200.

1201.
540.

A. Propranolol
B. Enalapril
C. Phentolamine
D. None of the above
A 22 year old boy with asthma, is brought to the emergency with
status asthmaticus. He uses an inhaler of salbutamol for the
treatment of acute bronchospasms. The emergency treatment of
this boy should first be:
A. Cromolyn sodium (inhalation)
B. Epinephrine (SC)
C. Hydrocortisone (IM)
D. Terbutaline(Inhalation)
In a 22 year old woman, desloratadine for the relief of nasal
symptoms of seasonal allergic rhinitis would be better than earlier
second generation antihistamines because of its decreased
incidence of :
A. Dry mouth
B. Q T prolongation
C. Dyspepsia
D. Myalgia
54 year old man has constant high BP and his plasma rennin
activity is elevated. Which of the following agents would most
likely reduce BP?
A. Losartan
B. Methyl dopa
C. Clonidine
D. Guanethidine
In a 45 year old woman, with chronic hypertension, which of the
following neuropeptides would be most likely implicated in his
hypertension.:
A. Angiotensin II
B. Atrial natriuretic peptide
C. Bradykinin
D. Vasoactive intestinal peptide
Lomotil which contains diphenoxylate plus atropine, was
prescribed for a 36 year old man with severe diarrhoea. Atropine
is included in this preparation to:
A. act in synergy with diphenoxylate
B. antagonise sedation caused by diphenoxylate
C. discourage deliberate abuse of diphenoxylate
D. Reduce clearance of diphenoxylate
A 67 year old woman is given docusate, which is a (an)
A. antidiarrhoeal
B. anti-emetic
C. sedative
D. stool softener
Used to control postpartum hemorrhage
A. Oxytocin
B. Ergonovine
C. Both
D. Neither

202
Pharmacy

1202.
541.

A 44 year old woman is diagnosed with a thyroid storm. Which of D


the following medications will most rapidly reduce the
cardiovascular symptoms associated with thyrotoxicosis in this
patient?
A. Lohexol
B. Liothyromine
C. Methimazole
D. Propranolol

1203.
542.

Oxytocin is effective when given


A. IV
B. IM
C. Intranasally
D. All of the above

1204.
543.

Which of the following agents would provide the most rapid B


improvement in symptoms in a 43 year old woman with severe
myxedema coma?
A. Levothyroxine
B. Liothyronine
C. Thyroidreleasing hormone (TRH)
D. Thyroidstimulating homone(TSH)
A 27 year old woman complains of heat intolerance, excessive D
sweating, tremors, palpitations, difficulty in swallowing, and
muscular weakness. Oral treatment with which of the following
will produce relief within 2-7days in this patient?
A. Levothyroxine
B. Methimazole
C. Propylthiouracil
D. Potassium iodide

545.
1205.

1206.
546.
An ion that often shows expectorant action is
A. iodide
B. phosphate
C. fluoride
D. iodate
1207.
548.

549.
1208.

Oxytocin
A&C
A. increases contraction of the pregnant uterus at term
B. increases contractions of the normal nonpregnant uterus
C. sensitivity of the uterus to oxytocin gradually increases
during gestation
D. actions antagonized by estrogen
A 63 year old type 2 diabetic man with severe chronic obstructive D
pulmonary disease (COPD) drinks at least a sixpack of beer each
evening. Which of the following medications used to control
diabetes is most likely to product lactic acidosis in this patient?
A. Acarbose
B. Chlorpropamide
C. Insulin
D. Metformin

203
Pharmacy

1209.
550.

Increase motility of the gut


A. oxytocin
B. vasopressin
C. both
D. neither

1210.
551.

A 57 year old man has type 2 diabetes mellitus. Unfortunately, he A


also often has several drinks on Saturday evenings. Which of the
following antidiabetic medications would be most likely to
produce a severe disulfiram-like reaction?
A. Chlorpropamide
B. Glipizide
C. Glyburide
D. Repaglinide

1211.
552.

Pressor effect on the cardiovascular system


A. oxytocin
B. vasopressin
C. both
D. neither

1212.
553.

An 87 year old man has type 2 diabetes mellitus. Which of the


oral sulfonylurea antidiabetic agents would be preferred to treat this
patient?
A. Chlorpropamide
B. Glipizide
C. Glyburide
D. Tolbutamide

1213.
554.

Released from the posterior pituitary


A. oxytocin
B. vasopressin
C. both A & B
D. neither A nor B

1214.
555.

Which of the following insulin formulations can be administered D


without injection to maintain adequate glucose control in a 26year
old woman with type 1 diabetes mellitus?
A. Insulin Aspart, Insulin Aspart Protamine
B. Insulin Detemir
C. Insulin Glulisine
D. Insulin, Inhaled

1215.
557.

A 21 year old type 1 diabetic man presents at the emergency D


department in a state of severe diabetic ketoacidosis. Which of the
following insulin preparations would be most appropriate to
administer intravenously to this patient?
A. Insulin Aspart
B. Insulin Gluisine
C. Insulin Lispro
D. Regular Insulin

204
Pharmacy

1216.
558.

Hormones of the posterior pituitary:


A. Oxytocin
B. Vasopressin
C. both A & B
D. neither A nor B

559.
1217.

A 13 year old boy with type 1 diabetes mellitus is being B


successfully managed with an insulin therapeutic regimen that
utilizes different insulin preparations. Which of the following
insulin preparation is most likely being used in this young boy to
maintain basal insulin levels with once a day dosing?
A. Insulin Aspart
B. Insulin Glargine
C. Isophane Insulin
D. Regular Insulin

1218.
560.

Select the possible cause of the adverse effect Gastrointestinal


upset or nausea or bloating
A. estrogen excess
B. estrogen deficiency
C. progestin excess

1219.
561.

Causes increased volume of endocervical mucus and salt content A


greater than 1 percent which results in a ferning pattern when
dried on a slide.
A. Estrogenic
B. Progestational

562.
1220.

Fluid retention, edema, vascular headache,breast congestion, and


hypertension
A. estrogen excess
B. estrogen deficiency
C. progestin excess

1221.
563.

Results in some salt and water retention


A. estrogenic
B. progestational

1222.
564.

Increased appetite, change in libido, or fatigue


A. estrogen excess
B. estrogen deficiency
C. progestin excess

1223.
565.

Which of the following is a longlasting glucocorticoid with A


minimal mineralocorticoid activity that can be used for the
treatment of a 47 year old woman with rheumatoid arthritis?
A. Dexamethasone
B. Fludrocortisone
C. Hydrocortisone
D. Prednisone

205
Pharmacy

1224.
566.

Beta1 receptors in the heart are:


A. excitatory
B. inhibitory
C. both
D. neitherr

1225.
567.

In a 51 year old woman after surgical removal of a small D


carcinoma from her right breast, if the tumor tissue is positive for
both estrogen and progesterone receptors, she would be a
candidate for treatment with:
A. anastrazole
B. diethylstilbestrol
C. nafarelin
D. tamoxifen

568.
1226.

Bronchial muscles: what type of receptors are found here?


A. Alpha1 receptors
B. Beta2 receptors
C. Alpha 2 receptors
D. None of the above

1227.
569.

Which of the following is selective estrogen receptor modulator D


(SERM) with no estrogenic effects on endometrial tissue which
can be used in lieu of hormone replacement therapy (HRT) for the
prevention of osteoporosis in a 56 year old postmenopausal
woman?
A. Clomiphene
B. Danazol
C. Mifepristone
D. Raloxifene
Contraction of the radial muscle of the iris produces pupillary A
dilation and would therefore have which type of receptors.
A.
Alpha1 receptors
B.
Beta2 receptors
C.
Both A & B
D.
Neither A nor B

1228.
570.

1229.
571.

572.
1230.

Which of the following is a 5 alpha reductase inhibitor that would B


improve symptoms, reduce the risk of acute urinary retention, and
delay the need for a transurethral resection in a 55 year old man
with an enlarged prostate?
A.
Dehydroepiandrosterone (DHEA)
B.
Finasteride
C.
Flutamide
D.
Ketoconazole
The one principal area of integration of the autonomic nervous B
system is
A.
The spinal cord
B.
The Hypothalamus
C.
The medulla oblongata
D.
The Cortex

206
Pharmacy

1231.
574.

A 58 year old woman with Pagets disease complains of headache, A


backache, and pain in the lower extremities. Her plasma bone
alkaline phosphatase is markedly elevated. She could be treated
with
A.
B.
C.
D.

1232.
575.

The beneficial effects of anastrazole given to a 56 year old A


postmenopausal woman for treatment of estrogen receptor positive
advanced breast cancer with metastatic disease results from
blocking:
A.
B.
C.
D.

1233.
576.

1234.
577.

aromatase
estrogen receptors
gonadotropin receptors
progestin receptors

Which of the following statements are true with respect to


insulin?
A.

Only regular insulin is recommended for IV use

B.

Only regular insulin is recommended for IM use

C.

Adsorption of insulin to IV tubing can cause difficulties


in dosing adjectments

D.

All of the above

A 23 year old woman 28 weeks pregnant was given terbutaline to A


delay labor and drug X to induce fetal lung maturation to
prophylaxis against anticipated neonatal respiratory distress (RDS)
to the premature infant. Drug X is:
A.
B.
C.
D.

1235.
578.

calcium
calcitriol
cholecalciferol
ergocalciferol

betamethasone
clomiphene
leuprolide
magnesium sulfate

A 29 year old woman complains of intermittent heat intolerance,


tremor, tachycardia, sweats, and muscle weakness. Initially her
symptoms were managed with various drugs, including
propranolol. Her symptoms and treatment are charactereristic of:
A.
Addisons disease
B.
Cushings disease
C.
Diabetes
D.
Graves disease

207
Pharmacy

1236.
579.

A 49 year old woman has symptoms of excess adrenergic activity. B


Her temperature was elevated with marked flushing and sweating.
She had marked gastrointestinal symptoms, including nausea,
vomiting and diarrhea. Her blood pressure was 190/110 mm Hg
and heart rate was 110 bpm. She was given a slow intravenous
injection of propranolol to attenuate the acute symptoms and drug
Xto reduce the release of thyroid hormones. Drug X is:
A.
liothyronine
B.
potassium iodide
C.
propylthiouracil
D.
sodium iodide 131 I

1237.
580.

A 19 year old woman 10 weeks pregnant complains of heat D


intolerance. Her thyroid gland is nontender but slightly enlarged.
Thyroid function analyses reveal a serum thyroxine (T4) level of
14.5g/dL and a serum thyroid stimulating hormone
(TSH) level of 2.5U/mL . These results are normal
during pregnancy because of:
A.
decreased thyroid hormone catabolic enzymes
B.
decreased volume of distribution of thyroid hormone
C.
increased synthesis of T 3
D.
increased synthesis of thyroid binding globulin (TBG)

1238.
581.

A 39 year old man subjected to bilateral adrenalectomy will A


require hormone replacement therapy to avoid:
A.
Addisons disease
B.
Cushings disease
C.
Graves disease
D.
Hashimotos disease

1239.
582.

A 51 year old obese woman with type 2 diabetes mellitus and C


congestive heart failure has been managed for the past 4 years
with enalapril. She is being treated with Glucovance(glyburide
plus metformin) and is brought to the Emergency Department with
symptoms of hypothermia, hypotension, and resistant
bradyarrhythmias. These symptoms are most likely due to:
A.
Hepatic failure
B.
Keto acidosis
C.
Lactic acidosis
D.
Renal shutdown

1240.
583.

A moderately obese 59 year old woman with type 2 diabetes


mellitus is prescribed glipizide and recommended dietary control.
Which of the following is a frequent and serious problem with
glipizide treatment?
A.
Blood dyscrasia
B.
Hypersensitivity reaction
C.
Idiosyncratic reaction
D.
Hypoglycemia

208
Pharmacy

1241.
585.

If the signs and symptoms of acromegaly in a 45 year old woman D


are partially reversed by surgical removal of a pituitary tumor,
which of the following drugs could be used?
A.
Bromocriptine
B.
Cosyntropin
C.
Ganirelix
D.
Octreotide

1242.
586.

A 39 year old woman who has been smoking 2 packs of cigarettes D


daily for 20 years has also been taking combination oral
contraceptives (ethinyl estradiol plus norethindrone) during the
same period. She has an increased risk of developing:
A.
adrenal failure
B.
breast cancer
C.
polycystic ovary disease
D.
thromboembolism

1243.
587.

In a 54 year old postmenopausal woman with a family history of


breast cancer, which of the following can be given once weekly to
prevent osteoporosis?
A.
Alendronate
B.
Conjugated estrogens
C.
Calcitonin
D.
Estradiol+norgestimate

1244.
589.

A 70 year old woman is being treated with raloxifene for C


osteoporosis. There is an increased risk of her developing:
A.
breast cancer
B.
uterine cancer
C.
vein thrombosis
D.
atrophic vaginitis

1245.
590.

A 23 year old woman has failed to become pregnant after two C


years of unprotected intercourse. Which of the following would
be effective in treating infertility due anovulatory cycles?
A.
A combination of an estrogen and progestin
B.
Estrogen alone
C.
Clomiphene
D.
Raloxifene

1246.
591.

Which of the following is drugs is suitable for treating osteoporosis?


A.

Alendronate

B.
C.
D.

Estradiol
Tamoxifen
Ibuprofen

209
Pharmacy

1247.
592.

593.
1248.

1249.
594.

595.
1250.

596.
1251.

597.
1252.

Estrogen replacement therapy in menopausal women:


A.
restores bone loss accompanying osteoporosis
B.
may induce hot flashes
C.
may cause atrophic vaginitis
D.
is most effective if instituted at the first signs of
menopause
Measurements
of
cortisol
precursors
and
plasma
dehydroepiandrosterone sulfate confirm the diagnosis of
congenital adrenal hyerplasia in a child. This condition can be
effectively treated by:
A.
suppressing the release of ACTH
B.
administering an androgen antagonist
C.
administering metapyrone to decrease cortisol
synthesis
D.
removing the adrenal gland surgically
Osteoporosis is a major adverse effect caused by the
glucocorticoids. It is due to their ability to :
A.
increase the excretion of calcium
B.
inhibit absorption of calcium
C.
Both A & B
D.
Neither A nor B
A child with asthma is being treated effectively with an inhaled
preparation of beclomethasone dipropionate.
Which of the
following adverse effects is of particular concern?
A.
Hypoglycemia
B.
Hirsutism
C.
Growth suppression
D.
Cushing syndrome
A 12 year old girl with childhood history of asthma complained of
cough, dyspnea and wheezing after visiting a riding stable. Her
symptoms became so severe that her parent brought her to the
emergency room. Physical examination revealed diaphoresis,
dyspnea, tachycardia, and tachypnea. Her respiratory rate was 42
breaths per min, pulse rate 110 beats per minute, and blood
pressure 132/65mm Hg. Which of the following is the most
appropriate drug to rapidly reverse her bronchoconstriction?
A.
Albuterol by aerosol
B.
Cromolyn by inhaler
C.
Fluticasone by aerosol
D.
Theophylline orally
A 68 year old patient with cardiac failure is diagnosed with
ovarian cancer. She is started on cisplatin but becomes nauseous
and suffers from severe vomiting. Which of the following
medications would be most effective to counteract the emesis in
this patient without exacerbating her cardiac problem?
A.
Droperidol
B.
Domperidone
C.
Prochloperazine
D.
Ondansetron

210
Pharmacy

1253.
598.

A 45 year old woman distressed by her divorce, She has been


D
under tension, which led to overeating and she complains of
persitent heatburn and an unpleasant, acid like taste in her mouth.
The clinician suspects gastrointestinal reflux disease
and advises her to raise the head of her bed six to eight inches, not
to eat several hours before retiring, to avoid alcohol, and to eat
smaller meals. Two weeks later, she returns and says the
symptoms have subsided slightly but still are a concern. The
clinician prescribes:
A.
an antacid such as aluminium hydroxide
B.
dicyclomine
C.
an antianxiety agent such as alprazolam
D.
esomeprazole

1254.
599.

Which of the following agents interferes with most of the C


cytochrome P450enzymes and, thus, leads to many drug
interactions?
A.
Famotidine
B.
Omeprazole
C.
Cimetidine
D.
Sucralfate

1255.
600.

A couple celebrating their fortieth wedding anniversary is given a B


trip to Peru to visit Machu Picchu. Due to past experiences while
traveling, they ask their doctor to prescribe an agent for diarrhea.
Which of the following would be effective?
A.
B.
C.
D.

1256.
601.

Omerprazole
Loperamide
Famotidine
Lorazepam

A 66 year old man complained of decreased libido and difficulty D


maintaining an erection. He is currently taking tamsulosin for
benign prostatic hyperplasia. He is concerned about the use of
drugs to restore sexual function, particularly about the need to
time therapy with anticipated sexual activity. Which one of the
following therapeutic options is indicated for this patient?
A.

PDE5 inhibitors contraindicated because of treatment


with tamsulosin.

B.

Sildenafil is indicated because of its long duration of


action.

C.

Vardenafil is indicated because its absorption is not


affected by food.

D.

Tadalafil is indicated because it


concomitantly with tamsulosin.

can

be

used

211
Pharmacy

603.
1257.

1258.
604.

605.
1259.

606.
1260.

607.
1261.

1262.
608.

A 58 year old male has been effectively treated for paget disease
for approximately six months. He is now beginning to experience
renewed bone pain and radiologic evidence of advancing disease.
Which of the following drugs is most likely to have resulted in this
failure of therapy?
A.
Alendronate
B.
Calcitonin
C.
Dihydrotachysterol
D.
Ergocalciferol
Which one of the following patients is least likely to require
antimicrobial treatment tailored to the individuals condition?
A.
Patient undergoing cancer chemotherapy
B.
Patient with kidney disease
C.
Elderly patient
D.
Patient with hypertension
In which one of the following clinical situations is the prophylactic
use of antibiotics not warranted?
A.
Prevention of meningitis among individuals in close
contact with infected patients.
B.
Patient with hip prosthesis who is having a tooth
removed.
C.
Presurgical treatment for implantation of a hip
prosthesis.
D.
Patient who complains of frequent respiratory illness
Which one of the following is the best route of administration
schedule for treatment with aminoglycosides based on the drugs
concentration dependent killing property?
A.
Oral every 8 hours
B.
Oral every 24 hours
C.
Parenterally by continuous intravenous infusion
D.
Parenterally every 24 hours
An elderly diabetic patient is admitted to the hospital with
pneumonia. The sputum culture stains for a gram negative rod.
The patient is started on intravenous ampicillin. Two days later,
the patient is not improving, and the microbiology laboratory
reports the organism to be a -lactamase producing H.influenzae.
What course of treatment is indicated?
A.
Continue with the intravenous ampicillin
B.
Switch to intravenous cefotaxime
C.
Switch to oral vancomycin
D.
Add gentamicin to the ampicillin therapy

A 70 year old alcoholic male with poor dental hygiene is to have B


his remaining teeth extracted for subsequent dentures. He has
mitral valve stenosis with mild cardiac insufficiency and is being
treated with captopril, digoxin, and furosemide. The dentist
decides that his medical history warrants prophylactic antibiotic
therapy prior to the procedure and prescribes which of the
following drugs?
A.
Vanomycin

212
Pharmacy

609.
1263.

1264.
610.

611.
1265.

1266.
612.

1267.
613.

614.
1268.

B.
Amoxicillin
C.
Tetracycline
D.
Cotrimoxazole
A patient with degenerative joint disease is to undergo insertion of
a hip prosthesis. To avoid complications due to postoperative
infection, the surgeon will pretreat this patient with an antibiotic.
This hospital has a significant problem with methicillin resistant
Staphylococcus aureus. Which of the following antibiotics should
the surgeon select?
A.
Ampicillin
B.
Imipenem/cilastain
C.
Gentamicin/piperacillin
D.
Vancomycin
A 25 year old male returns home from a holiday in the Far East
and complains of three days of dysuria and a purulent urethral
discharge. You diagnose this to be a case of gonorrhea. Which of
the following is appropriate treatment?
A.
Ceftriaxone IM
B.
Penicillin G IM
C.
Gentamicin IM
D.
Piperacillin/Tazobactam IV
A patient with gunshot wound to the abdomen, which has resulted
in spillage of intestinal contents, is brought to the emergency
room. Which antibiotic would you select to effectively treat an
infection due to Bacteroides fragillis?
A.
Aztreonam
B.
Clindamycin
C.
Gentamicin
D.
Azithromycin
A pregnant woman was hospitalized and catheterized with a foley
catheter. She developed a urinary tract infection caused by
Pseudomonas aeruginosa, and was treated with gentamicin.
Which of the following adverse effects was a risk to the fetus
when the woman was on gentamicin?
A.
Skeletal deformity
B.
Hearing loss
C.
Teratogenesis
D.
Blindness
Children younger than eight years of age should not receive
tetracyclines, because these agents:
A.
cause rapture of tendons.
B.
do not cross into the cerebrospinal fluid
C.
are not bactericidal
D.
deposit in tissues undergoing calcification
A 46 year old woman is in the intensive care unit for treatment of
a vancomycin resistant strain of Enterococcus faecium caused
bacteremia. You want to limit the risk of drug interactions in this
woman who is receiving five other medications. Which one of the
following antibiotics would you choose?
A.
Azithromycin

213
Pharmacy

B
1269.

B
1270.

B
1271.

1272.

1273.

B
1274.

214
Pharmacy

feet. You suspect that he might be deficient in which one of the


following vitamins?
A.
Ascorbic acid
B.
Niacin
C.
Pyridoxine
D.
Calcitriol
621.
A
35
year
old male, formerly a heroin abuser, has been on D
1275.
methadone maintenance for the last 13 months. 2 weeks ago, he
had a positive PPD test, and a chest radiograph showed evidence
of right upper lobe infection. He was started on standard
antimycobacterial therapy. He has come to the emergency
department complaining of withdrawal symptoms. Which of the
following antimycobacterial drugs is likely to have caused this
patients acute withdrawal reaction?
A.
Ethambutol
B.
Isoniazid
C.
Pyrazinamide
D.
Rifampin
622.
A 25 year old male AIDS patient has a fever of 1020F and C
1276.
complains of severe headaches during the past week. Staining of
his CSF with Indian ink reveals Cryptococcus neoformans. The
patient is admitted to the hospital and is treated with:
A.
intravenous amphotericin B plus flucystosine
B.
oral ketoconazole
C.
intrathecal amphotericin B
D.
oral fluconazole
A
24-year
old
AIDS patient need to be put on Pneumocystis pneumo- B
623.
1277.
nia (PCP) and Cerebral Toxoplasmosis prophylaxis. Which of the
following prophylactic agents is appropriate choice for the prevention of both ?
A. Nitrofurantoin
B. Co trimoxazole
C. Norfloxacin
D. Methenamine
624.
1278.

625.
1279.

A 22 year old male has been treating his athletes foot with an D
overthecounter drug without much success. Upon examination, it
is found the nail bed of both great toes is infected. Which one of
the following antifungal agents would be most appropriate for this
patient?
A.
Caspofungin
B.
Fluconazole
C.
Griseofulvin
D.
Terbinafine
A 36 year old male of Lebanese ancestry is being treated for vivax A
malaria. He experiences severe fatigue, back pain, and darkened
urine. Which one of the following antimalarial drugs is most
likely to have caused his symptoms?
A. Primaquine
B. Quinine

215
Pharmacy

1280.
626.

627.
1281.

1282.
628.

629.
1283.

1284.
630.

631.
1285.

C. Pyrimethamine
D. Chloroquine
Which of the following drugs is recommended for the treatment of
severe, multi drug resistant falciparum malaria?
A. Artemisinin
B. Chloroquine
C. Quinine
D. Sodium stibogluconate
A 22 year old man, who frequently backpacks, complains of
diarrhea and fatigue. Examination of stool specimens shows
binucleate organisms with four flagellae. Which one of the
following drugs would be effective in treating this patients
infestation?
A. Metronidazole
B. Quinidine
C. Pentamidine
D. Sulfadoxine
A 48 year old immigrant from Mexico presents with seizures and
other neurologic symptoms. Eggs of Taenia solium are found
upon examination of a stool specimen. A magnetic resonance
image of the brain shows many cysts, some of which are calcified.
Which one of the following drugs would be of benefit to this
individual?
A. Ivermectin
B. Pyrantel pamoate
C. Albendazole
D. Diethylcarbamazine
A 56 year old man from South Am erica is found to be parasitized
by both schistosomes and Taenia solium the pork tapeworm.
Which of the following antihelmintic drugs would be effective for
both infestations?
A. Albendazole
B. Ivermectin
C. Mebendazole
D. Praziquantel
A 30 year old male patient with an HIV infection is being treated
with a HAART regimen. Four weeks after initiating therapy, he
comes to the emergency department complaining of severe flank
pain, nausea and frequent urination. Which one of the following
drugs is most likely the cause of his symptoms?
A. Zidovudine
B. Nelfinavir
C. Indinavir
D. Efavirenz
Chills, fever, and muscle aches are common reactions to which
one of the following antiviral drugs?
A. Acyclovir
B. Ganciciovir
C. Amantadine
D. Foscarnet

216
Pharmacy

632.
1286.

1287.
633.

634.
1288.

1289.
635.

636.
1290.

1291.
637.

A 25 year old man is diagnosed with HIV, and therapy is initiated.


After the first week of therapy, the patient complains of
headaches, irritability and nightmares.
Which one of the
following antiretroviral drugs is most likely to be causing these
symptoms?
A. Efavirenz
B. Indinavir
C. Lamivudine
D. Nevirapine
A patient with colonic cancer is being treated with 5fluorouracil as
well as leucovorin (N5, N10methylene tetrahydrofolate). The
rationale for administering the coenzyme depends on its being
essential for:
A. conversion of 5fluorouracil to fluorodeoxyuridylic acid
(FdUMP)
B. protection against the anemia caused by 5fluorouracil
treatment.
C. The inhibition of thymidylate synthase by FdUMP
D. Prolongation of the antitumor effect of 5fluorouracil
Hydration and/or diuresis can prevent the renal toxicity associated
with:
A. cisplatin
B. chlorambucil
C. tamoxifen
D. gemcitabine
A patient is being treated with allopurinol to control
hyperuricemia resulting from cancer chemotherapy. Which of the
following would have to have its dose reduced to prevent toxicity?
A. 5 Fluorouracil
B. 6 Mercaptopurine
C. 6 Thioguaine
D. Fludarabine
A 45 year old male who received a renal transplant three months
previously and is being maintained on methylprednisolone,
cyclosporine, and mycophenolate mofetil is found to have
increased creatinine levels, indicating possible rejection. Which of
the following courses of therapy would be appropriate?
A. Increased dose of methylprednisolone
B. Hemodialysis
C. Treatment with muromonabCD3
D. Treatment with sirolimus

A 23 year old female suffering from grand mal epilepsy is being D


controlled with phenytoin. She is a candidate for a renal
transplant. Which agent/agents belong to the group of
immunosuppressants for her therapy?
A. Mycophenolate mofetil
B. Sirolimus
C. Cyclosporine
D. All of the above

217
Pharmacy

1292
638.

1293.
639.

1294.
640.

1295.
641.

1296.
642.

1297.
643.

644.
1298.

Which of the following drugs used to prevent allograft rejection


can cause hyperlipidemia?
A. Azathioprine
B. Basiliximab
C. Tacrolimus
D. Sirolimus
Which of the following drugs specifically inhibit calcineurin in the
activated T lymphocytes?
A. Daclizumab
B. Tacrolimus
C. Prednisone
D. Sirolimus
In which of the following conditions would aspirin be contraindicated?
A. Myalgia
B. Fever
C. Peptic ulcer
D. Rheumatoid arthritis
Which one of the following statements concerning COX2
inhibitors is correct?
A. The COX2 inhibitors show greater analgesic activity than
traditional NSAIDs.
B. The COX2 inhibitors decrease platelet function
C. The COX2 inhibitors do not affect the kidney.
D. The COX2 inhibitors show antiinflammatory activity
similar to that of the traditional NSAIDs.
An 8 year old girl has a fever and muscle aches from a
presumptive viral infection. Which one of the following drugs
would be most appropriate to treat her symptoms?
A. Acetaminophen
B. Aspirin
C. Celecoxib
D. Codeine

A 70 year old man has a history of ulcer disease. He has recently C


experienced swelling and pain in the joints of his hands. His
physician wants to begin therapy with a nonsteroidal
antiinflammatory drug. Which one of the following drugs might
also be prescribed along with the NSAID in order to reduce the
risk of activating this patients ulcer disease?
A. Allopurinol
B. Colchicine
C. Misoprostol
D. Probenecid
Dihydroergotamine:
C
A. causes vasoldilation
B. exerts its actions by binding to specific ergotamine
receptors.
C. is useful in treating acute migraine headaches.
D. is useful for maintaining uterine muscle tone during
pregnancy.

218
Pharmacy

1299.
645.

646.
1300.

647.
1301.

648.
1302.

1303.
649.

650.
1304.

A 43 year old ships captain complained of seasonal allergies. D


Which one of the following would be indicated?
A. Cyclizine
B. Doxepin
C. Doxylamine
D. Fexofenadine
Which one of the following statements concerning H 1 A
antihistamines is correct?
A. Second generation H 1 antihistamines are relatively free of
adverse effects
B. Because of the established longterm safety of
first generation H 1 antihistamines, they are the first choice
for initial therapy.
C. The motor coordination involved in driving an automobile
is not affected by the use of first generation H 1 antihistamines.
D. H 1 antihistamines can be used in the treatment of acute
anaphylaxis.
Which one of the following drugs could significantly impair the
ability to drive an automobile?
A. Diphenhydramine
B. Ergotamine
C. Fexofenadine
D. Ranitidine
A patient is given by aerosol an adrenergic agonist. Which of the
following agents would be MOST LIKELY to produce
tachycardia and nervousness?
A. albuterol
B. isoproterenol
C. terbutaline
D. none of the above
The following statements relate to specific properties of drugs
used in the treatment of asthma. Select the correct statement.
A. The oxidation and demethylation of theophylline occurs in
the liver.
B. Albuterol is a beta adrenergic blocker with specificity for
beta2 receptors.
C. Ipratropium bromide is a quaternary ammonium
anticholinergic drug that is well absorbed in the GI tract,
and is associated with dry mouth.
D. Beclomethasone enhances the release of chemical mediators
of bronchospasm while diminishing the effectiveness
of beta2 adre nergic agonists on their receptors.
Choose the best statement(s) that describe the use of
corticosteroids in the treatment of asthma.
A. corticosteroids relax smooth muscle
B. corticosteroids inhibit the inflammatory response.
C. Corticosteroids reduce responsiveness of the patients to
beta agonists.
D. In acute asthma, corticosteroids increase airway
obstruction which is why they should only be used for
chronic treatment.

219
Pharmacy

1305.
651.

Which of the following drugs used in the treatment of asthma is


classified as a phosphodiesterase inhibitor?
A. Ipratropium bromide
B. Aminophylline
C. Metaoriterenol
D. Cyclosporine

1306.
652.

In the management of asthma which drug is thought to be useful


only as a prophylactic and acts by preventing the release of
mediators of bronchoconstriction from the mast cell?
A. beclomethasone
B. cromolyn
C. metaproterenol
D. albuterol

1307.
653.

Peptic ulcer disease consists of many syndromes that produce


ulceration of the upper GI tract. Several pharmacological treatment
options are available. Choose the treatment option and the
associated correct drug example?
A. Acid neutralization: calcium carbonate
B. Cytoprotection: bismuth
C. Antibacterial: bismuth
D. All of the above
Drugs are available to treat peptic ulcer disease. Choose the
treatment option and the associated correct drug example?
A. Interference with acid secretion : Pirenzepine
B. Acid neutralization : Aluminum hydroxide
C. Cytoprotection: Bismuth
D. All of the above
The secretory elements of the gastric mucosa are the parietal cells.
When parietal cells are activated by which one(s) of the following
drug(s), they release acid and intrinsic factor?
A. acetylcholine
B. histamine
C. gastrin
D. All of the above
What is the mechanism of action of cimetidine in producing a
decrease in gastric secretion?
A. Cimetidine interferes with the gastric acid pump.
B. Cimetidine blocks the action of intracellular cAMP by
preventing protein phosphorylation.
C. Cimetidine competes with histamine for H2receptor sites.
D. Cimetidine competes with gastrin for H2receptor sites.
A 35 year old male complains of stomach distress and is
diagnosed with peptic ulcer disease. He had read that taking one
aspirin per day was useful in preventing a second heart attack.
Although he had not had a first attack, he was not taking chances
and figuring that if one aspirin was good four would be better, he
takes four per day. His physician suggests that he cut back on the
aspirin and decides, to be on the safe side, to prescribe an H2

1308.
654.

655.
1309.

656.
1310.

657.
1311.

220
Pharmacy

658.
1312.

659.
1313.

1314.
660.

661.
1315.

662.
1316.

1317.
663.

blocker. After taking the medica tion for some time, the patient
observes some swelling of his breasts and calls his doctor. Which
agent was likely the cause of this response?
A. Ranitidine
B. Cimetidine
C. Atropine
D. Pirenzepine
Which of the following antigastric ulcer agent works by binding to
the protein substrate of the mucosa, forming a gel that coats the
ulcer?
A. Bismuth compounds
B. Misoprostol
C. Sucralfate
D. Metroclopramide
Choose the antiulcer agent that works by blocking the generation
of protons by the parietal cell?
A. Sucralfate
B. Omperazole
C. Bismuth compounds
D. Metroclopramide
Choose the correct relationships between the following antacids
and their effects.
A. Calcium carbonate : constipation
B. Aluminum hydroxide : diarrhea
C. Magnesium hydroxide : diarrhea
D. A & C
Which of the following drugs is NOT classified as an H2 receptor
blocker?
A. Famotidine
B. Terfenadine
C. Nizatidine
D. None of the above
Reflux esophagitis responds best to?
A. Famotidine
B. Cimetidine
C. Omeprazole
D. Sucralfate
Dry mouth and skin are common side effects of many
antihistamines. These side effects are most likely due to?
A. Interaction with the histamine receptor
B. Interaction with the adrenergic receptor
C. Interaction with the serotonergic receptor
D. Interaction with the muscarinic receptor

1318.
664.
An important, potentially fatal side effect is associated with the
coadministration of terfenadine and the following drug.
A. Aspirin
B. Clofibrate
C. Warfarin
D. Erythromycin

221
Pharmacy

1319.
665.

1320.
666.

1321.
667.

1322.
668.

1323.
669.

1324.
670.

1325.
671.

1326.
672.

1327.
673.

1328.
674.

Which is the most progestational and the most antiestrogenic


progestin used in oral contraceptives?
A. Norethynodrel
B. Norethindrone
C. Norethindrone acetate
D. Norgestrel
Oxytocin is the most effective in stimulating the uterus to contract
during which time?
A. The 1st trimester
B. The 2nd month of pregnancy
C. The 2nd trimester
D. The 3rd trimester
Output of ACTH is controlled by a shortchain peptide hormone
from the hypothalamus called CRF (corticotropin releasing factor).
Release of CRF (and subsequently ACTH) is influenced by:
A. positive feedback by adrenal androgens
B. negative feedback by cortisol (hydrocortisone)
C. neural input from other parts of the CNS
Based on what you have learned so far, at what time would you
expect an intravenous dose of cortisol to have the greatest effect
on a night sleepers physiology:
A. at 8 a.m.
B. at 4 p.m.
C. at 10 p.m.
The syndrome of excess mineralocorticoid is (choose one letter):
A. Addisons disease
B. Cushings syndrome
C. Conns syndrome
In Conns syndrome, the most likely findings would be:
A. hypernatremia, hypokalemia, acidosis, hypotension
B. hyponatremia, hypokalemia, alkalosis, hypertension
C. hyponatremia, hypokalemia, alkalosis, hypotension
D. hypernatremia, hypokalemia, alkalosis, hypertension
Serum levels responsive to fluctuations in ACTH levels:
A. aldosterone
B. 11desoxycorticosterone
C. both A & B
D. neither A nor B
Effective orally as a swallowed tablet:
A. aldosterone
B. 11desoxycorticosterone
C. both A & B
D. neither A nor B
Potassium sparing diuretic which competitively blocks aldosterone action:
A. triamterene
B. spironolactone
C. both A & B
D. neither A nor B
Least suppression of hypothalamic pituitary adrenal axis:
A. 50 mg of prednisolone, orally every other morning

B&C

222
Pharmacy

1329.
675.

676.
1330.

1331.
677.

678.
1332.

679.
1333.

680.
1334.

681.
1335.

B. 25 mg of hydrocortisone, orally every six hours


C. the effects of these regimens on HPA axis suppression are
equivalent
As an alternative to systemic steroid therapy, use of ACTH
A. produces less HPA axis suppression
B. causes worsening of acne and hirsutism greater than with
glucocorticoids
C. causes less dermal atrophy than glucocorticoids
D. all of the above
Primary adrenal insufficiency (Addisons disease) usually
requires:
A. Both glucocorticoid and mineralcorticoid in replacement
dosages
B. Doubling or tripling glucocorticoid during infections or
other stresses
C. Glucocorticoid and high salt diet in mild cases
D. Mineralcorticoid alone
Excess of adrenal steroids possessing mineralcorticoid properties
in addition to glucocorticoid properties has a tendancy to cause the
additional problems of:
A. de novo hypertension or exacerbation of preexisting
hypertension
B. muscle weakness or paralysis
C. hypokalemia and increased susceptibility to digitalis toxicity
D. all of the above
Symptoms of adrenal insufficiency resulting from withdrawal of
chronic glucocorticoid therapy:
A. Depression, fever, myalgia, arthralgia, malaise
B. Pseudotumor cerebri and papilledema
C. Reduced adrenal reserve for up to 2 years in response to
stress
D. All of the above
Absolute contraindications to glucocorticoid therapy include:
A. Herpes simplex keratitis (dendritic corneal ulcers)
B. Gastric ulcers (active)
C. Diseases responsive to safer modalities of therapy
D. All of the above
Parkinsonism is associated with degeneration of which neuronal
pathway:
A. the mesolimbic dopamine pathway
B. the mesocortical dopamine pathway
C. the nigrostriatal dopamine pathway
D. the reticular formation
The neurotransmitter of primary importance in Parkinsons
disease:
A. Serotonin
B. GABA
C. Dopamine
D. Noradrenaline

223
Pharmacy

1336.
682.

Symptoms of Parkinsons disease include:


A. Tremor
B. Rigidity
C. Bradykinesia
D. All of the above

1337.
683.

The primary treatment strategy in treatment of Parkinsons disease is:


A. Block dopamine receptors
B. Pharmacologically replace dopamine
C. Foster regrowth of dopamine receptors.
D. Increase dopamine synthesis

1338.
684.

L Dopa used in treatment of Parkinsons disease for all following


reasons EXCEPT:
A. It can cross blood brain barrier and dopamine cant
B. It is the metabolic precursor of dopamine
C. L Dopa is rapidly metabolized to dopamine by Dopa
decarboxylase
D. Dopamine is toxic

685.
1339.

The primary goal in the treatment of Parkinsonism is the


stimulation of dopamine receptors in what brain region:
A. Substantia nigra
B. Neostriatum
C. Cerebellum
D. Cortex
Carbidopa
A. is a dopamine agonist
B. is a dopamine antagonist
C. Crosses the blood barrier
D. Inhibits dopa decarbioxylase
Use of Carbidopa does all of the following EXCEPT:
A. Inhibits Dopamine catabolism in the brain
B. Blocks conversion of L Dopa to dopamine in periphery
C. Enables the use of lower doses of L Dopa
D. Increases tolerance of L Dopa therapy by decreasing side effects.
Side effects of L dopa include:
A. Nausea and vomiting
B. Postural hypotension
C. Arrhythmias
D. All of the above
Problems of long term therapy with L Dopa can include all of the
following EXCEPT:
A. Hirsuitism
B. Hypersexulality
C. Hallucinations
D. Fluctuations in efficacy
Drug holidays in Parkinsonism treatment
A. Can improve efficacy of L Dopa treatment
B. Are a period of time when the drug therapy is curtailed
C. Are weekend spent in ingesting illicit substances
D. A and B

1340.
686.

1341.
687.

1342.
688.

1343.
689.

1344.
690.

This is the terminal field


of Nigrostriatal pathway.

224
Pharmacy

1345.
691.

Direct dopamine agonists:


A. Bromocriptine
B. Pramipexole
C. Pergolide
D. All of the above

692.
1346.

Selegiline does all except:


A. Inhibits dopamine catabolism in the brain
B. Blocks conversion of L dopa to Dopamine in periphery
C. Inhibits MAOB
D. Can increase efficacy of L Dopa
Other classes of drugs used in treatment of Parkinsonism:
A. Amantadine
B. Procyclidine
C. Both A & B
D. Neither A nor B
Anticholinergic drug used in treatment of Parkinsonism:
A. Benztropine
B. Diphenhydramine
C. Trihexyphenidyl
D. Both A & C

695.
1349.

Acts at the level of the muscle to decrease Ca ++ release from the


sarcoplasmic reticulum:
A. Baclofen
B. Diazepam
C. Dantrolene
D. Amantadine

1350.
696.

Enhances GABA ergic transmission at spinal and supra spinal


levels:
A. Baclofen
B. Diazepam
C. Dantrolene
D. Diphenhydramine

697.
1351.

GABA B agonist that act at the level of spinal chord:


A. Baclofen
B. Diazepam
C. Dantrolene
D. Benztropine
Major side effects include generalized muscle weakness:
A. Baclofen
B. Diazepam
C. Dantrolene
D. Phenytoin
Side effects include hepatotoxicity:
A. Baclofen
B. Diazepam
C. Dantrolene
D. None of the above

693.
1347.

694.
1348.

698.
1352.

699.
1353.

225
Pharmacy

700.
1354.

1355.
701.

702.
1356.

1357.
703.

704.
1358.

705.
1359.

706.
1360.

707.
1361.

1362.
708.

709.
1363.

Useful in the treatment of malignant hyperthermia:


A. Baclofen
B. Diazepam
C. Dantrolene
D. None of the above
Useful in both grand-mal and petit-mal seizures:
A. Ethosuccimide
B. Va lproic acid
C. Phenytoin
D. Carbamazepine
Potential for dispositional interactions with other drugs:
A. Ethosuccimide
B. Diazepam
C. Gabapentin
D. Carbamazepine
Most sedating:
A. Pentobarbital
B. Diazepam
C. Gabapentin
D. Carbamazepine
Side effects include hirsuitism and gingival hyperplasia:
A. Phenytoin
B. Diazepam
C. Gabapentin
D. Carbamazepine
Effective in treating Petit-mal seizures:
A. Ethosuccimide
B. Diazepam
C. Gabapentin
D. Carbamazepine
Anticonvulsants that inhibit T type Ca ++ channels:
A. Ethosuccimide
B. Diazepam
C. Gabapentin
D. Carbamazepine
Anticonvulsants that augment GABAergic neurotransmission:
A. Clonazepam
B. Tiagabine
C. Both A & B
D. Neither A nor B
Anticonvulsants with a mechanism of action that involves
prolonging inactivation of sodium channels:
A. Phenytoin
B. Carbamazepine
C. Va lproic acid
D. All of the above
What is the major use of Clonazepam?
A. Anticonvulsant
B. Fast acting anxiolytic
C. Longer acting anxiolytic
D. None of the above

226
Pharmacy

1364.
710.

What is the major use of alprazolam?


A. Anticonvulsant
B. Fast acting anxiolytic especially for panic attacks
C. Longer acting anxiolytic for generalized anxiety
D. None of the above

1365.
711.

What is the major use of Triazolam?


A. Anticonvulsant
B. Fast acting anxiolytic especially for panic attacks
C. Longer acting anxiolytic for generalized anxiety
D. Hypnotic
What is the major use of midazolam?
A. Anticonvulsant
B. Fast acting anxiolytic especially for panic attacks
C. Longer acting anxiolytic for generalized anxiety
D. Preanesthetic
Which of the following will cross the placenta?
A. Ethanol
B. Diazepam
C. Both A & B
D. Neither A nor B
Which of the following is the longest acting benzodiazepine?
A. Flurazepam
B. Diazepam
C. Midazolam
D. Flumazenil
Which anxiolytic is least likely to produce sedation?
A. Alprazolam
B. Buspirone
C. Diazepam
D. Lorazepam
Antagonist for the treatment of overdose of benzodiazepines:
A. Flumazenil
B. Flurazepam
C. Ethanol
D. Pentobarbitone
Tolerance and physiological dependance occur for which of the
following:
A. Phenobarbitone
B. Flumazenil
C. Buspirone
D. All of the above
Which of the following drugs are currently used in treatment of
anxiety?
A. Phenobarbitone
B. Flumazenil
C. Both A & B
D. Neither A nor B
Which of the following drugs induce microsomal enzymes?
A. Phenobarbitone

712.
1366.

1367.
713.

1368.
714.

1369.
715.

1370.
716.

1371.
717.

718.
1372.

1373.
719.

227
Pharmacy

720.
1374.

721.
1375.

722.
1376.

723.
1377.

724.
1378.

1379.
725.

726.
1380.

B. Ethanol
C. Both A & B
D. Neither A nor B
Glyceryl trinitrate in therapeutic doses may produce:
A. Constriction of cerebral vessels
B. Reflex bradycardia
C. Tolerance
D. None of the above
For the treatment of migraine headaches, 31 year old woman was
using a nasal spray that causes a selective vaso constriction of the
carotid vascular bed by stimulating 5HT 1D receptors. The drug is:
A. Amitryptilline
B. Dihydroergotamine
C. Methysergide
D. Sumatriptan
If a 39 year old woman with open angle glaucoma, is treated with
a drug combination consisting of a non selective beta blocker and
an analogue of prostaglandin F 2 alpha. The drug combination is:
A. Atenolol+Alprostadil
B. Atenolol+Epoprostenol
C. Propranolol+ Epoprostenol
D. Timolol+latanoprost
In a 41 year old woman with chronic asthma uses an albuterol
inhaler and takes a daily medication that selectively interferes with
leukotriene synthesis. The drug is most likely:
A. Cromolyn
B. Zileuton
C. Ipratropium
D. Zafirlukast
In a 36 year old woman treated with a 5HT 3 antagonist for post
operative nausea and vomiting, the drug is:
A. Dronabinol
B. Hydroxyzine
C. Ondansetron
D. Prochlorperazine
Which of the following medications is given by oral inhalation for
the treatment of cholinergic mediated bronchospasms associated
with chronic obstructive pulmonary disease in a 36 year old
woman who is a known smoker?
A. Ipratropium
B. Albuterol
C. Atropine
D. Monteleukast
Which of the following medications would be most appropriate to
dilate bronchi during an acute asthmatic attack for a boy who is
brought to the Emergency?
A. Ipratropium
B. Cromolyn
C. Atropine
D. Terbutaline

228
Pharmacy

727.
1381.

1382.
728.

1383.
729.

730.
1384.

731.
1385.

1386.
732.

1387.
733.

1388.
734.

In a 31 year old pregnant, near term woman being prepared for


labour induction with oxytocin, which of the following agents is
appropriate for cervical ripening induction?
A. Alprostadil
B. Dinoprostone
C. Epoprostenol
D. Ergonovine
To prevent nausea and vomiting during a cruise on a ship, the
Physician prescribes a patch, to be applied behind her ear 4 hours
prior to departure, the acive ingredient of the patch is:
A. Scopolamine
B. Diphenhydramine
C. Granisetron
D. Promethazine
Which of the following agents is most likely to cause bronchoconstriction in a 55 year old man with chronic asthma?
A. Bethanechol
B. Desloratadine
C. Fexofenadine
D. Prazosin
61 year old woman, treated with Theophylline (800mg/day) and
fluticasone for chronic asthma, stabilized and tolerating
medications well. She then develops a gastric ulcer, for which
she begins therapy with Cimetidine, Bismuth and Amoxycillin.
Within a few days she complains that her heart is racing ,
develops arrhythmias, and fatal status epilepticus. What is the
most likely explanation for this series of events.
A. Amoxycillin hypersensitivity
B. Cimetidine inhibition of theophylline metabolism
C. Synergistic action of bismuth and fluticasone
D. Stress response associated with asthma and ulcer.
Which of the following are bulk forming laxatives?
A. Cascara
B. Dioctyl sodium sulfosuccinate
C. Both
D. Neither
Which of the following are bulk forming laxatives?
A. Bran
B. Methyl cellulose
C. Both
D. Neither
Which of the following is an osmotic catharitic?
A. Magnesium sulfate
B. Phenolphthalein
C. Castor oil
D. Plantago
To avoid diarrhoea or constipation, combined therapy by which of
the following pairs of antacids would be appropriate?
A. Aluminium hydroxide and milk of magnesia
B. Aluminium hydroxide and calcium carbonate

229
Pharmacy

735.
1389.

1390.
736.

737.
1391.

1392.
738.

1393.
739.

1394.
740.

1395.
741.

742.
1396.

743.
1397.

C. Magnesium carbonate and magnesium trisilicate


D. All of the above
A phosphate deficiency in patients on limited phosphate diets is
more likely in patients taking which of the following?
A. Sodium bicarbonate
B. Aluminium hydroxide
C. Calcium carbonate
D. Magnesium trisilicate.
Can produce a deficiency of Vitamins A, D and K
A. Atropine
B. Magnesium hydroxide
C. Mineral oil
D. Aluminium hydroxide
Which of the following antacids would be most likely to produce
depressed mental function in a patient with poor renal function?
A. Sodium bicarbonate
B. Aluminium hydroxide
C. Calcium carbonate
D. Magnesium trisilicate.
A nonprescription drug to treat heartburn:
A. Famotidine
B. Aluminium hydroxide
C. Cimetidine
D. Nizatidine
Which of the following antacids cause acid rebound?
A. Sodium bicarbonate
B. Aluminium hydroxide
C. Simethicone
D. Magnesium hydroxide
Omeprazole is efficacious in the treatment of :
A. Duodenal ulcers
B. Gastric ulcers
C. Zollinger Ellison syndrome
D. All of the above
Side effects of cimetidine include:
A. Male sexual dysfunction
B. Gynecomastia
C. Decrease metabolism of warfarin and theophylline
D. All of the above
Which of the following medications is recommended for the
prevention of nephrolithiasis due to hypercalciuria in a 56 year old
man?
A. Bumetanide
B. Ethacrynic acid
C. Hydrchlorthiazide
D. Frusemide
56 year old woman is given frusemide for the treatment of
peripheral edema associated with a nephrotic syndrome. What is
the molecular target for frusemide in this patient?
A. Aldosterone receptor

230
Pharmacy

1398.
744.

745.
1399.

746.
1400.

747.
1401.

748.
1402.

1403.
749.

B. Carbonic anhydrase
C. NaCl transporter
D. Na+/K+/Cl cotransport system
56 year old man typically experiences altitude sickness when he
hikes at high altitudes. Which of the medications would be
appropriate for him to take prophylactically at least 24 hours prior
to initiating his hiking?
A. Acetazolamide
B. Amiloride
C. Bumetanide
D. Ethacrynic acid
A 16 year old asthmatic girl with severe bronchospasms, which of
the following beta 2 selective agonists can be administered S/C to
treat severe bronchospasms in this patient?
A. Albuterol
B. Terbutaline
C. Salmeterol
D. Levalbuterol
A 14 year old girl whose asthma is inadequately controlled with
inhaled corticosteroids is given an add on therapy with a
leukotriene receptor antagonist. Which of the following agents
was likely selected for add on therapy for this patient?
A. Nedocromil
B. Theophylline
C. Zafirlukast
D. Tiotropium
A 14 year old girl with exercise induced asthma always takes an
inhaled medication to inhibit mast cell degranulation prior to
taking her morning run. Which of the following prophylactic
medications was most likely in this patient?
A. Budesonide
B. Cromolyn
C. Zafirlukast
D. Tiotropium
32 year old man uses oral inhalation of triamcinolone acetonide
several times a day to decrease pulmonary inflammation
associated with bronchial asthma. Which of the following ADR is
most likely to be seen?
A. Cataracts
B. Diabetes Mellitus
C. Fluid retention
D. Oral Candidiasis
44 year old woman was given an I/M dose of methyl prednisolone
for the treatment of severe asthma exacerbation in the emergency
dept. It will increase the synthesis of which of the following
proteins in this patient?
A. Cyclooxygenase2
B. Interleukin 1
C. Lipocortin
D. Lipooxygenase

C The antiinflammatory
actions of cortico steroids
involve phospholipase A 2
inhibitory proteins known as
lipocortins.

231
Pharmacy

1404.
750.

1405.
751.

1406.
752.

1407.
753.

1408.
754.

1409.
755.

1410.
756.

1411.
757.

1412.
758.

Which of the following synthetic fluorinated corticosteroid is


administered with an oral metered dose inhaler for the treatment of
steroid dependent bronchial asthma in a 25 year old woman?
A. Cromolyn
B. Flunticasone
C. Ipratropium
D. Montelukast
May induce extrapyramidal side effects:
A. Metoclopramide
B. Diphenoxylate
C. Sucralfate
D. Carbenoxolone
Which of the following may be indicative of decreased potassium
blood levels?
A. Diuretic use
B. Chronic laxative use
C. Both A & B
D. Neither A nor B
Sucralfate :
A. Acts by binding to necrotic ulcer tissue
B. Activates pepsin in stomach
C. Should be used in conjunction with H2 blockers
D. Is not active in the stomach
Mucosal protective agents include:
A. Sucralfate
B. Pirenzepine
C. Neither A nor B
D. Both A & B
Olanzapine (Zyprexa*) is:
A a growth regulator
B a benzodiazepine
C a neuroleptic agent
D an agonist of 5HT2 serotonin receptors
Finasteride is
A a glucocorticoid
B a 5 alpha reductase inhibitor
C A phosphodiesterase inhibitor
D a specific inhibitor of gonadorelin GnRH secretion
Physostigmine and neostigmine act by:
A inhibiting muscarinic receptors
B inhibiting cholinesterases
C inhibiting monoamine oxidase
D stimulating 5HT3 serotonin receptors
The following drugs are nitric oxide donors, NO donors:
A sodium nitroprusside
B molsidomine
C glyceryl trinitrate
D All of the above

232
Pharmacy

1413.
759.

Clozapine (Leponex*) is:


A lipid lowering 3rd generation drug
B a benzodiazepine
C a neuroleptic agent
D an agonist of the 5HT2 serotonin receptors

1414.
760.

Adverse effects of neuroleptic agents, such as galactorrhea,


gynecomastia, ejaculation disorders, are attributed to:
A an antimuscarinic effect
B an inhibition of angiotensin receptors
C an inhibition of dopamine receptors
D a decrease of prolactin secretion

1415.
761.

The following drug (s) decrease(s) or suppress(es) delusions and


hallucinations:
A imipramine
B phenobarbital
C a phenothiazine such as chlorpromazine
D a benzamide such as amisulpride
The following drugs act mainly by inhibiting the reuptake
of transmitter, serotonin and/or noradrenaline:
A oxazepam
B levomepromazine
C citalopram
D Phenobarbitone
The drug(s) is (are) often prescribed for the treatment of intense
pain:
A. morphine
B. lithium
C. diazepam
D. buprenorphine
The following drugs are used commonly as anxiolytics:
A clorazepate
B flumazenil
C buprenorphine
D oxazepam
The following drugs act mainly by inhibiting the reuptake of
serotonin:
A fluoxetine
B sertraline
C paroxetine
D All of the above

1416.
762.

1417.
763.

1418.
764.

1419.
765.

1420.
766.

Which drug issed


issuedasasspecific
specifictreatment
treatmentfor
forMania?
Mania?
A phenobarbital
B methylphenidate (ritalin)
C Lithium
D Phenytoin

A&D

A&D

233
Pharmacy

1421.
767.

The following drug (s) induce(s) adversely an increase of prolactin


secretion (with galactorrhea, amenorrhea):
A fluphenazine
B haloperidol
C Lorazepam
D Oxazepam

A&B

1422.
768.

The main mechanism(s) proposed to explain the antidepressant


action of the antidepressants is (are):
A an inhibition of of dopamine receptors
B inhibition of the reuptake of serotonin or norepinephrine
C inhibition of adenosine reuptake
D inhibition of serotonin and noradrenaline catabolism
The term reuptake related to a transmitter, for example reuptake of
serotonin, means that the transmitter
A is quickly inactivated by the P450 cytochromes
B is bound to plasma proteins
C is inactivated by monoamine oxidase
D is taken up by the presynaptic element which released it
The drugs usually used for the treatment of obsessive compulsive
disorders are
A hypnotic benzodiazepines
B antimuscarinic agents
C antidepressants, inhibitors of serotonin reuptake
D antiepileptic drugs
Imipramine:
A is an antiepileptic
B is a neuroleptic agent
C has atropinic effect
D is a tricyclic antidepressant
Carbimazole or NEOMERCAZOLE is:
A an antifungal
B a cephalosporin of second generation
C a synthetic antithyroid drug
D antiepileptic
Androstanolone, also called dihydrotestosterone
A is a steroid androgen
B is an antagonist of testosterone
C is an active metabolite of testosterone
D is a glucocorticoid

B,D

Acarbose
A Is administered by injection
B stimulates insulin secretion
C inhibits intestinal absorption of carbohydrates
D is an inhibitor of alpha glucosidases

C&D

769.
1423.

1424.
770.

771.
1425.

772.
1426.

773.
1427.

1428.
774.

A&C

234
Pharmacy

1429.
775.

776.
1430.

777.
1431.

778.
1432.

779.
1433.

780.
1434.

781.
1435.

782.
1436.

783.
1437.

Cabergoline is:
A is chemically an ergot derivative
B is used for the treatment of hyperprolactinemia
C Both A & B
D Neither A nor B
Apomorphine is:
A a morphinomimetic drug
B an antagonist of morphine
C a direct dopaminomimetic
D used in the treatment of the Parkinson's disease
Fexofenadine is:
A a platelet aggregation inhibitor
B an antivitamin K
C a H1 antihistamine
D an anticholinesterase
Clozapine
A is a benzodiazepine
B is atypical neuroleptic
C frequently causes agranulocytosis as adverse effect,
D is a 5HT2 agonist
Piroxicam is:
A an ACE (angiotensin converting enzyme) inhibitor
B an inhibitor of cyclooxygenases
C an antihypertensive drug
D an antidiabetic by stimulation of insulin secretion
Alfuzosine is:
A a dopaminergic agonist
B a betablocker
C an alpha1adrenoreceptor blocker
D used for the treatment of functionnal symptoms of prostatic
adenoma
The following ophthalmic solutions can cause mydriasis:
A atropine
B tropicamide
C pilocarpine
D Aceclidine
In the biosynthesis of noradrenaline one finds the following
precursors:
A tyrosine
B histidine
C Serotonin
D dopamine
Monoamine oxidases are enzymes which:
A catalyze the oxidative deamination of monoamines
B inactivate norepinephrine
C activate serotonin
D inactivate GABA

C&D

B&C

C&D

A&B

A&D

A&B

235
Pharmacy

784.
1438.

The biosynthesis of serotonin is carried out from:


A histidine
B tryptophan
C arachidonic acid
D phenylalanine
The biosynthesis of nitric oxide, NO, is carried out from:
A histidine
B tryptophan
C arginine
D arachidonic acid

1440.
786.

An elderly patient develops a bradycardia since he uses an


ophthalmic solution for glaucoma treatment. This ophthalmic
solution could contain:
A an angiotensin II antagonist
B a beta adrenergic blocker
C scopolamine
D an antibiotic like rifamycine

787.
1441.

A patient a few hours after having taken a neuroleptic agent such


as haloperidol presents an acute facial dyskinesia . The following
drugs could correct this disorder:

785.
1439.

A
B
C
D
788.
1442.

trinitrine
naltrexone
dexamethasone
trihexylphenidyl

The inhibitors of 5HT3 serotonin receptors (setrons) are usually


used as:
A
B
C
D

Antidepressants
CNS stimulants
antiemetic agents
anxiolytic agents

1443.
789.

Sulfonylureas:
A are active in absence of the pancreas
B increase insulin release
C are given by subcutaneous route
D are used for insulind ependant diabetes, type1

1444.
790.

A patient after the intake of a drug presents coldness of the


extremities (hands, feet). The drug responsible could be:
A an alphadrenergic blocking drug such as prazosin
B Ergotamine
C Nifedipine
D an ACE inhibitor such as captopril

236
Pharmacy

791.
1445.

A person regularly takes high doses of morphine. A compound


is injected to him and the fast advent of symptoms such as anxiety,
agitation, diarrhea, mydriasis, is observed. This compound can be:
A morphine itself
B a morphine antagonist
C methadone
D a benzodiazepine such as diazepam

1446.
792.

During a treatment, an patient develops a drug induced dry cough.


The drug(s) possibly responsible for this cough is (are):
A Indomethacin
B captopril
C Prindopril
D Aspirin

B,C

1447.
793.

When a poisoning with acetaminophen is observed, it should be


treated by:
A Aspirin
B Ibuprofen
C Indomethacin
D N-acetyl cysteine

794.
1448.

Fluoxetine (Prozac*):
A is a specific inhibitor of noradrenaline reuptake
B is an atypical neuroleptic agent
C is an antidepressant
D is administered by sublingual route

795.
1449.

Atropine:
A can be used as a Preanesthetic medication
B induces miosis
C induces a contracture of smooth muscle fibers
D increases salivary secretion

1450.
796.

Clomifene:
A inhibits the effects of estradiol at the hypothalamic level
B increase the release of FSH and LH
C is used for the treatment of female infertility
D All of the above

237
Pharmacy

1451.
797.

The inhibitors of 5 alpha reductase, such as finasteride, are used for D


the treatment of:
A Cushing disease
B male sterility
C hyperprolactinemia
D benign prostatic hypertrophy

1452.
798.

Desmopressin (Minirin) is a drug used for the treatment of:


A renal impairment
B non insulin dependant diabetes mellitus
C diabetes insipidus
D insulin dependant diabetes mellitus

1453.
799.

Naltrexone (Revia):
A is an opioid analgesic drug
B is a benzodiazepine
C is a phenothiazine
D is a specific opioid antagonist

1454.
800.

Hyperprolactinemia is an adverse effect frequently


Hyperprolactinemia

observed during treatment with:


A ACE inhibiters
B anxiolytic benzodiazepines
C hypnotic benzodiazepines
D neuroleptic agents
1455.
801.

Acute dyskinesia occurring a short time after the onset of a


treatment with a neuroleptic agent is treated by:
A increase of the dosage of the prescribed neuroleptic drug
B an antimuscarinic such as trihexyphenidyl
C a neuroleptic agent of a different chemical group
D an anxiolytic benzodiazepine like diazepam

1456.
802.

Which statement best describes bioavailability?


A. Relation between physical and chemical properties of a drug
and its systemic absorption.
B. Measurement of the rate and amount of therapeutically
active drug that reaches the systemic circulation.
C. Movement of drug into body tissues over time
D. Dissolution of the drug in GI tract.

803.
1457.

The route of administration that gives the most rapid onset of


pharmacologic effect is:
A. I/M injection
B. I/V injection
C. I/D injection
D. S/C injection

238
Pharmacy

1458.
804.

The route of administration that provides complete bioavailability?


A. I/M injection
B. I/V injection
C. I/D injection
D. S/C injection

1459.
805.

After per oral administration, drugs generally are absorbed best


from the:
A. Buccal cavity
B. Stomach
C. Duodenum
D. Ileum
The characteristics of active transport process includes all of the
following EXCEPT :
A. Active transport moves drug molecules against a
concentration gradient
B. Active transport follows Ficks law of diffusion
C. Active transport is carrier mediated transport system
D. Active transport requires energy
The passage of drug molecules from a region of high
concentration to a region of low drug concentration is known as:
A. Active transport
B. Bioavailability
C. Biopharmaceutics
D. Simple diffusion
What equation describes the rate of drug dissolution from a tablet?
A. Ficks Law
B. Law of mass action
C. Michaelis Menten equation
D. Noyes Whitney equation
Which condition usually increases the rate of drug dissolution
from a tablet?
A. Increase in particle size of the drug
B. Decrease in surface area of the drug
C. Use of free acid from the drug
D. Use of ionized or salt form of the drug.
The rate limiting step in bioavailability of a lipid soluble drug
formulated as an immediate release compressed tablet is the rate
of:
A. Biotransformation or metabolism of the drug
B. Dissolution of the drug
C. Transport of drug molecules across intestinal mucosal cells.
D. Blood flow to the GI tract

C&D

The extent of ionization of a weak electrolyte drug is dependent on


the:
A. pH of the media and pKa of the drug
B. oil water partition coefficient of the drug
C. particle size of the drug
D. polymorphic form of the drug

806.
1460.

1461.
807.

808.
1462.

809.
1463.

810.
1464.

1465.
811.

239
Pharmacy

812.
1466.

813.
1467.

814.
1468.

815.
1469.

816.
1470.

817.
1471.

818.
1472.

819.
1473.

The rate of bioavailability is most rapid when the drug is


C
formulated as a
A. Controlled release product
B. Suspension
C. Solution
D. Hard gelatin capsule
A 40 year old man complains of dysuria and urinary urgency and
A
is diagnosed to have gonococcal urethritis. He is given Procaine
Penicillin and in addition probenecid. This type of combined
effect is known as:
A. Potentiation
B. Competitive Antagonism
C. Non competitiveAntagonism
D. Addition
Which of the following statements concerning a drug receptor is true? C
A. It mediates non specific action of anesthetics
B. Its expression is induced only by exogenously added drugs
C. It can bind endogenous ligand to produce physiological
activity
D. It mediates catharitic activity of magnesium citrate.
Which of the following drugs would most likely be used in the
B
treatment of bronchospasm that is associated with COPD:
A. Edrophonium
B. Ipratropium
C. Ambenonium
D. Propantheline
All of the following are ADR manifested with cholinergic
C
agonists, except :
A. Bradycardia
B. Bronchoconstriction
C. Xerostomia
D. Lacrimation
Which of the following drugs is considered to be the agent of
C
choice for anaphylactic reactions?
A. Clonidine
B. Isoprenaline
C. Adrenaline
D. Terbutaline
Which of the following neuromuscular blocking agents cause
B
muscarinic responses such as bradycardia and increased glandular
secretions?
A. Tubocurarine
B. Succinylcholine
C. Pancuronium
D. G
Gallamine
Which of the following agents would not be appropriate in the
A
treatment of glaucoma?
A. Atropine
B. Pilocarpine
C. Physostigmine
D. Timolol

240
Pharmacy

1474.
820.

Adverse reactions to atropine include all of the following except:


A. Photophobia
B. Drymouth
C. Sedation
D. Diarrhea

821.
1475.

Which of the following is a volatile substance that is administered


by inhalation?
A. Thiopental
B. Halothane
C. Buspirone
D. Fentanyl
Which is the shortest acting Barbiturate?
A. Thiopental
B. Phenobarbitone
C. Pentobarbitone
D. Primidone
The brief duration of action of an ultra short acting barbiturate is
due to a
A. Slow rate of metabolism in the liver
B. Low lipid solubility
C. Rapid rate of redistribution from the brain due to drugs high
lipid solubility
D. High degree of binding to plasma proteins.
Which of the following mechanisms of action most likely
contributes to the treatment of Parkinsonism?
A. Amantadine which stimulate release of striatal dopamine
B. Striatal H1 receptors are blocked by the antihistaminic
trihexyphenidyl
C. Bromocryptine stimulates release of striatal dopamine
D. The ability of dopamine to cross blood brain barrier allows
to restore neurotransmitter balance.

825.
1479.

All of the following are ADR of Levodopa except:


A. Sialorrhea
B. Orthostatic hypotension
C. Delusion
D. Livedo reticularis

1480.
826.

The activity of which of the agents is dependent on GABAergic


mechanism?
A. imipramine
B. Meperidine
C. Diazepam
D. Chlorpromazine

1481.
827.

Opioids are used as all of the following agents except:


A. Antitussives
B. Analgesics
C. Antiinflammatory
D. Antidiarrhoeal

822.
1476.

823.
1477.

1478.
824.

241
Pharmacy

828.
1482.

Which of the following agents would not be an alternative to


Phenobarbital in the treatment of partial seizures?
A. Trimethadione
B. Gabapentin
C. Felbamate
D. Lamotrigene

829.
1483.

All of the following are therapeutic applications of salicylates


except:
A. Rheumatoid arthritis
B. Fever in children with influenza
C. Spasmodic dysmenorrhoea
D. Prophylaxis against Myocardial infarction
All of the following statements describing Paracetamol are true
except:
A. it has a greater antiinflammatory activity than salicylates
B. it acts as analgesic and antipyretic
C. it may cause skin rash
D. acute overdosage is characterized by severe hepatotoxicity.
All of the following are therapeutic applications of Prostaglandins
except:
A. Hypertension
B. Miscarriage with retained fetus
C. Premature labor
D. Delayed labor

832.
1486.

Calcium channel blockers have all of the following characteristics


except:
A. They block the slow inward current carried by calcium
during Phase 2 of the cardiac action potential.
B. They dilate peripheral arterioles
C. They constrict coronary arteries and arterioles
D. They are useful in treating stable angina

833.
1487.

The termination of activity of he parin by Protamine sulfate is due to: D


A. A chelating action
B. The inhibition of GI absorption of heparin
C. The displacement of heparin plasma protein binding
D. Acid base interaction
Which of the following cardiovascular agents is classified
B
chemically as a glycoside?
A. Nifedipine
B. Digoxin
C. Flecainide
D. Cholestyramine
Cardiac glycosides may be useful in treating all of the following
D
except:
A. Atrial flutter
B. Atrial tachycardia
C. Congestive heart failure
D. Ventricular tachycardia

830.
1484.

831.
1485.

834.
1488.

835.
1489.

242
Pharmacy

1490.
836.

1491.
837.

1492.
838.

1493.
839.

1494.
840.

1495.
841.

1496.
842.

Ingestion of the following vitamin should be avoided by a patient


taking oral anti coagulant.
A. Vitamin A
B. Vitamin B
C. Vitamin C
D. Vitamin K
All of the following are true of lithium except:
A. Lithium is useful in the treatment of all types of affectivedisorders.
B. Should not be used in pregnant women
C. Side effects include thyroid enlargement and polyuria
D. Tremor is a sign of intoxication
Opioid with the lowest addictive potential:
A. codeine
B. fentanyl
C. pentazocine
D. loperamide
Shortest acting opioid drug:
A. fentanyl
B. sufentanil
C. alfentanil
D. morphine
High efficacy opioid analgesic:
A. codeine
B. dextromethorphan
C. propoxyphene
D. fentanyl
Opioid antagonist used in the treatment of acute overdose:
A. naloxone
B. naltrexone
C. nalbuphine
D. methadone
Which of the following are used to improve cognitive function in
patients with dementia?
A. donepezil
B. tacrine
C. methylphenidate
D. A & B

1497.
843.

Which of the following is a nonstimulant drug used to improve


attention in ADHD?
A. damphetamine
B. methylphenidate
C. nicotine
D. atomoxetine

1498.
844.

Which of the following are approved uses of damphetamine?


A. ADHD
B. narcolepsy
C. weight loss
D. A & B

243
Pharmacy

1499.
845.

The depth of the anesthesia is primarily dependent upon:


A. concentration of the anesthetics in the lung
B. concentration of the anesthetics in the gas mixture inhaled
C. concentration of the anesthetics in the CNS
D. the number of drugs in the combination

1500.
846.

Which anesthetic is the quickest in reaching saturation in the blood? B


A. methoxiflurane
B. nitrous oxide
C. halothane
D. enflurane
Select the correct statement for the inhaled anesthetics:
B
A. the more time it takes to develop, the less time it takes to
recover
B. the less time it takes to develop, the less time it takes to
recover
C. the less time it takes to develop, the more time it takes to
recover
The reason why thiopental has a short action (minutes) is because: A
A. it has a unique distribution pattern
B. quickly metabolized by the liver
C. quickly metabolized by enzymes in the lung and blood
D. tolerance develops in minutes

847.
1501.

848.
1502.

849.
1503.

Atracurium acts by:


A. binding to acetylcholine receptors at the neuromuscular
junction and producing excess stimulation
B. binding to acetylcholine receptors at the neuromuscular
junction and preventing access of the transmitter
C. binding to acetylcholine receptors at the neuromuscular
junction and preventing metabolism of the transmitter
D. decreasing the release of intracellular calcium in the
muscle cells

1504.
850.

The mechanism of action of local anesthetics involves:


A. blocking voltage gated Na+ channels
B. blocking voltage gated Ca+ channels
C. A only
D. Both A & B

851.
1505.

Adverse effects of local anesthetics:


A. decreased myocardial conduction
B. seizures
C. CNS depression
D. Both A & B

1506.
852.

Local anesthetics liable to cause an allergic reaction:


A. cocaine
B. benzocaine
C. procaine
D. all of the above

244
Pharmacy

853.
1507.

1508.
854.

1509.
855.

1510.
856.

1511.
857.

1512.
858.

1513.
859.

The class of drugs used as first-line agents in panic disorders is:


A. benzodiazepines
B. MAO inhibitors
C. tricyclic antidepressants
D. selective serotonin re-uptake inhibitors
A 33 year old man in a state of status epilepticus was brought to
the Emergency Department by his wife. The most appropriate drug
to stop his seizures is?
A. alprazolam
B. diazepam
C. ethosuximide
D. valproic acid
A 58 year old woman with a history of uncontrolled complex partial
seizures is referred to the Epilepsy Clinic for further evaluation
and treatment since phenytoin therapy no longer provides adequate
control. The neurologist selects to add a drug to the therapeutic
regiment that lacks pharmacokinetic interactions with phenytoin
and is thought to potentiate GABAergic activity in the brain. The
drug most likely selected is:
A. carbamazepine
B. felbamate
C. gabapentin
D. lamotrigine
Which of the following drugs would you recommend for
monotherapy of major depression in a 37 year old man who has
prominent mood congruent psychotic features:
A. amoxapine
B. bupropion
C. carbamazepine
D. maprotiline
A 72 year old man with Parkinsons disease has been managed with
levodopa/carbidopa for 3 years. An additional drug is now added
to the therapeutic regiment to further improve control of
symptoms. The drug added prevents the oxidative deamination of
dopamine. This drug is:
A. entacapone
B. procyclidine
C. ropinerole
D. selegiline
A 25 year old man has been recently diagnosed with schizophrenia.
A firstline drug for the longterm management of schizophrenia
because of its low incidence of extrapyramidal, sedative,
hypotensive and anticholinergic effects is:
A. clozapine
B. fluphenazine
C. haloperidol
D. risperidone
All of the following substances are endogenous tropic hormones
secreted by the pituitary gland EXCEPT:
A. Somatotropin

245
Pharmacy

860.
1514.

861.
1515.

862.
1516.

863.
1517.

1518.
864.

1519.
865.

866.
1520.

867.
1521.

B. Human chorionic gonadotropin


C. Follicle stimulating hormone
D. Thyroid stimulating hormone
Which of the following substances when present in urine is the
most likely positive sign of pregnancy?
A. Somatotropin
B. Human chorionic gonadotropin
C. Follicle stimulating hormone
D. Thyroid stimulating hormone
Which of the following glucocorticoids produces the least water
retention?
A. Cortisone
B. Hydrocortisone
C. Prednisolone
D. Dexamethasone
Which of the following insulins can be administred intravenously?
A. regular insulin
B. Isophane Insulin
C. Protamine zinc insulin
D. Semilente insulin
In comparing levothyroxine to liothyronine which of the following
statements is not correct?
A. Both are naturally occurring thyroid hormones
B. Liothyronine can be converted in the peripheral circulation to
levothyroxine
C. Liothyronine is more potent
D. Liothyronine has a shorter duration of action
Which of the following class of compounds stimulates the release
of insulin from Pancreatic Beta cells?
A. Sulphonylureas
B. Biguanides
C. Alpha glucosidase inhibitors
D. Progestins
Exertion induced angina, which is relieved by rest, nitroglycerin or
both, is referred as:
A. Prinzmetals angina
B. Unstable angina
C. Classic angina
D. Variant angina
Myocardial oxygen demand is increased by all of the following
factors EXCEPT:
A. Exercise
B. Smoking
C. Propranolol
D. Isoprenaline
Which of the following agents used in Prinzmetals angina has
spasmolytic actions which increase coronary blood supply?
A. Nitroglycerin
B. Nifedipine
C. Timolol
D. Propranolol

246
Pharmacy

1522.
868.

Patients with angina receiving Propranolol plus Diltiazem must be


monitored for which ADR?
A. Decreased Cardiac out put
B. Decreased heart rate
C. Increased heart rate
D. A & B

869.
1523.

The term ischemic heart disease is used to designate all of the


following conditions except:
A. Prinzmetals angina
B. Sudden cardiac death
C. Congestive Heart failure
D. Myocardial infarction
The development of ischemic pain occurs when the demands for
oxygen exceed the supply. Determinants of oxygen demand
include all of the following except:
A. Myocardial ejection time
B. Contractile state of heart
C. Systolic pressure
D. Right atrial pressure
Myositis is an ADR of all of the following except:
A. Atorvastatin
B. Pravastatin
C. Gemfibrosil
D. Simvastatin
Strong anticholinergic effects limit the antiarrhythmic use
of:
A. Quinidine
B. Procainamide
C. Tocainide
D. Disopyramide
A pronounced slowing of phase 0 of the myocardial action
potential would be reflected on the electrocardiogram as a :
A. shortened QRS complex
B. Shortened P wave
C. Prolonged QRS complex
D. Flipped T wave
Which of the following class I antiarrhythmics would be
most capable of inducing the torsades de points type of
ventricular tachycardia?
A. Quinidine
B. Procainamide
C. Tocainide
D. Disopyramide
A patient receiving a class I antiarrhythmic on a chronic
basis complains of fatigue, low grade fever, and joint pain
suggestive of systemic lupus erythmatosus (SLE). The
patient is most likely receiving:
A. Quinidine
B. Procainamide

870.
1524.

871.
1525.

872.
1526.

1527.
873.

874.
1528.

875.
1529.

247
Pharmacy

1530.
876.

877.
1531.

878.
1532.

879.
1533.

1534.
880.

1535.
881.

1536.
882.

883.
1537.

C. Tocainide
D. Disopyramide
Class IA antiarrhythmic do all of the following to the cardiac
cells action potential except:
A. slow the rate of rise of Phase 0
B. Delay the fast channel conductance of sodium ions
C. Prolong phases 2 and 3 of repolarization.
D. Inhibit slow channel conductance of calcium ions.
Which of the following drugs is a class IV antiarrhythmic
agent that is primarily indicated for the treatment of
supraventricular tachyarrhythmias?
A. Quinidine
B. Procainamide
C. Diltiazem
D. Disopyramide
Sinus tachycardia is characterized by a heart rate:
A. in excess of 100 with impulses initiated by the AV node
B. in excess of 60 with impulses initiated by the SA node
C. in excess of 100 with impulses initiated by the SA node
D. Less than 60 impulses initiated by the SA node
Which of the following agents has a direct effect on the AV
node, delaying the calcium channel depolarization?
A. Quinidine
B. Procainamide
C. Verapamil
D. Disopyramide
Which of the following drugs is a class III antiarrhythmic
agent that is effective in the acute management of ventricular
tachycardia, including ventricular tachycardia, including
ventricular fibrillation?
A. Bretylium
B. Procainamide
C. Verapamil
D. Disopyramide
All of the following problems represent concerns when
patients are started on Amiodarone except:
A. Extremely long elimination half life
B. Need for multiple daily doses
C. development of hyperthyroidism
D. Development of pulmonary fibrosis
Which of the following agents represent angiotensin receptor
antagonist?
A. Bretylium
B. Propranolol
C. Atenelol
D. Irbesartan
Reflex tachycardia, headache, and postural hypotension are
ADR that limit the use of which of the following
antihypertensive agents?
A. Hydralazine

248
Pharmacy

884.
1538.

885.
1539.

886.
1540.

887.
1541.

1542.
888.

B. Methyl dopa
C. Captopril
D. Irbesartan
A 60 year old man with moderate hypertension, that is
refractory to diuretics, beta blockers and methyl dopa. He
has renovascular hypertension with elevated rennin levels,
confirmed by lab finding. This patients antihypertensive
regimen would be enhanced best by adding which of these
following agents?
A. Hydralazine
B. Methyl dopa
C. Captopril
D. Clonidine
A hypertensive patient who is asthmatic and very non
compliant would be best treated with which of the following
beta blocking agents?
A. Acebutalol
B. Esmolol
C. Captopril
D. Clonidine
Long standing hypertension leads to tissue damage in all of
the following organs EXCEPT:
A. Heart
B. Skin
C. Kidneys
D. Brain
All of the following agents are suitable for initial therapy for
treating hypertension EXCEPT:
A. Guanethidine
B. Hydrochlorthiazide
C. Atenolol
D. Nifedipine
Which of the following terms describe a redused number
of platelets?

A
Acebutalol is
cardiospecific and has long
duration of action requiring
fewer dosages per day for
non compliant individuals.

A. Thrombocytopenia
B. Leukopenia
C. Leukocytosis
D. Thrombophlebitis
1543.
889.

Which of the following combinations of drugs, when used


together, reduce both preload and after load?
A. Hydralazine and isosorbide dinitrate
B. Methyl dopa and Captopril
C. Prazosin and Irbesartan
D. Methyl dopa and Hydralazine

890.
1544.

When digitalis glycosides are used in patients with


congestive heart failure, they work by exerting a positive
effect on :
A. Stroke volume

249
Pharmacy

B. Total peripheral resistance


C. Heart rate
D. Venous return
891.
Because of the direct dilating effects on the lung, certain
1545.
agents aid in the treatment of congestive heart failure in
patients suffering from pulmonary congestion. Which of the
following agents is in this category?
A. Hydrochlorthiazide
B. Triamterene
C. Furosemide
D. Metolazone
1546.
892.
Which of the following vasodilators is an orally effective
preload and after load reducer in the patient with congestive
heart failure?
A. Hydralazine
B. Prazosin
C. Isosorbide dinitrate
D. Nitroprusside
1547.
893.
The use ACE inhibitors in congestive heart failure, centers
around their ability to cause:
A. Direct reduction in renin levels
B. Indirect reduction in angiotensin II and aldosterone levels
C. Direct reduction in aldosterone secretion
D. Inhibition of angiotensin II receptor
894.
Isoniazid is a primary antitubercular agent that
1548.
A. That requires Pyridoxine supplementation
B. May discolor the tears, saliva, urine
C. Causes ocular complications
D. May be ototoxic and nephrotoxic
895.
All of the following factors may increase the risk of
1549.
nephrotoxicity from gentamicin therapy except:
A. Age over 70years
B. Prolonged courses of gentamicin therapy.
C. Concurrent amphotericin therapy.
D. trough gentamicin levels below 2g/ml
896.
In which of the following groups do all four drugs warrant
1550.
careful monitoring for drug related seizures in high risk
patients?
A. Penicillin G, Imipenem, Amphotericin B, Metronidazole
B. Penicillin G , Chloramphenicol, Tetracycline,
Vancomycin
C. Imipenem , Tetracycline, Vancomycin, sulfadiazine
D. Vancomycin, sulfadiazine , Metronidazole, Cycloserine
1551.
897.
Spectinomycin is an aminoglycoside like antibiotic indicated
for the treatment of:
A. gram negative bacillary septicemia
B. Tuberculosis
C. Penicillin resistant gonococcal infections
D. Syphilis

250
Pharmacy

1552.
898.

899.
1553.

900.
1554.

901.
1555.

1556.
902.

903.
1557.

904.
1558.

1559.
905.

A woman has an Upper respiratory tract infection, 6 years ago she


experienced an episode of bronchospasm following Penicillin
V therapy. The cultu res now reveal a strain of Streptococcus
pneumoniae that is sensitive to all of the following drugs.
Which of the drugs would be best choice for this patient?
A. Amoxycillin Clavulanate
B. Erythromycin
C. Ciprofloxacin
D. Carbenicillin
All of the following drugs are suitable for oral therapy for a
lower urinary tract infection due to P aeruginosa EXCEPT:
A. Norfloxacin
B. Trimethoprim / sulfamethoxazole
C. Ciprofloxacin
D. Carbenicillin
Which of the following regimens would be appropriate for
oral therapy for a mild staphylococcal cellulites?
A. Dicloxacillin 125 mg q6h
B. Vancomycin 250mg q6h
C. Methicillin 500mg q6h
D. Cefazolin 1g q8h
Which of the following drugs has demonstrated in vitro
activity against M avium intracellulare?
A. Norfloxacin
B. Clarithromycin
C. Ciprofloxacin
D. Carbenicillin
Phenytoin is used for the treatment of all of the following
type of seizures EXCEPT :
A. Grandmal
B. Simple partial
C. Complex partial
D. Absence Seizure
Which of the following anticonvulsants is contraindicated in
patients with a history of hypersensitivity to tricyclic
antidepressants?
A. Carbamazepine
B. Ethosuximide
C. Acetazolamide
D. Phenytoin
Which anticonvulsant drug requires therapeutic monitoring
of phenobarbitone serum levels as well as its own levels?
A. Carbamazepine
B. Ethosuximide
C. Primidone
D. Phenytoin
A 23 year old patient is diagnosed with simple partial
seizures. What would be the drug of choice in this patient?
A. Carbamazepine
B. Ethosuximide

251
Pharmacy

906.
1560.

907.
1561.

908.
1562.

909.
1563.

910.
1564.

911.
1565.

912.
1566.

913.
1567.

C. Primidone
D. Phenytoin
The maximum recommended dose of Levodopa is :
A. 80mg
B. 5mg
C. 5g
D. 8g
When administered with Carbidopa, the dosage of Levodopa,
is usually decreased by:
A. 50%
B. 75%
C. 40%
D. 10%
Which of the following agents should not be used
concurrently with levodopa?
A. Diphenhydramine
B. Benztropine
C. Amantadine
D. MAO inhibitors.
Most antipsychotic agents except clozapine are potent
blockers of which neurotransmitters?
A. Dopamine
B. Norepinephrine
C. Acetyl choline
D. Epinephrine
The earliest movement disorder to appear during neuroleptic
therapy is:
A. Pseudoparkinsonism
B. Dystonia
C. Tardive dyskinesia
D. Akathisia
Specific therapy for an acute extrapyramidal reaction to
chlorpromazine is:
A. Physostigmine
B. Diphenhydramine
C. Acetyl choline
D. Epinephrine
All of the following patterns are associated with bipolar
major affective disorder, EXCEPT:
A. a history of manic episodes only
B. a history of depressed episodes only
C. a history of several depressed episodes and only one
manic episode
D. a history of several manic episodes and only one
depressed episode
Unipolar affective disorder is characterized by which of the
following signs?
A. Flight of ideas
B. Unusual verbosity
C. Poor judgment leading to reckless driving
D. Loss of interest in job and family

252
Pharmacy

1568.
914.

Symptoms of intoxication with inorganic mercury salts


A. colic, constipation, increased excretion of aminolevulinic
acid, stippled erythrocytes.
B. emphysema, proteinuria, disturbances of calcium
metabolism.
C. tremor, colic, proteinuria.
D. colic, gastrointestinal bleeding, and stippled erythrocytes.

1569.
915.

The most reliable method of identifying an agent responsible


for poisoning is
A. Locating a toxic agent near the victim
B. observation of the symptoms
C. pathologic changes observed grossly or microscopically
D. chemical analysis of the blood or tissues

1570.
916.

The drug of choice used to treat lead poisoning by forming a


lead chelate which is excreted by the kidney
A. dimercaprol (BAL)
B. penicillamine
C. calcium disodium edetate (CaNa 2EDTA)
D. trisodium edetate (Na3HEDTA)

917.
1571.

Activated charcoal can reasonably be expected to


A. adsorb certain alkaloids and metals
B. increase the excretion of some poisons into the urine
C. coat the gastric mucosa and thereby decrease the surface
area available for absorption
D. all of the above

1572.
918.

This chelating agent can reactivate sulfhydryl groups that


have been deactivated by some heavy metals
A. CaNa2EDTA
B. penicillamine
C. dimercaprol
D. All of the above

1573.
919.

Overdose with aspirin is associated with


A. metabolic acidosis
B. respiratory alkalosis
C. coma
D. all of the above

1574.
920.

Is capable of protecting sulfhydryl enzyme inactivation by


heavy metals and also reactivates sulfhydryl enzyme systems
already inhibited
A. penicillamine
B. dimercaprol
C. mercury
D. lead

253
Pharmacy

921.
1575.

Sign and symptoms of organophosphate insecticide


poisoning usually do not occur in less than 18 hours after
exposure
A. True
B. False
Chronic poisoning is characterized by paroxysmal pain
(colic)
A. mercury
B. lead
C. both
D. neither

923.
1577.

Atropine is useful in treating poisoning produced by


organophosphate insecticides because it
A. reactivates inhibited acetylcholinesterase
B. stimulate alpha receptors directly
C. stimulate beta receptors directly
D. blocks the action of acetylcholine at both central and
peripheral sites

924.
1578.

Produces hypocalcemia unless given in the calcium chelate


form
A dimercaprol
B. edetate
C. penicillamine
D. deferoxamine
Antidotal therapy includes administration of atropine and
pralidoxime (2PAM)
A. lead poisoning
B. mercury poisoning
C. organophosphate poisoning
D. carbon monoxide poisoning
Dimercaprol (BAL) protects against the toxic action of
certain metals because it
A. combines with the blood forming elements
B. combines with sulfhydryl groups in proteins
C. forms a stable complex with the metal
D. increases the excretion of the metal by a direct action on
the kideny

927.
1581.

Parathion poisoning is best treated by


A. dimercaprol
B. nitrite
C. atropine
D. disulfiram

928.
1582.

In the treatment of acute lead poisoning, the most


satisfactory chelating agent is
A. dimercaprol
B. penicillamine
C. EDTA
D. calcium disodium edetate

922.
1576.

1579.
925.

926.
1580.

254
Pharmacy

1583.
931.

Symptoms of organophosphate poisoning include


A. pinpoint pupils
B. sympathomimetic effects
C. salivation
D. psychotic delusions

A&C

1584.
932.

Excretion of toxic substances increased by administration of


diuretics
A. chronic mercury poisoning
B. chronic lead poisoning
C. both
D. neither

1585.
933.

Salivation, lacrimation, urination, defecation, muscle


twitches, and respiratory depression represent a particular
syndrome which
A. may be caused by an overdose of neostigmine
B. may result from organophosphate insecticide poisoning
C. occurs with poisoning by anticholinesterase agents
D. all of the above

1586.
934.

Useful in treatment of mercury poisoning


A. caffeine
B. calcium disodium edetate
C. universal antidote
D. dimercaprol (BAL)

1587.
935.

Improve skeletal muscle function in anticholinesterase


intoxication
A. atropine
B. pralidoxime
C. both
D. neither

1588.
936.

Effective treatment of lead poisoning


A. calcium disodium edetate
B. penicillamine
C. both
D. neither

255
Pharmacy

937.
1589.

A child who has ingested an insecticide containing malathion


will have each of the following symptoms EXCEPT:
A. muscular fasciculations
B. abdominal cramps
C. spasm of accommodation
D. dry skin

1590.
938.

BAL (dimercaprol) is the treatment of choice in acute arsenic


and acute lead poisoning
A. True
B. False

1591.
939.

Organophosphorus cholinesterase inhibitors are effective as


chemical warfare agents because organophosphorus
cholinesterase inhibitors are highly soluble and can be
absorbed readily through the skin and lungs
A. True
B. False

1592.
940.

Mercury poisoning
A. may be influenced by the chemical forms of mercury
encountered
B. is effectively antidoted with calcium disodium EDTA
C. is most severe when the central nervous system is
involved.
D. is not a real problem in the present day

A&C

941.
1593.

Specific treatment for organophosphate insecticide poisoning


should not be started until blood cholinesterase activity has
been determined
A. True
B. False

1594.
942.

Calcium disodium edetate is superior to dimercaprol in the


treatment of chronic intoxication of
A. carbon monoxide
B. cyanide
C. ethanol
D. lead

943.
1595.

A 4 years old girl drank an unknown quantity of an insect


spray from a coke bottle found in garage. She vomited on the
way to the hospital and had a convulsive seizure. On
admission, she was in coma and had respiratory problems.
Amyl nitrite by inhalation followed by I.V. sodium nitrite
should be given immediately to this child
A. True
B. False

256
Pharmacy

1596.
944.

Chronic poisoning seen most commonly as peripheral


neuropathy in adults but as encephalopahthy in children
A. arsenic
B. lead
C. mercury
D. chromium

1597.
945.

A 4 years old girl drank an unknown quantity of an insect


spray from a coke bottle found in garage. She vomited on the
way to the hospital and had a convulsive seizure. On
admission, she was in coma and had respiratory problems.
A normal value for blood cholinesterase would be strong
evidence against organophosphate poisoning in this girl
A. True
B. False

1598.
946.

Inhibition of heme synthesis


A. lead salts (inorganic, chronic administration)
B. mercury salts (inorganic, chronic administration)
C. Both
D. Neither

947.
1599.

The symptoms of organophosphate poisoning usually begin


within 12 hours after exposure to the organic phosphate
insecticide
A. True.
B. False

1600.
949.

Pentavalent arsenicals
A. are less toxic than trivalent arenicals
B. may cause liver damage
C. may cause kidney damage
D. all of the above

1601.
951.

Which of the following blood levels also should be done


usually when a blood level for digoxin is done?

A.
B.
C.
D.

Calcium
Potassium
Sodium
Chloride

257
Pharmacy

953.
1602.

In an acetaminophen overdose, the organ most damaged is


the
A. kidney
B. brain
C. liver
D. spleen

955.
1603.

All of the following statements concerning the therapeutic


use of the acetaminophen are true EXCEPT:
A. two hours is the half life for therapeutic doses
B. a normal therapeutic dose is 40 to 50 mg/kg every 4
hours
C. peak plasma levels are reached in one hour after
ingestion of 650mg
D. more than one of the above

1604.
956.

Which of the following may be factors in digoxin toxicity?

A.
B.
C.
D.

Hypokalemia
Drug interactions
Hypothyroidism
All of the above

957.
1605.

A patient is brought to the Emergency room with an


acute acetaminophen overdose. Seven hours later the plasma
level of acetaminophen is higher than the admission
plasma level. On this basis which of the following is most
likely to occur?
A. No damage, this is the normal half life for
acetaminophen
B. Coma and death of the patient
C. Hepatic damage
D. None of the above

1606.
958.

N acetylcysteine is administered
A. orally
B. intravenously
C. Both A & B
D. Neither A nor B

1607.
959.

The substance involved in the endogenous detoxification of


acetaminophen is
A. homocysteine
B. glutathione
C. methione
D. glycine

258
Pharmacy

1608.
960.

A 32 year old comatose woman with slow and shallow


respiration and pinpoint pupils most likely took an overdose
of
A. acetaminophen
B. amphetamine
C. heroin
D. phencyclidine

961.
1609.

Which of the following statements concerning endogenous


glutathione is correct
A. endogenous glutathione does not become depleted in
normal daily usage of acetaminophen metabolite
B. intravenous injections of glutathione will not increase
the endogenous supply
C. the sulfhydryl side chain of glutathione cleaves the
acetaminophen metabolite
D. all of the above

1610.
962.

A 35 year old farmer exposed to toxic level of a pesticides


was treated with atropine plus pralidoxime. He was most
likely exposed to
A. aldicarb
B. carbaryl
C. 2,4 dichlorophenoxyacetic acid
D. malathion

1611.
963.

Which of the following are symptoms consistent with phase


one in an acetaminophen overdose?
A. Nausea
B. Right upper quadrant pain
C. Vo miting
D. A & C

1612.
964.

A 72 year old woman brought to the emergency department


with symptoms of fever, severe gastritis, vomiting, and
dehydration 24 hours after a family picnic has likely been
exposed to
A. bee sting
B. botulinus toxin food poisoning
C. pesticides
D. salmonella food poisoning

1613.
965.

In phase two of an acute acetaminophen overdose, which of


the folloing observations can usually be found?
A. Decrease in SGOT level
B. Lower right guardant pain
C. Faster prothrombin time
D. Increase bilirubin level

259
Pharmacy

1614.
966.

967.
1615.

1616.
968.

969.
1617.

1618.
970.

1619.
971.

972.
1620.

1621.
973.

If a 7 year old child has symptoms of severe gastrointestinal


distress, abdominal colic, and signs and symptoms of
encephalopathy after exposure to an acute metal poisoning,
he would be most appropriately treated with
A. acetylcysteine
B. calcium disodium edetate
C. deferoxamine
D. penicillamine
As a group, cyclic antidepressants have which of the
following characteristics?
A. They have similar efficacy
B. They have potent anticholinergic activity
C. They lack H1 blocking activity
D. They prevent the synthesis of neurotransmitters.
Which of the following relatively mild adverse effects is
associated with initiation of lithium therapy?
A. Blurred vision
B. Dystonic reactions
C. Fine hand tremor
D. Tinnitus
Foods high in tyramine such as pickled herring and most
cheeses should be avoided by patients who are taking:
A. Doxepin
B. Phenelzine
C. Maprotiline
D. Alprazolam
Which of the following drugs prescribed for control of a
major affective disorder is associated with an almost
immediate onset of activity?
A. Lithium carbonate
B. Protryptilline
C. Tranylcypromine
D. Chlorpromazine
Under the biogenic amine hypothesis, mania is thought to be
due to:
A. An excess of epinephrine activity
B. An excess of norepinephrine activity
C. An excess of dopamine activity
D. An deficiency of epinephrine activity
All of the following symptoms are potential
anticholinergic side effects of amitriptylline except:
A. Blurred vision
B. Constipation
C. Sinus bradycardia
D. Urinary retention
The symptoms of allergen mediated asthma result from:
A. Increased release of mediators from mast cells
B. Increased adrenergic responsiveness of the airways
C. Increased vascular permeability of bronchial tissue
D. Decreased calcium influx into the cell

260
Pharmacy

974.
1622.

975.
1623.

976.
1624.

977.
1625.

978.
1626.

1627.
979.

1628.
980.

1629.
981.

Acute exacerbations of asthma can be triggered by all of the


following EXCEPT:
A. Bacterial or viral pneumonia
B. Hypersensitivity reaction to Penicillin
C. Hot dry weather
D. Discontinuation of asthma medication
The selection of an oral theophylline product depends on:
A. The percentage of theophylline content of the product
B. Pre existing disease states (eg. Gout, peptic ulcer
disease)
C. Theophylline half life
D. Concurrent asthma medication
In the emergency room, the preferred first line therapy for
asthma is:
A. Theophylline
B. Beta agonist
C. A corticosteroid
D. Cromolyn sodium
The primary goals of asthma therapy include all of the
following EXCEPT:
A. Maintain normal activity levels
B. Maintain control of symptoms
C. Prevent lung tissue destruction
D. Prevent acute exacerbation of chronic symptoms
In the treatment of COPD, Corticosteoids:
A. Are more effective than in Asthma
B. Are more beneficial when used alone
C. Produce more side effects when used in the aerosol form
D. Should be used for at least two weeks before efficacy is
assessed.
A 50 year old woman is admitted to hospital with a chief
complaint of bilateral swelling of her knee and early morning
stiffness and lethargy. She is not responding to 650 mg of
Aspirin three times daily. A diagnosis of rheumatoid
arthritis has been made. Initial suggestion for drug therapy
in this patient is :
A. Hydroxychloroquine, 200mg daily
B. Steroid injection into all swollen joints
C. Aspirin 975 mg four times a day
D. Penicillamine 250 mg four times a day
Which of the following agents and dosage regimens is the
best choice treatment for a patient with rheumatoid arthritis
who is considered sensitive to aspirin?
A. Ibuprofen 800 mg three times daily
B. Paracetamol 650 mg every 4 hours
C. Gold injections, 25mg intramuscularly once a week
D. Azathiporine 75 mg daily
Which of the following statements best describes the usual
course of rheumatoid arthritis?
A. It is an acute exacerbation of joint pain treated with short

261
Pharmacy

1630.
982.

983.
1631.

984.
1632.

1633.
985.

1634.
986.

term antiinflamatory therapy.


B. It is a chronic disease characterized by acute changes
within non synovial joints.
C. It is an acute disease that is characterized by rapid
synovial changes due to inflammation.
D. It is a chronic disease characterized by acute
exacerbations followed by remissions with consequences
associated with chronic inflammatory changes.
All of the following statements concerning an acute gouty
attack are correct EXCEPT :
A. The diagnosis of gout is assured by a good therapeutic
response to colchicines because no other form of arthritis
responds to this drug.
B. To be assured of the diagnosis, monosodium urate
crystals must be identified in the synovial fluid of the
affected joint.
C. Attacks frequently occur in the middle of the night.
D. an untreated attack may last up to 2 weeks
A 42 year old man diagnosed to have gout, rational treatment
of this patient between the gouty attacks might include any
of the following EXCEPT :
A. Paracetamol or aspirin 650 mg as needed for joint pain
B. Probenecid
C. Colchicine
D. Allopurinol
Which of the following organisms has been implicated as a
possible cause of chronic gastritis and peptic ulcer disease?
A. Campylobacter jejuni
B. Escheritia coli
C. Helicobacter pylori
D. Giardia lamblia
All of the following statements concerning antacid therapy
used in the treatment of duodenal or gastric ulcers are correct
EXCEPT:
A. Antacids may be used to heal ulcer but are ineffective in
controlling ulcer pain
B. Antacids neutralize acid and decrease the activity of
pepsin.
C. If used alone for ulcer therapy, antacids should be
administered 1 hour and 3 hours after meals and at bed time.
D. If diarrhea occurs the patient may alternate the antacid
product with aluminum hydroxide
As part of the comprehensive management strategy to treat
peptic ulcer, patients should be encouraged to do all of the
following EXCEPT :
A. Decrease caffeine ingestion
B. Eat only mild foods without spice
C. Stop smoking
D. Avoid alcohol

B
Bland foods are no longer
recommended. Research
indicates bland foods do not
accelerate ulcer healing.

262
Pharmacy

1635.
987.

A gastric ulcer patient requires close follow up to document


complete ulcer healing because:
A. Perforation into the intestine is common
B. Spontaneous healing of the ulcer may occur in 30% to
50% of the cases.
C. There is risk of the ulcer being cancerous
D. Weight loss may be severe

1636.
988.

Current criteria used in the diagnosis of diabetes mellitus


include all of the following symptoms EXCEPT :
A. Fasting hyperglycemia
B. Ployuria
C. Polydipsia
D. Tinnitus

989.
1637.

The most useful glucose test used in monitoring diabetes


mellitus therapy is:
A. Urine monitoring
B. Blood monitoring
C. Renal function monitoring
D. Cardiovascular monitoring
Which of the following statements about insulin is (are) true?
A. Insulin is a hormone
B. Insulin is a protein
C. Insulin is secreted by the islets of Langerhans
D. All of the above
A mass of adipose tissue that develops at the injection site is
usually due to the patients neglect in rotating the insulin
injection site. This is known as:
A. Lipoatrophy
B. Degenerative adiposity
C. Lipohypertrophy
D. Atrophic skin lesion
Sulfonylureas are a primar y mode of therapy in the
treatment of:
A. Insulin dependent type1 diabetes mellitus patients
B. Diabetic patients experiencing severe hepatic dysfunction
C. Diabetic pregnant women
D. Non insulin dependent type2 DM patients
Patients taking chlorpropamide should avoid products
containing :
A. Acetaminophen
B. Ethanol
C. Vitamin A
D. Milk products
The standard recommended dose of Glyburide is :
A. 0.5 to 2 mg per day
B. 1.25 to 20 mg per day
C. 50 to 200 mg per day
D. 200 mg per day

1638.
990.

991.
1639.

992.
1640.

1641.
993.

994.
1642.

263
Pharmacy

995.
1643.

What precursor besides dietary iodine is required for


thyroxine biosynthesis?
A. Triiodothyronine
B. Threonine
C. Tyrosine
D. Thyroxine binding globulin
All of the following conditions are causes of
hyperthyroidism EXCEPT :
A. Graves disease
B. Hashimotos thyroiditis
C. Toxic nodular goiter
D. Plummers disease
Which of the following measures would the nurse expect to
include in the teaching plan for a multiparous client who
delivered 24 hours ago and is receiving intravenous
antibiotic therapy for cystitis?
A. Limiting fluid intake to 1 L daily to prevent overload
B. Emptying the bladder every 2 to 4 hours while awake.
C. Washing the perineum w ith povidone iodine (Betadine)
after voiding.
D. Avoiding the intake of acidic fruit juices until the
treatment is discontinued.
The nurse teaches her client, who has recently been
diagnosed with hypertension, about his dietary restrictions: a
low calorie, low fat, low sodium diet. Which of the following
menu selections would best meet the clients needs?
A. Mixed green salad with blue cheese dressing, crackers,
and cold cuts
B. Ham sandwich on rye bread and an orange
C. Baked chicken, an apple, and a slice of white bread.
D. Hot dogs, baked beans, and celery and carrot sticks.
Which of the following preparations is used to attain
remission of thyrotoxicosis?
A. Propranolol
B. Liotrix
C. Levothyroxine
D. Propylthiouracil

1648.
1000.

The thyroid gland normally secretes which of the following


substances into the serum?
A. Thyroxine
B. Thyroglobulin
C. Diiodothyronine
D. Thyrotropin

1001.
1649.

All of the following conditions are causes of


hypothyroidism EXCEPT :
A. Graves disease
B. Hashimotos thyroiditis
C. Endemic goitre
D. Surgical excision

1644.
996.

997.
1645.

1646.
998.

1647.
999.

264
Pharmacy

1002.
1650.

1651.
1003.

1004.
1652.

1005.
1653.

1006.
1654.

1007.
1655.

1008.
1656.

1009.
1657.

1010.
1658.

Common tests to monitor patients on replacement therapy in


hypothyroidism include all of the following EXCEPT :
A. TSH stimulation
B. Sensitive TSH assay
C. Free thyroxin index
D. Total thyroxin
The protein precursor of thyroxin is
A. ornithine
B. threonine
C. tryptophan
D. thyroglobulin
The inhibition of pituitary thyrotropin secretion is controlled
by which of the following?
A. TRH
B. Free thyroxine
C. Total thyroxine
D. Reverse triiodothyronine
Which of the following agents has been shown to interact
with oral thyroxine replacement therapy?
A. Cholestyramine
B. Propylthiouracil
C. Thyrotropin
D. Lovastatin
What lab tests are currently recommended to diagnose
thyroid disease?
A. Triiodothyronine resin uptake and total thyroxine
B. Tyrotropin and free thyroxine index
C. Total thyroxine
D. Free T4 and sensitive TSH assay
What patient population should be screened for thyroid
disease?
A. Hospitalized patients
B. Elderly patients with chronic disease
C. Elderly hospitalized patients
D. Women over 20 years old
What is the average replacement dose of levothyroxine for
an otherwise healthy adult?
A. 25-50g/day
B. 50-100 g/day
C. 75-150 g/day
D. 100-200 g/day
What factors affect the optimal replacement dose of
levothyroxine?
A. Age, height and weight
B. Duration of hypothyroidism
C. Pretreatment TSH level
D. All of the above
A patient with multiple sclerosis is receiving Baclofen. The
nurse determines that the drug is effective when it achieves
which of the following?

265
Pharmacy

1011.
1659.

1012.
1660.

1013.
1661.

1014.
1662.

1015.
1663.

1664.
1016.

1665.
1017.

A. Induces sleep
B. Stimulates the appetite
C. Relieves muscular spasticity
D. All of the above
The nurse is planning to teach the client how to properly use
a metered dose inhaler of corticosteroid to treat asthma.
Which of the following instructions should the nurse include
in the teaching plan?
A. Rinse the mouth after each use
B. Inhale quickly while administering the medication
C. inhale the medication and then exhale through the nose
D. Cough and take a deep breath
An older woman with Parkinsons disease is placed in a
nursing home for basic nursing care. When the nurse
observes that the client has difficulty swallowing the
capsules, which action is best to take?
A. Offer water before giving her the capsule
B. Soak the capsule in water until soft
C. Tell her to chew the capsule
D. Empty the capsule in the clients mouth
An older woman with Parkinsons disease is placed in a
nursing home for basic nursing care. Which instruction
should the nurse plan to give while administering the
Nystatin oral suspension?
A. Drink the medication through a straw
B. Dilute the medication with cold water
C. Retain the drug as long as possible in the mouth
D. Swish the drug in the mouth but avoid swallowing it
A common storage condition for most biotechnology
products after reconstitution is:
A. Room temperature
B. Cool place
C. Warm place
D. Freezer
What are the most common adverse effects of
anticonvulsive drugs?
A. Headache and dizziness
B. GI symptoms
C. Alternation of cognition and mentation
D. All of the above
In choosing a study perspective the current
pharmacoeconomic guidelines have suggested which one of
these perspectives to be included?
A. Society
B. Payers
C. Patients
D. All of the above
The primary use of antipsychotics is the treatment of :
A. Schizophrenia
B. Depression

266
Pharmacy

1018.
1666.

1667.
1019.

1020.
1668.

1021.
1669.

1022.
1670.

1023.
1671.

1024.
1672.

1673.
1025.

C. Epilepsy
D. Alcoholism
Other uses of antipsychotics include all of the following
EXCEPT:
A. Tourettes syndrome
B. Acute mania
C. Alcoholic hallucinosis
D. Seizures
The dopamine hypothesis of schizophrenia proposes that :
A. schizophrenia results from degeneration of the
nigrostriatal dopamine system
B. schizophrenia results from hyperactivity of nigrostriatal
dopamine system
C. schizophrenia results from hyperactivity of the
mesolimbic and/or mesocortical dopamine system
D. schizophrenia results from hypoactivity of the
mesolimbic and/or mesocortical dopamine system
As a group, antipsychotic drugs may interact with all of the
following receptors EXCEPT:
A. D 1 Dopamine
B. D 2 Dopamine
C. Alpha adrenergic
D. Beta adrenergic
The therapeutic effect of antipsychotic drugs are believed to
result from blockade of which of the following receptors?
A. D 1 Dopamine
B. D 2 Dopamine
C. Alpha adrenergic
D. Beta adrenergic
Antipsychotic drugs can produce all of the following side
effects EXCEPT:
A. Dry mouth
B. Akathasia
C. Dystonia
D. Urinary urgency
Extrapyramidal effects which consists of Parkinsonian
symptoms including tremor, rigidity, and bradykinesia result
from the blockade of which receptors?
A. Dopamine
B. Cholinergic
C. Alpha adrenergic
D. Beta adrenergic
Postural hypotension results from the blockade of which
receptors?
A. Dopamine
B. Cholinergic
C. Alpha adrenergic
D. Beta adrenergic
Antipsychotic side effects such as dry mouth and urinary
hesitance result from blockade of which receptors?

267
Pharmacy

1026.
1674.

1675.
1027.

1028.
1676.

1677.
1029.

1030.
1678.

1031.
1679.

1680.
1032.

1033.
1681.

A. Dopamine
B. Cholinergic
C. Alpha adrenergic
D. Beta adrenergic
Tardive dyskinesia
A. occurs in some patients after prolonged treatment with
anti psychotics
B. involves abnormal limb and oral movements
C. May be irreversible
D. All of the above
Gama aminobytyric acid (GABA) functions as a
A. dehydrogenase enzyme
B. neurotransmitter
C. insulin precursor
D. amino acid precursor
All of the following are true of neuroleptic malignant
syndrome EXCEPT:
A. is a potentially life threatening malignant hyperthermia
B. is unrelated to other side effects
C. is aggravated by drugs with anticholinergic activity
D. is treated with dantrolene
Which antipsychotics have the highest liability for
extrapyramidal effects?
A. Aliphatic phenothiazines
B. Thioxanthines
C. Butyrophenones
D. Clozapine
Identify the Buterophenone:
A. Haloperidol
B. Clozapine
C. Olanzepine
D. Molindone
Which drug has the greatest liability of postural
hypotension?
A. Chlorpromazine
B. Fluphenazine
C. Thiothixene
D. Risperidone
All of the following has liability of anticholinergic effects
EXCEPT:
A. Chlorpromazine
B. Fluphenazine
C. Thiothixene
D. Haloperidol
Extrapyramidal effects can be treated with :
A. Trihexyphenidyl
B. Dantrolene
C. Baclofen
D. Bromocriptine

268
Pharmacy

1034.
1682.

1683.
1035.

1036.
1684.

1685.
1037.

1686.
1038.

1687.
1039.

1040.
1688.

1041.
1689.

Which drug has the lowest incidence of Extrapyramidal side


effect and tardive dyskinesia?
A. Chlorpromazine
B. Fluphenazine
C. Clozapine
D. Risperidone
Social considerations for use of clozapine include:
A. It is expensive
B. It is approved only for use in patients who were not
satisfactorily treated by other drugs.
C. It can produce agranulocytosis which is potentially fatal.
D. All of the above
Which of the following statements is FALSE?
A. Bipolar disorder involves periods of depressed mood and
periods of mania.
B. Unipolar disorder involves periods of depressed mood
C. Mood stabilizing agents are used commonly in the
treatment of unipolar disorder
D. Antidepressants can be used in the treatment of bipolar
disorder.
Symptoms of depression can include:
A. Feelings of worthlesslessness
B. Suicidal thoughts
C. Feelings of guilt
D. All of the above
Which of the following is an MAOI ?
A. Carbamazepine
B. Tranylcypromine
C. Paroxetine
D. Nefazodone
Imipenem is a betalactam antibiotic which is neither a
penicillin nor a cephalosporin. Correct statements regarding
imipenem include:
A. it should not be given to patients having a history of
allergic reactions to penicillin
B. it is very active against many gram negative rods
C. resistant pseudomonas may emerge during treatment
D. All of the above
In addition to their anxiolytic properties, benzodiazepines are
indicated for use:
A. As preanesthetic medications
B. As anticonvulsants
C. during acute withdrawal of alcohol
D. All of the above
Emulsions made with tweens are usually
A. unstable
B. w/o
C. o/w
D. clear

269
Pharmacy

1690.
1042.

Which of the following factors affect the distribution of a


drug?
A. Lipid solubility
B. Plasma protein binding
C. Polarity
D. All of the above

1691.
1043.

The rate of zero order reactions


A. changes constantly
B. is independent of temperature
C. is independent of concentration
D. holds only for lightcatalyzed reactions

1044.
1692.

Spans and tweens are


A. highly polymerized mannuronic acid anhydrides
B. phospholipids
C. non ionic emulsifying agents
D. glycosides

1693.
1046.

The two major properties of drugs that are usually modified


by complexation are
A. stability and solubility
B. taste and solubility
C. chemical structure and solubility
D. all of the above

1694.
1047.

What will result if the distribution of drugs is slower than the


processes of biotransformation and elimination?
A. high blood levels of drug
B. low blood levels of drug
C. failure to attain diffusion equilibrium
D. Potentiation

1695.
1051.

Which of the following drugs undergo marked hydrolysis in


the GI tract?
A. Aspirin
B. Penicillin G
C. Acetaminophen
D. Hydrocortisone

1696.
1052.

Precipitated sulfur is often incorporated into ointments to be


used as a?
A. Parasiticide (e.g., for scabies)
B. emollient
C. mild keratolytic
D. A & C only

270
Pharmacy

1697.
1053.

Solutions that contain bacteriostatic agents


A. cannot be tested for sterility in normal state
B. must be cultured on agar plates for sterility tests
C. must be diluted beyond the bacteriostatic level for
sterility tests
D. do not require a sterility test

1059.
1698.

A prescription calls for 25 mEq of potassium chloride. How


many grams of KC1 (MW 74.6) are needed?
A. 7.46 g
B. 0.746 g
C. 8.86 g
D. 1.86 g

1060.
1699.

An IV order requires 5 million units of sodium penicillin G


to be added to 100 ml of normal saline. How many mEq of
sodium are present in this solution?
A. 154 mEq
B. 10 mEq
C. 8.4 mEq
D. 25.4 mEq

1700.
1061.

An order calls for 500 mL of a solution of potassium sulfate


to be made so that it contains 10 mEq of K+. How many
grams of potassium sulfate are required?
A. 0.440 g
B. 4.44 g
C. 0.044 g
D. 0.870 g

1701.
1062.

How many milliliters of a 10% KC1 (MW 74.6) solution


contain 5.0 mEq of K+?
A. 2.10 mL
B. 21.0 mL
C. 3.73 mL
D. 37.3 mL

1702.
1063.

A doctor prescribes one and a half tablet of Lasix available in D


the market in Oman for a period of one month to a patient
diagnosed to have hypertension. How many tablets will be
dispensed to the patient?
A. 60
B. 75
C. 30
D. 45

271
Pharmacy

1066.
1703.

Sodium bicarbonate is compatible with which of the


following IV medications?
A. Potassium chloride
B. Calcium salts
C. Atropine
D. Catecholamines

1704.
1067.

The pKw of water at 25C is


A. 7
B. 14
C. 1 x 1014
D. 1 x 107

1705.
1068.

The enzymes involved in ethanol metabolism are primarily:

A.
B.
C.
D.

phosphorylases
dehydrogenases
lipases
ketases

1706.
1069.

When making pharmacokinetic recommendations in


aminoglycoside antibiotics dosing, if you want to decrease
only the trough concentration, you would
A. decrease the dose
B. take the trough measurement 3 hrs before the next dose
C. lengthen the dosing interval
D. increase the dose

1707.
1070.

The dose of a drug is 0.5 mg/kg. What dose should be given


a 6yearold child who weighs 44 lb?
A. 0.003 g
B. 0.033 g
C. 0.010 g
D. 0.100 g

1708.
1071.

If prescription order requires 30 gs of concentrated sulfuric


acid (density is 1.8 g/mL, what volume should the
pharmacist measure?
A. 1.67 mL
B. 18 mL
C. 30 mL
D. 16.67 mL

272
Pharmacy

1709.
1072.

Calculate the weight of 25 mL of hydrochloric acid whose


density is 1.18 g/mL
A. 29.5 g
B. 2.95 g
C. 0.295 g
D. 295 g

1710.
1073.

Most nitrogenous waste is converted to urea because:


I. Ammonia and uric acid are toxic
II. urea is highly soluble
III. urea is nontoxic
A. I only
B. II only
C. I &II only
D. I, II, and III

1711.
1074.

To calculate a loading dose, one must first determine


A. t 1/2
B. body clearance
C. fraction protein bound
D. volume of distribution

1712.
1075.

Convert 60 grams to grains


A. 9240 grains
B. 924 grains
C. 9.24 grains
D. 0.924 grains

1713.
1076.

Radioactive decay follows a


A. mixedorder rate
B. fractionalorder rate
C. zeroorder rate
D. firstorder rate

1714.
1077.

The HLB system is used to classify


A. flavors
B. colors
C. surfactants
D. organic ring structures

1715.
1078.

Alpha particles are very similar to


A. hydrogen atoms
B. helium atoms
C. neutrons
D. protons

273
Pharmacy

1079.
1716.

To achieve the same steadystate plasma concentration (for a


drug that is excreted by the kidney) in renal failure patients
as in patients with normal renal function, you should
A. increase the dosing interval
B. decrease the dose
C. adjust both the dose and dosing interval
D. do any of the above, depending on the
pharmacodynamic properties of the drug.

1080.
1717.

The most common disintegrator in compressed tablets is


A. dextrose
B. lactose
C. starch
D. potassium bitartrate

1081.
1718.

Freons are
A. alkanes
B. alkenes
C. alkynes
D. fluorinated hydrocarbons

1082.
1719.

Which of the following factors may make it necessary to


give lower doses of drugs to geriatric patients?
A. Reduced enzyme activity
B. Reduced kidney function
C. Enhanced absorption
D. A & B only

1083.
1720.

Convert 2 pints 3 fluid ounces into mL.


A. 1500 mL
B. 1050 mL
C. 150 mL
D. 105 mL

1084.
1721.

Tablet hardness range is normally


A. 0.5 to 1.0 kg
B. 1.0 to 2.0 kg
C. 2.0 to 3.5 kg
D. 3.5 to 7.0 kg

1722.
1086.

The dose of a drug is 0.6 mg. How many doses are


contained in 96 mg of the drug?
A. 16
B. 160
C. 360
D. 600

274
Pharmacy

1723.
1088.

The expression ppm as used in compounding prescriptions


most often is defined as:
A. parts per million
B. parts per mL
C. parts per mole
D. pieces per million

1724.
1089.

GMP regulations (USFDA) primarily apply to


A. controlled drugs
B. wholesalers
C. pharmaceutical manufacturers
D. hospital pharmacy

1725.
1090.

pH is equivalent to pka at
A. pH of 1
B. pH of 7
C. half neutralization point
D. neutralization point

1726.
1093.

In dosing aminoglycoside antibiotics, which of the following


patient population commonly require an extended interval
(>8 hours)?
A. elderly patients
B. neonatal patients
C. cystic fibrosis patients
D. A & B

1094.
1727.

Glycerin has a specific gravity of 1.25. One gallon weighs


A. 591.25 g
B. 473 g
C. 4528.0 g
D. 128 g

1728.
1097.

Another name for polyethylene glycol polymers is


A. sodium alginate
B. silica gel
C. carbowax
D. friar paste

275
Pharmacy

1729.
1100.

How many milli grams of the medicament is available in a


syrup containing 10% w/v?
A. 10 g in 100 ml
B. 10 mg in 10 ml
C. 10 mg in 100 ml
D. 10 g in 1000 ml

1103.
1730.

Which of the following ions plays a significant role in


preventing convulsions?
A. Potassium
B. Lithium
C. Magnesium
D. Phosphorus

1731.
1106.

Separation of emulsions is characterized by


A. irreversible precipitation
B. reversible precipitation
C. creaming
D. changing of external phase

1732.
1108.

Freeze drying is based on


A. pressure filtration
B. sublimation
C. polymerization
D. pasteurization

1733.
1109.

Lubricants in tablets serve the following functions.


I. They improve the follow of tablet granulation
II. They prevent adhesion to dies and punches
III. They facilitate ejection from die cavity
A.
B.
C.
D.

I only
II only
I and II only
I, II and III

276
Pharmacy

1734.
1110.

1735.
1112.

1736.
1113.

The most prevalent commercial solid dosage forms are


A. hard capsules
B. soft gelatin capsules
C. tablets
D. bulk powders

The purpose of sorbitol in formulations of soft gelatin


capsules is as a(n)
A. plasticizer
B. disintegrating agent
C. lubricant
D. thickener

Hard gelatin capsules for human use are available in the


following sizes.
I. 1
II. 00
III. 0000

A.
B.
C.
D.

1737.
1114.

Ointments are typically used as


I. emolients
II. protective barriers
III. vehicles for applying drugs
A.
B.
C.
D.

1738.
1115.

I only
II only
III only
I and II only

I only
III only
I and II only
I, II and III

A humectant retards
A. bacterial growth
B. degradation
C. surface evaporation
D. spreadability

277
Pharmacy

1739.
1116.

The NoyesWhitney equation describes


A. zeroorder kinetics
B. firstorder kinetics
C. mixedorder kinetics
D. dissolution rate

1117.
1740.

Starch is used in tabletting as a


I. binder
II. glidant
III. disintegrant

A.
B.
C.
D.

II only
III only
II and III only
I, II and III

1741.
1118.

Which of the following properties are characteristic of


flocculated particles in suspension?
A. Particles from loose aggregates
B. Rate of sedimentation is high
C. A sediment is formed rapidly
D. All of the above

1742.
1119.

Gums are used in tabletting primarily as


A. disintegrators
B. glidants
C. lubricants
D. binding agents

1120.
1743.

Which of the following is (are) true for buccal and


sublingual tablets?
I. They are useful for drugs destroyed by gastric fluid.
II. They are readily soluble
III. They are useful for drugs that are poorly absorbed in the
intestinal tract

A.
B.
C.
D.
1121.
1744.

I and II only
I and III only
II only
I, II and III

Vanishing creams are classified as


A. oleaginous
B. absorption bases
C. o/w bases
D. w/o bases

278
Pharmacy

1745.
1122.

Syrup NF is
A. selfpreserving
B. a supersaturated solution
C. a dilute solution
D. highly unstable

1746.
1123.

Which of the following would be classified as a strong acid


according to acidbase theory?
A. 10% acetic acid
B. 5% boric acid
C. 0.5% hydrochloric acid
D. 100% oleic acid

1124.
1747.

Reaction rate is increased most readily by


A. humidity
B. high temperature
C. freezing
D. photolysis

1748.
1125.

Dissolution rate is increased by


I. an increase in surface area
II. a decrease in particle size
III. the formation of molecular aggregates

A.
B.
C.
D.

I only
II only
II and III only
I and II only

1126.
1749.

Units for renal clearance are in


A. mg/L
B. g/L
C. mL/min
D. mg%

1750.
1128.

A primary disadvantage of ethylene glycol as a solvent in


oral preparations is its
A. potential toxicity
B. lack of solvent action
C. very limited miscibility
D. high cost

279
Pharmacy

1751.
1129.

Glucose is not subject to hydrolysis because it is


A. a disaccharide
B. a monosaccharide
C. a polysaccharide
D. insoluble

1130.
1752.

Purified water USP ma y not be used in


A. syrups
B. topical preparations
C. parenteral preparations
D. elixirs

1753.
1131.

Ferritin is a(n)
A. vitamin
B. micelle
C. emulsion
D. amino acid

1754.
1132.

Vitamin B 6 is also known as


A. thiamine
B. riboflavin
C. niacin
D. pyridoxine

1755.
1133.

HLB is a system used to distinguish between


A. surfactants
B. glidants
C. suspending agents
D. excipients

1756.
1134.

Disadvantages of chlorobutamol in germicide solutions


include

I.
II.
III.
A.
B.
C.
D.
1757.
1135.

heat instability
instability at alkaline pH
loss of potency in presence of air
I only
II only
I & II only
I, II & III

A solution has an osmolal concentration of 1 when it


contains:
A.
B.
C.
D.

1 osmol of solute/kg of water


1 osmol of solute/mL of water
1 osmol of solute/L water
1 osmol of solute/kg of sodium chloride

280
Pharmacy

1758.
1136.

Sunscreen preparations containing paminobenzoic acid


should be
A. used sparingly
B. applied 2 hours before exposure
C. packaged in airtight containers
D. packaged in lightresistant containers

1759.
1137.

Which of the following statements is (are) true for


undecylenic acid?
I.
It is often used as the zinc salt
II. It may cause irritation and sensitization
III. It is the most fungistatic of the fatty acids

A.
B.
C.
D.

I only
II and II only
II only
I, II and III

1760.
1138.

Which of the following is classified as fat soluble?


A. Vitamin D
B. Niacin
C. Ascorbic acod
D. Thiamine hydrochloride

1761.
1139.

The active constituents of saline laxatives are


A. absorbable anions
B. nonabsorbable cations
C. tribasic cations
D. nonabsorbable cations and anions

1762.
1140.

Bismuth subsalicylate is used in antidiarrheals for its


A. hydrophobic action
B. hydrophilic action
C. adsorbent action
D. absorbent action

1141.
1763.

Sublimation is the term applied to the following type(s) of


transformation
I. Solid to vapor
II. Liquid to solid
III. Liquid to vapor

1142.
1764.

A. I only
B. II only
C. III only
D. I & II only
Which of the following is a cationic emulsifing agent?
A. Potassium laurate

281
Pharmacy

B. Calcium oleate
C. Sodium lauryl sulfate
D. Cetyltrimethylammonium bromide
1765.
1143.

Mineral oil exerts laxative action primarily by


A. bulk formation
B. fecal softening
C. catharsis
D. lubrication

1766.
1144.

Cellulose acetate phthalate is used in pharmacy as a(n)


A. emulsifier
B. enteric coating material
C. suspending agent
D. flavouring agent

1767.
1145.

Cocoa butter (theobroma oil) is useful as a suppository base


because of its
A. solubility
B. melting point
C. miscibility
D. reactivity

1768.
1146.

The most widely used method for sterilization of


pharmaceuticals is
A. microfiltration
B. radiation
C. ethylene oxide exposure
D. moist heat

1769.
1147.

Ophthalmic solutions should be formulated to include which


of the following?
I. Sterility
II. Isotonicity
III. Buffering

A.
B.
C.
D.
1770.
1148.

I only
II only
I and II only
I, II and III

One form of the buffer equation is pH = pKa + log salt/acid.


For in this equation, the salt and acid concentration is expressed
A.
B.
C.
D.

g/100 mL
moles
mg %
g/L

282
Pharmacy

1771.
1149.

If a buffer solution is prepared using equal concentrations of


acetic acid and sodium acetate, the pH would then be
A.
B.
C.
D.

1
14
equal to the pKa
equal to of the pKa

1772.
1150.

Another name for the buffer equation is the


A. Arrhenius equation
B. HendersonHasselbalch equation
C. Debye Huckel equation
D. buffer capacity equation

1773.
1151.

Which of the following is an ampholyte?


A. H 2 CO 3
B. HCl
C. NaOH
D. NaH 2 PO 4

1152.
1774.

Soda lime is used as a(n)


A. alkalinizer
B. therapeutic agent in topical preparations
C. stabilizer in emulsions
D. reagent for absorption of carbon dioxide

1153.
1775.

Pumice is often used in


I. soaps and cleansing powders
II. dental preparation
III. filtering

A.
B.
C.
D.

I only
II only
III only
I, II and III

1776.
1154.

There was no requirement to establish efficacy for drug


products until
A. 1906
B. 1938
C. 1962
D. 1965

1777.
1155.

The type of instability known as bleeding is usually


associated with
A. isotonic solutions
B. emulsions
C. alcoholic solutions
D. ointments

283
Pharmacy

1778.
1156.

The GMP regulations first became official in


A. 1938
B. 1962
C. 1963
D. 1979

1779.
1157.

The Limulus test is a relatively new method of testing for


A. pyrogens
B. microbial growth
C. acidity
D. creaming

1780.
1158.

Which of the following is associated with excessive infusion


of hypotonic fluids
A. Hemolysis
B. Hyperglycemia
C. Dehydration
D. Glycosuria

1159.
1781.

Which of the following is (are) associated with excessive


infusion of hypertonic dextrose solutions?
A. Loss of electrolytes
B. Hyperglycemia
C. Dehydration
D. All of the above

1782.
1160.

Normal rectal temperature is usually


A. measured in the morning
B. measured in the evening
C. about the same as the normal oral temperature
D. about 1 higher than oral

1161.
1783.

The Latin oculo utro is translated to mean


A. right eye
B. each eye
C. left eye
D. each ear

1784.
1162.

Advantages of tablets over liquid dose forms include the


following:
I. Enhanced stability
II. Ease of administration
III. Greater accuracy of dosing
A. II only
B. III only
C. I and II only
D. I, II and III

1785.
1163.

Which of the following is (are) exempted from safety


packaging?
I. Oral contraceptives

284
Pharmacy

II. Nitroglycerin
III. Oral analgesics
A.
B.
C.
D.

I only
II only
I and II only
II and III only

1164.
1786.

The Latin post cibos can be translated to mean


A. after meals
B. before meals
C. with meals
D. without food

1787.
1165.

Ferrous sulfate oral drops contain 75 m/0.6 mL. How much


elemental iron does this provide?
A. 75 mg
B. 50 mg
C. 25 mg
D. 15 mg

1166.
1788.

Prednisone is converted to which of the following by the


liver?
A. Cortisone
B. Hydrocortisone
C. Prenisolone
D. Methylprednisolone

1789.
1167.

Which of the following is (are) true for water soluble


vitamins?
I. They have limited storage in the body
II. They may be toxic in dose about MDR
III. Most are eliminated by the kidneys

A.
B.
C.
D.

I only
II only
III only
I, II and III

1168.
1790.

The dose of a drug is 5 mg/kg of body weight. What dose


should be given to a 100lb female patient?
A. 2500 mg
B. 250 mg
C. 25 mg
D. 44 mg

1169.
1791.

How many grains of a drug are needed to make 4 fluid


ounces of a 5% (w/v) solution?
A. 91 grains
B. 96 grains

285
Pharmacy

C. 48 grains
D. 100 grains
1170.
1792.

An ointment is prepared by incorporating 10g of a drug into


100g of white petrolatum. What is the percentage w/w of
active ingredient?
A. 10.0%
B. 9.1%
C. 0.91%
D. 0.95%

1793.
1171.

What is the proof strength of a 50% (v/v) solution of


alcohol?
A. 25 proof
B. 50 proof
C. 100 proof
D. 75 proof

1794.
1172.

Certified dyes may not be used in the area of?


A. the eye
B. the scalp
C. the lips
D. the nose

1173.
1795.

The principal use of magnesium stearate in pharmaceutics is


as a(n)
A. lubricant
B. antacid
C. source of Mg ion
D. disintegrator

1174.
1796.

A synonym for vitamin C is


A. riboflavin
B. tocopherol
C. ascorbic acid
D. cyanocobalamin

1175.
1797.

Activated charcoal is used in some antidotes because of


which of its properties?
A. Neutralizing
B. Emetic
C. Absorptive
D. Adsorptive

1176.
1798.

What is the pH of a solution that has a hydrogen ion


concentration of 1x108?
A. 8
B. 6
C. 1.8
D. 1.6

286
Pharmacy

1177.
1799.

The major use of titanium dioxide in pharmacy is in?


A. sunscreens
B. antacid tablets
C. capsules as a diluent
D. effervescent salts

1800.
1178.

Which of the following is used as a hemostatic agent?


A. Heparin
B. Oxidised Cellulose
C. Coumadins
D. Indanediones

1179.
1801.

Another name for extended insulin zinc suspension is


A. NPH
B. lente
C. ultralente
D. regular

1802.
1180.

The Latin term oculus sinister can be translated as


A. right eye
B. left eye
C. both ears
D. both eyes

1181.
1803.

The recommended method of mixing insulins is to


A. shake vigorously
B. mix gently by rolling between palms of hands
C. add simultaneously to container
D. none of the above

1804.
1182.

Membrane filters with 0.22 microns pores can remove which


of the following when used to filter solutions?
A. bacteria
B. pyrogens
C. fungi
D. A & C

1805.
1183.

Which of the following is not used as a vehicle injections?


A. Peanut oil
B. Cottonseed oil
C. Corn oil
D. Theobroma oil

1806.
1184.

Which preservative has been removed from several vaccine


products to decrease the potential of toxicity events in
children?
A. Benzyl alcohol
B. Sodium bicarbonate
C. Thimerosal
D. Peanut oil

287
Pharmacy

1807.

1808.

1809.

1810.

1811.

288
Pharmacy

10 mL.
C. Amino Acids 15%, Insulin 5 units/liter, Electrolytes.
D. Dextrose 2.5%, Fat Emulsion 30%, Electrolytes
1190.
1812.

The most commonly used method to sterilize talc powder for


the treatment of pleural effusion is
A. gas sterilization
B. irradiation
C. low level heat
D. moist heat

1191.
1813.

Alfuzosin is used mainly to treat


A. gout
B. benign prostatic hypertrophy
C. Diabetes
D Carcinomas

1814.
1192.

Dosage of anticonvulsants is adjusted


A. when seizures occur frequently
B. every 2 weeks
C. every 2 years
D only when side effects are seen

1193.
1815.

Which of the following symptoms is not present in digitalis


intoxication
A. AV block
B. ventricular tachycardia
C. vomiting
D Vagal arrest of the heart

1194.
1816.

Sulfonmides are excreted free and combined as


A. the acetyl derivative
B. The amino derivative
C. The sulfate derivative
D. The glycine conjugate

1817.
1195.

Which of the following agents are associated with tinnitus as


a result of toxicity?
A. Salicylate
B. Phenytoin
C. Propranolol
D. Acetaminophen

1818.
1196.

Parkinsonism is probably due to


A. Too little dopamine in the brain
B. Too little levodopa in the brain
C. Too little acetylcholine in the brain
D. Too much levodopa in the brain
The greatest threat from morphine poisoning is
A. renal shutdown

1819.
1197.

289
Pharmacy

1198.
1820.

1821.
1199.

1822.
1200.

1823.
1201.

1202.
1824.

1203.
1825.

1204.
1826.

1827.
1205.

1828.
1206.

B. paralysis of spinal cord


C. respiratory depression
D. cardiovascular collapse
An endocrine glad that plays an important role in calcium
metabolism is the :
A. pancreas
B. hypophysis
C. thyroid
D. parathyroid
Which of the following is used to curtail chronic uric acid
stone formation
A. Allopurinol
B. Trimethoprim
C. Methenamine
D. Ethacrynic acid
Which of the following is used to lower blood lipid levels?
A. Trimethadione
B. Rosuvastatin
C. Flucytosine
D. Coumarin
Therapeutically, vitamin B 1 has been employed most
successfully in the treatment of
A. microcytic anemia
B. pellagra
C. scurvy
D. beriberi
Aprepitant is used to treat
A. Hypertension
B. muscular injuries
C. ulcers
D. chemotherapy induced emesis
Magnesium ion is necessary in
A. stimulating enzyme systems
B. muscular contraction
C. nerve conduction
D. all of the above
A class of plant alkaloids that is widely used to treat
migraine is
A. vinca alkaloids
B. digitalis glycosides
C. stramonium alkaloids
D. ergot alkaloids
Complications of enteral tube feedings include?
A. Aspiration pneumonia
B. Diarrhea
C. hyperglycemia
D. All of the above
Which of the following drugs is used to rescue patients who
have received high dose methotrexate ?
A. Allopurinol
B. Leucovorin

290
Pharmacy

1829.

1830.

1831.

1832.

1833.

1834.

1835.

1836.

291
Pharmacy

C. Patent ductus arteriosus


D. Limb deformation
1837.
1215.

Which of the following chemotherapy agents requires a


special restricted distribution program due to severe
teratogenicity effects?
A. Vincristine
B. Cisplatin
C. 5Fluorouracil
D. Thalidomide

1217.
1838.

The fat-soluble vitamins are

A.
B.
C.
D.
1218.
1839.

B complex, C, and D
A, D, E, and K
A, B, C, and D
B complex, E and K

Gluconeogenesis involves the formation of glucose from


A.
B.
C.
D.

noncarbohydrate sources
pentoses
aldohexoses
ketohexoses

1840.
1219.

Eletriptan was approved by the FDA is 2002 to treat which


of the following diseases?
A. Migraines
B. Muscle spasms
C. Parkinsons disease
D. Neuralgias

1841.
1220.

Primidone, used in the treatment of generalized tonic clonic


seizures, is metabolized to
A. Phenytoin
B. Phenobarbital
C. metformin
D. carbmazepine

1842.
1221.

Vincristine has been commonly associated with which of the


following adverse events?
A. Neurotoxicity
B. Gout
C. Duodenal ulcers
D. Blood clotting

1843.
1222.

Gemifloxacin is a (n)
A. adrenergic blocker
B. antiinfective
C. glucocorticoid
D. local anesthetic

292
Pharmacy

1223.
1844.

Botulinum toxin is FDA approved to treat


A. blepharospasms
B. cervical dystonias
C. facial wrinkles
D. A, B and C
Telithromycin is a member of which class of antiinfectives?
A. Cephalosporin
B. Penicillin
C. Aminoglycoside
D. Ketolid
Cholestyramine resin has the prevalent side effect of
A. blocking absorption of some vitamins
B. raising cholesterol levels
C. causing intoxication
D. increasing sensitivity to UV light
All of the following posses diuretic properties EXCEPT:
A. aminophylline
B. lanthanum
C. spironolactone
D. bumetanide
Which of the following is (are) true for tetracyclines?
I. They may cause yellowing of the teeth in adolescents
II. they are bacteiostatic
III. they exhibit a broad spectrum of activity
A. I only
B. III only
C. I and II only
D. I, II, and III
Adefovir is a member of which class of antiinfectives?
A. Cephalosporin
B. Penicillin
C. Aminoglycoside
D. Antiviral
Rasburicase is used to treat
A. hay fever
B. vertigo
C. drug allergies
D. hyperuricemia

1851.
1230.

Lorazepam produces which of the following action(s)?


A. Sedation
B. Loss of memory
C. Reduction of anxiety
D. All of the above

1852.
1231.

Celecoxib is used as a(n)


A. antimalarial
B. cardiotonic
C. antihistaminic
D. analgesic

1224.
1845.

1225.
1846.

1226.
1847.

1227.
1848.

1228.
1849.

1229.
1850.

293
Pharmacy

1853.
1232.

Tenoxicam is a:
A. diuretic
B. cardiotonic
C. selective betablocker
D. antiinflammatory agent
1233. Which of the following drugs has been associated with
1854.
Reyes syndrome in children?
A. Aspirin
B. Acetaminophen
C. Ibuprofen
D. Naproxen

1234.
1855.

1856.
1235.

1236.
1857.

1237.
1858.

1238.
1859.

1239.
1860.

1240.
1861.

1241.
1862.

Ultrashort acting barbiturates are used primarily as


A. sedatives
B. hypnotics
C. antispasmodic agents
D. preanesthetic agents
Norethindrone is a drug commonly used in
A. mixed estrogens
B. oral contraceptives
C. treating carcinomas
D. diagnostic testing
Some anticonvulsants (e.g., carbamazepine, Phenytoin,
gabapentin) are FDA approved to treat seizures but have also
been effective in the treatment of
A. parkinsonism
B. neuralgias or neuropathies
C. colitis
D. nausea
Voriconazole is classified as a(n)
A. muscle relaxant
B. sedativehypnotic
C. tranquilizer
D. antifungal
Treprostinil is used mainly to treat
A. gastritis
B. minor anxiety states
C. pulmonary arterial hypertension
D. severe pain
Histamine is found in the human body in
A. the granules of mast cells in blood
B. the mucosal layer of the GI tract
C. the hypothalamus
D. all of the above
Oxaliplatin is used as a(n)
A. narcotic antagonist
B. narcotic analgesic
C. antineoplastic
D. antiepileptic
Which of the following is not a side effect of codeine?
A. Miosis
B. Nausea

294
Pharmacy

1863.
1242.

1864.
1243.

1865.
1244.

1245.
1866.

1867.
1246.

1868.
1247.

1248.
1869.

1249.
1870.

1871.
1250.

C. Diarrhea
D. respiratory depression
Which of the following may be associated with hyperprolactinemia?
A. Galactorrhea
B. Prolactin deficiency
C. Use of antipsychotic drugs
D. Both A & C
Lidocaine is used as a local anesthetic or as a(n)?
A. general anesthetic
B. antipruritic
C. preanesthetic
D. antiarrhythmic
Cocaine has a long duration of local anesthetic action
because
A. it is more stable than most local anesthetics
B. it is readily absorbed
C. it is not biotransformed
D. it is a vasoconstrictor
All of the following are natural estrogens or congeners
except
A. estradiol
B. diethylstilbestrol
C. estrone
D. ethinyl estradiol
Which of the following is used to treat hyperthyroidism?
A. Liotrix
B. Thyroglobulin
C. Liothyronine
D. Propylthiouracil
Carbidopa is used to
A. treat Parkinsons disease
B. treat hypertension
C. potentiate levodopa
D. treat depression
Hyoscyamine has the same action as atropine but is
A. twice as potent
B. three times more potent
C. 10 times more potent
D. half as potent
Emtricitabine is used primarily as a(n)
A. cardiotonic
B. antidepressant
C. diuretic
D. antiviral
The principal toxic effect of heparin is
A. hemorrhage
B. bronchospasm
C. chills
D. fever

295
Pharmacy

1251.
1872.

1252.
1873.

1253.
1874.

1254.
1875.

1876.
1255.

1877.
1256.

1257.
1878.

1879.
1258.

1259.
1880.

1881.
1260.

Tenecteplase is used primarily to reduce mortality associated B


with which of the following clinical problems?
A. diabetes
B. myocardial infarction
C. hemorrhage
D. prostate cancer
Albuterol is usually administered by which route?
C
A. IV
B. IM
C. Nasal
D. Rectal
Omalizumab is used most commonly to treat
C
A. AIDS
B. genital herpes
C. asthma
D. CMV retinitis
Barbiturates, in general, are particularly noted for
B
A. lack of habituation
B. producing microsomal enzymes in liver
C. instability
D. slow absorption
Epinastine exerts its action because it is a(n)
A
A. antihistamine
B. MAO inhibitor
C. alkaline
D. calcium channel blocker
The principal mechanism of action of penicillins on microorganisms is A
A. inhibition of cell wall synthesis
B. destruction of the nucleus
C. bacteriostatic
D. causing mutation
Epinephrine is not given by the oral route because
D
A. it is inactivated in the gastric mucosa
B. local vasoconstriction inhibits absorption
C. it is rapidly metabolised
D. all of the above
Which of the following cardiotonic which is not from natural D
sources?
A. Digoxin
B. Ouabain
C. Digitoxin
D. Amrinone
Efalizumab is used to treat
A
A. Plaque psoriasis
B. Hodgkins disease
C. angina pectoris
D. breast cancer
Which of the following is classified as a cholinergic
C
antagonist?
A. Acetylcholine

296
Pharmacy

1261.
1882.

1262.
1883.

1263.
1884.

1264.
1885.

1265.
1886.

1887.
1266.

1267.
1888.

1889.
1268.

1269.
1890.

B. Neostigmine
C. Atropine
D. Bethanechol
The action of heparin is terminated by
A. coumarin
B. indanediones
C. insulins
D. protamine sulfate
Nitroglycerin has a relatively short halflife due to
A. its volatility
B. its chemical instability
C. its poor absorption
D. firstpass metabolism
Various opiates may be used as all of the following except
A. analgesics
B. antiinflammatories
C. antidiarrheals
D. aids to anesthesia
Trihexyphenidyl is used to treat
A. parkinsonism
B. angina
C. xerostomia
D. glaucoma
The prevalent mechanism of action of antihistaminics is that
they
A. prevent formation of histamine
B. speed up elimination of histamine
C. destroy histamine
D. competitively inhibit histamine
Isotretinoin is commonly used to treat
A. lice infestations
B. fungal infestations
C. ringworm
D. acne
Endorphins play a significant role in
A. pain modulation
B. absorption
C. diuresis
D. hypertension
The most effective substance in treating acute attacks of gout
is:
A. Allopurinol
B. Probenecid
C. Aspirin
D. Colchicine
Nystatin is used to treat
A. Trichomonas infections
B. Staphylococcus aureus infections
C. Candidiasis
D. Escherichia coli infection

297
Pharmacy

1891.
1270.

Which of the following NSAIDs is available in a parenteral


form?
A. Piroxicam
B. Ketorolac
C. Both A & B
D. Neither A nor B

PSA is considered a useful marker for carcinoma of:


A. prostate
B. breast
C. pancreas
D. stomach

All of the following have some effect on blood glucose


EXCEPT:
A. glucagon
B. epinephrine
C. thyroid hormone
D. androgens
Nateglinide stimulates the release of
A. insulin
B. epinephrine
C. glucose
D. glucagon
Somastatin is also known as:
A. growth hormone-release inhibitor
B. secretin
C. cholecystokinin
D. oxytocin
Revastigmine has the primary action of inhibiting
A. acetylcholinesterase
B. epinephrine
C. gastric acid
D. histamine release
Which of the following are low molecular weight heparins
A. Tinzaparin
B. Enoxaparin
C. Dalteparin
D. All of the above

1898.
1277.

Ocular timolol is used primarily in


A. glaucoma
B. pink eye
C. conjunctivitis
D. eye infections

1899.
1278.

What type of receptor antagonist is palonosetron?


A. 5HT 3
B. Pyridoxine
C. Cyanocobalamin
D. Ascorbic acid

1892.
1271.

1893.
1272.

1273.
1894.

1274.
1895.

1275.
1896.

1276.
1897.

298
Pharmacy

1279.
1900.

Lorazepam is classified as a(n)


A. loop diuretic
B. MAO inhibitor
C. thiazide diuretic
D. dibenzazepine
1280. Propantheline is contraindicated in patients with
1901.
A. glaucoma
B. myasthenia gravis
C. obstructive disease of GI tract
D. all of the above
1902.
1281. Antiepileptics as a group are noted for developing or causing
A. rashes
B. aterial tachycardias
C. tolerance
D. spasms
1282. Selenium sulfide is used
1903.
A. Orally
B. topically
C. by injection
D. by none of the above
1283. Memantine is used to treat
1904.
A. bipolar disorder
B. dementia of Alzheimers
C. seasonal allergies
D. gastric ulcers
1905.
1284. Which of the following is NOT a stimulant laxative?
A. Cascara sagrada
B. Senna
C. Castor oil
D. Magnesium sulfate
1285.
1906. Which of the following actions is not seen with
sympathommetics?
A. they constrict the pupil of the eye
B. they increase heart rate
C. they stimulate sweat glands
D. they dilate bronchioles
1907.
1286. Baclofen is used primarily as a(n)
A. sympathomimetic
B. antianxiety agent
C. muscle relaxant
D. antispasmodic
1287. Bortezomib is indicated to treat which of the following?
1908.
A. adult onset of diabetes
B. hypertension
C. cardiac insufficiency
D. refractory multiple myeloma
1288. A prevalent side effect of Propylthiouracil is
1909.
A. hearing loss
B. visual impairment
C. acidosis
D. leukopenia

299
Pharmacy

1910.
1289.

1290.
1911.

1291.
1912.

1913.
1292.

1293.
1914.

1915.
1294.

1295.
1916.

1917.
1296.

1918.
1297.

Tinidazole is used primarily as a(n)


A. diuretic
B. antiprotozoal
C. hypotonic
D. antifungal
The pharmacologic actions of sulindac include
I. antiinflammatory properties
II. analgesic properties
III. antipyretic properties
A. I only
B. II only
C. I and III only
D. I, II, and III
Atomoxetine is used to treat symptoms of
A. ADHD
B. parkinsonism
C. gout
D. petit mall epilepsy
Gatifloxacin is classified as a(n)
A. antifungal
B. antiviral
C. broad spectrum antibiotic
D. narrow spectrum antibiotic
Zidovudine may cause which of the following as a frequent
side effect?
A. Vasoconstriction
B. Dryness of the mouth
C. Hyperactivity
D. Anemia
Mupirocin ointment is used topically to treat
A. conjunctivitis
B. all types of eye infections
C. impetigo
D. carcinomas
Pentoxifylline acts primarily by
A. dissolving cholesterol
B. decreasing viscosity of the blood
C. increasing biotransformation
D. oxidative mechanisms
Which of the following is most useful in treating
hypertensive crisis?
A. Sodium nitroprusside
B. Reserpine
C. Chlorothiazide
D. Spironolactone
Allopurinol differs from most other agents used to treat
gouty conditions because it
A. does not decrease uric acid levels
B. prevents formation of uric acid
C. increases elimination of uric acid
D. causes rapid biotransformation of uric acid

300
Pharmacy

1919.
1298.

Estrogens tend to increase the risk of


A. endometrial carcinoma
B. hirsutism
C. hearing loss
D. visual problems

1920.
1299.

Which of the following drugs are contraindicated when used


with sildenafil?
A. Fluroquinolones
B. Organic nitrates
C. Penicillins
D. Calcium channel blockers
Gentamicin exhibits
A. significant hepatotoxicity
B. significant cardiotoxicity
C. significant dermal toxicity
D. significant nephrotoxicity
Aripiprazole is classified as a(n)
A. cardiosuppressant
B. antipsychotic
C. diuretic
D. gastric acid suppressor
Carbamazepine is used as an anticonvulsant and also to treat
pain from
A. kidney infections
B. burns
C. muscle injuries
D. trigeminal neuralgia
Methylphenidate is used to treat
A. fatigue
B. ADHD
C. anxiety
D. depression
Doxorubicin is used to treat
A. a wide variety of infections
B. gram positive infections
C. gram negative infections
D. carcinomas
A drug that is useful in treating potentially fatal systemic
fungal infections is
A. nystatin
B. propionic acid
C. amphotericin B
D. miconazole

The use of thyroid hormones in the treatment of obesity is


A. widely accepted
B. unjustified
C. acceptable in combination with other drugs
D. only acceptable if other treatments fail

1921.
1300.

1922.
1301.

1923.
1302.

1303.
1924.

1304.
1925.

1305.
1926.

1927.
1306.

301
Pharmacy

1928.
1307.

1929.
1308.

1930.
1309.

1310.
1931.

1932.
1311.

1933.
1312.

1934.
1313.

An advantage of dextromethorphan over codeine as an


antitussive is that it
A. it twice as effective
B. is more stable
C. has no side effects
D. products very little depression of the CNS
Nonselective alpha adrenoreceptor antagonists are used primarily in
A. peripheral vascular disorders
B. tachycardia
C. migraine headache
D. atherosclerosis
All of the following statements about propranolol are true
EXCEPT
A. the oral route of administration is preferred
B. propranolol penetrates into the CNS
C. propranolol is primarily biotransformed in the liver
D. it is a nonselective alpha antagonist
Antimuscarinic drugs are contraindicated in
A. narrow angle glaucoma
B. paralytic ileus
C. pyloric or intestinal obstruction
D. All of the above
Zanamivir and oseltamivir are both FDA approved for
A. treatment of influenza A infection
B. treatment of influenza B infection
C. prophylaxis of influenza infections
D. A and B
Which of the following drugs is not a proton pump
inhibitor?
A. Pantoprazole
B. Rabeprazole
C. Lansoprazole
D. Sulfisoxazole
A danger of prolonged use of pilocarpine salts as a miotic is
A. tearing
B. glaucoma
C. conjunctivitis
D. lens opacity

1935.
1314.

A serious side effect of furosemide in treating heart patients


is that it
A. interacts with digitalis glycosides
B. causes arterial blockage
C. may cause anuria
D. causes hypertension

1936.
1315.

All of the following are side effects of progestins except


A. weight gain
B. headache
C. fatigue
D. constipation

302
Pharmacy

1937.
1316.

1938.
1317.

1939.
1318.

1319.
1940.

1941.
1320.

1321.
1942.

1322.
1943.

1944.
1323.

1324.
1945.

1325.
1946.

Which of the following drugs require electrocardiographic


monitoring prior to initiation of therapy?
A. Ziprasidone
B. Arsenic trioxide
C. Bumetanide
D. A and B
Atazanavir is usually classified as a(n)
A. H 2 antagonist
B. ulcer protectant
C. antiviral
D. oral antidiabetic
Prolonged usage of sublingual nitrates is likely to cause
A. ulcers
B. anuria
C. rashes
D. development of tolerance
Streptokinase is used to
A. dissolve blood clots
B. treat digestive disorders
C. promote carbohydrate degradation
D. treat muscle injuries
Gold compounds have been used to treat
A. worm infestations
B. ulcers
C. kindly failure
D. rheumatoid arthritis
Tamoxifen is classified as a(n)
A. estrogen
B. antiestrogen
C. androsterone
D. testosterone
Bacitracin is not usually given parenterally because of its
A. insolubility
B. lack of stability
C. pain at the injection site
D. nephrotoxicity
Beriberi is associated with a deficiency of
A. vitamin D
B. thiamine
C. vitamin C
D. niacin
A drug used to treat delirium tremens is
A. chlordiazepoxide
B. haloperidol
C. disulfiram
D. methadone
Cromolyn sodium acts by
A. destroying histamine
B. releasing histamine
C. biotranforming histamine
D. preventing the release of histamine

303
Pharmacy

1947.
1326.

The antiinflammatory action of aspirin is due to


A. analgesia
B. inhibition of clotting
C. antipyretic effect
D. inhibition of prostaglandin synthesis

1948.
1327.

Castor oil is classified as which type of laxative


A. Lubricating
B. Anthraquinone
C. Irritant
D. Stool softening
Penicillamine is most commonly used to treat
A. Parkinsonism
B. Wilsons disease
C. Neoplasms
D. Raynauds disease
Which of the following drugs were withdrawn from the U.S.
market in 2004 due to increased risk of cardiovascular
events?
A. Rofecoxib
B. Cisapride
C. Dexfenfluramine
D. Terfenadine
Doxycycline oral gel is used to treat
A. periodontitis
B. aphthous ulcers
C. acne
D. lymes disease
Which of the following drugs require(s) a dosage adjustment
in patients with renal impairment?
A. Famotidine
B. Capecitabine
C. Gentamicin
D. All of the above
A prevalent side effect of Norethindrone is
A. diarrhea
B. breakthrough bleeding
C. blood dyscrasias
D. cardiac insufficiency
The preferred way to offset hypokalemia is to
A. eat citrus fruits
B. eat seafood or fish
C. administer potassium salt
D. administer IV electrolytes
Persons receiving MAO inhibitors should control their intake
of
A. some fermented food and beverages
B. carbohydrates
C. fats and lipids
D. water

1328.
1949.

1329.
1950.

1330.
1951.

1331.
1952.

1953.
1332.

1333.
1954.

1955.
1334.

304
Pharmacy

1956.
1335.

1957.
1336.

1337.
1958.

1338.
1959.

1960.
1339.

1961.
1340.

Which of the following drugs should not be administered


with high fat meals?
I. Indinavir
II. Riluzole
III. Stavudine
A. I only
B. III only
C. I and II only
D. II and III only
Drugs that may inhibit the metabolism of ziprasidone include
which of the following?
I. Erythromycin
II. Ketoconazole
III. Phenobarbitol
A. I only
B. III only
C. I and II only
D. II and III only
Flumazenil is a specific antagonist for which of the
following drugs?
I. Meperidine
II. Propoxyphene
III. Diazepam
A. I only
B. III only
C. I and II only
D. II and III only
Which of the following antidepressants also has an
indication for smoking cessation?
A. Haloperidol
B. Bupropion
C. Citalopram
D. Mirtazapine

Deficiency of antidiuretic hormone in the body causes:


A. kidney failure
B. diabetes insipidus
C. diabetes mellitus
D. kidney blockage
Which of the following statements are true regarding the
administration of alendronate?
I. take more than 30 minutes before first food or beverage of
the day
II. take with 6 to 8 ounces of plain water
III. remain fully upright (sitting or standing) for at least 30
minutes
A. I only
B. III only
C. I, II, only
D. All statements

305
Pharmacy

1962.
1341.

Testosterone is available as which of the following dosage


forms?
I. Ointment
II. Injection
III. Transdermal patch
A. I only
B. III only
C. I and II only
D. I , II, III

1963.
1342.

Which of the following is an oral hypoglycemic agents?


A. Ketorolac
B. Pioglitazone
C. Doconazole
D. Riluzole

1964.
1343.

The concurrent administration (within 12 hours of dosing) of


nisoldipine should be avoided with which of the following
substances?
I. highfat meal
II. Grapefruit juice
III. Cocoa
A. I only
B. III only
C. I and II only
D. I , II, III

1965.
1344.

Which of the following drugs may be administered without


regard to meals?
I. Valacyclovir
II. Cetirizine
III. Acarbose
A. I only
B. III only
C. I and II only
D. I , II, III

1966.
1345.

Dexfenfluramine, an agent approved for the treatment


obesity, was removed from the U.S. market due to
A. cardiac valvular dysfunction
B. gastric ulcerations
C. renal toxicity
D. ototoxicity

1967.
1346.

The terminal half life of Alendronate


A. 10 days
B. 10 months
C. 10 years
D. 10 hours

306
Pharmacy

1347.
1968.

Serotonin syndrome is characterized by which of the


following symptoms
I. Mental status changes
II. Tremor
III. Diaphoresis
A. I only
B. III only
C. I and II only
D. I , II, and III

1348.
1969.

The FDA has recommended the removal of which ingredient


found in cough/cold products due to an association with
hemorrhagic stroke?
A. Dextromethorphan
B. Pseudoephedrine
C. Guaifenisen
D. Phenylpropanolamine

1970.
1349.

Which of the following antacids should not be used in


dialysis patients?
I. Aluminum hydroxide
II. Magnesium hydroxide
III. Calcium carbonate
A. I only
B. III only
C. I and II only
D. I , II and III

1971.
1350.

Melatonin regulates which of the following functions?


I. sleep
II. Circadian rhythms
III. Respiration
A. I only
B. III only
C. I and II only
D. I , II and III

1351.
1972.

Tarcolimus ointment is used primarily to treat:


A. pain
B. atopic dermatitis
C. sunburns
D. skin grafts

1973.
1352.

In 2002, which class of drugs experienced significant


revisions in the warnings section of the product labeling
regarding suicidal ideation and possible increased risk of
suicide?
A. SSRIs
B. Antihistamines
C. Antivirals
D. Fluoroquinolones

307
Pharmacy

1974.
1353.

Which of the following drug combinations has (have) been


effective therapy for the treatment of Helicobacter pylori?

I. Clarithromycin/ranitidine bismuth citrate


II. Omeprazole/ clarithromycin
III. Ciprofloxacin/erythromycin
A.
B.
C.
D.

1975.
1354.

I only
III only
I and II only
II and III only

Venlafaxine, an agent used to treat:


A.
B.
C.
D.

vomiting
depression
schizophrenia
none of the above

1976.
1356.

Cetirizine is a
A. H1 agonist
B. H2 antagonist
C. H1 antagonist
D. None of the above

1977.
1357.

Ezetimibe is considered a(n)


A. antihyperlipidemic
B. antiinfective
C. NSAID
D. antiviral

1978.
1360.

Which of the following statements regarding fosphenytoin


are true?

I. fosphenytoin is completely converted to Phenytoin after


intravenous or intramuscular administration
II. 1 mmol of fosphenytoin administrated produces 1 mmol
of Phenytoin
III. Monitoring serum Phenytoin levels is not necessary
A.
B.
C.
D.

I only
III only
I and II only
II and III only

308
Pharmacy

1979.
1361.

Sumatriptans efficacy in migraine therapy is attributed to


which of the following mechanisms?
I. Selective 5HT (1) agonist
II. Nonspecific 5HT agonist
III. Dopamine agonist
A.
B.
C.
D.

I only
III only
I and II only
II and III only

1362.
1980.

The principal adverse effect of acarbose is


A. hematologic
B. gastrointestinal
C. renal
D. hepatic

1981.
1363.

Which one of the following drugs is categorized as controlled B


non psychotropic drug?
A. Digoxin
B. Isotretinoin
C. Norfloxacin
D. Methenamine

1982.
1364.

Ketorolac therapy is limited to 5 days of


therapy due to an increased risk of
A. renal impairment
B. gastrointestinal bleeding/performation
C. liver failure
D. A and B

1983.
1365.

Which of the following HMGCoA reductase inhibitors


causes the greatest percentage increase in HDL?
A. Simvastatin
B. Pravastatin
C. Lovastatin
D. Fluvastatin

1984.
1367.

Which of the following statements regarding low molecular


weight heparins (LMWHs) is (are) true?
I. Predominantly inhibits thrombin
II. Does not require Activated Partial Thromboplastin time monitoring
III. has a stronger affinity than conventional heparin for
factor II

A.
B.
C.
D.

I only
III only
I and II only
II and III only

309
Pharmacy

1985.
1368.

Which of the following statements is true regarding


indinavir?
I. It is not commonly used in therapy
II. It competitively inhibits the HIV protease enzyme
III. It is a reverse transcriptase inhibitor
A.
B.
C.
D.

I only
III only
I and II only
II and III only

1986.
1369.

Diuretics tend to enhance lithium salt toxicity due to


A. sodium depletion
B. potassium depletion
C. direct drug interaction
D. increased absorption

1987.
1370.

Tadalafil is used to treat


A. Crohns disease
B. erectile dysfunction
C. psoriasis
D. photoallergic reaction

1371.
1988.

Sumatriptan use is contraindicated in patients


I. with a history of ischemic heart disease
II. taking phenelzine
III. with uncontrolled hypertension

A.
B.
C.
D.

I only
III only
I and II only
I, II, and III

1372.
1989.

Which type of cancer is Gefitinib approved to treat?


A. Colon cancer
B. Non small cell lung cancer
C. Pancreatic cancer
D. Breast cancer

1990.
1373.

Finasterides action in the treatment of benign prostatic


hypertrophy is as a(n)
A. HMGCoA reductase inhibitor
B. alpha 5 reductase inhibitor
C. serotonin reuptake
D. P450 hepatic enzyme inhibitor

1374.
1991.

Agent used in the treatment of bladder incontinence is


A. nalmefence
B. tolterodine
C. tolcapone
D. valrubicin

310
Pharmacy

1992.
1375.

Which of the following agents are approved for treatment of


narcolepsy:
I. Modafinil
II. Methylphenidate
III. Dextroamphetamine
A.
B.
C.
D.

I only
III only
I and II only
I, II, and III

1376.
1993.

Which of the following drugs, used to treat erectile


dysfunction, has the longest halflife?
A. Sildenafil
B. Tadalifil
C. Vardenafil
D. Alprostadil

1994.
1377.

The combination of use of metformin and iodinated contrast


media are contraindicated as it increase the risk of
A. hypertension
B. CNS depression
C. allergic reaction
D. lactic acidosis

1378.
1995.

Acarbose, a hypoglycemic agent, exerts which of the


following mechanisms of action?
A. increase insulin secretion
B. increase insulin receptor sensitivity
C. decreases circulating insulin antibodies
D. prevents absorption of carbohydrates

1379.
1996.

Which of the following drugs interferes with the metabolism


of indinavir?
A. Ketoconazole
B. Ciprofloxacin
C. Digoxin
D. Lorazepam

1997.
1380.

Which of the following adverse drug effects have been


associated with Isotretinoin use?
I. Skin peeling
II. Teratogenicity
III. Hypertriglyceridemia

1998.
1381.

A. I only
B. III only
C. I and II only
D. I, II, and III
How does rifampin decrease the efficacy of certain oral
contraceptives (OC)?
A. Increase OC renal clearance
B. Increase OC metabolism via hepatic enzyme induction

311
Pharmacy

1382.
1999.

1383.
2000.

1384.
2001.

1385.
2002.

1386.
2003.

1387.
2004.

2005.
1388.

C. Reduces OC systemic circulation via protein binding


D. Decreases intestinal absorption of OC
Fexofendine is a
C
A. metabolite of loratadine
B. metabolite of astemizole
C. metabolite of terfenadine
D. metabolic of hydroxyzine
Levetiracetam is considered a(n)
C
A. antianxiety agent
B. benzodiazepine
C. anticonvulsant
D. antihistamine
The mechanism of action responsible for dimercapto succinic C
acids efficacy in the treatment of lead poisoning:
I. increase the renal excretion of lead
II. chelates lead into water soluble complexes
III. promotes lead degradation to nontoxic metabolites
A. I only
B. III only
C. I and II only
D. I, II, and III
The advantage of transdermal estrogens as compared to oral
estrogens when used for estrogen replacement therapy
include
I. less frequent dosing
II. increased cardioprotection
III. decreased incidence of thrombolic events
A. I only
B. III only
C. I and II only
D. I, II, and III
Mirtazapine is used in:
A. General anesthesia
B. Local anesthetic preparations
C. Pain
D. Migraine
Which of the following activities are responsible for
prominent sedative effects?
I. H1 receptor antagonism
II. Increased melatonin secretion
III. Dopamine receptor antagonism
A. I only
B. III only
C. I and II only
D. I, II, and III
A combination of an antiretroviral and a protease inhibitor
may result in which of the following actions?
I. sustain reduction in viral load

312
Pharmacy

II. decrease the development of resistance


III. reduce the incidence of opportunistic infections

1389.
2006.

2007.
1390.

1391.
2008.

1392.
2009.

1393.
2010.

1394.
2011.

1395.
2012.

A. I only
B. III only
C. I and II only
D. I, II, and III
Naratriptan is useful in the treatment of acute migraine via
its action as a
A. serotonin agonist
B. serotonin antagonist
C. histamine antagonist
D. histamine inhibitor
Which of the following drugs has an FDA pregnancy
category X rating?
A. Lorazepam
B. Warfarin
C. Penicillin
D. Isotretinoin
Cystic fibrosis patients routinely need supplementation with
A. Fat soluble vitamins
B. carbohydrates
C. mannitol
D. water soluble vitamins
Immune globulin is administrated parenterally to provide
passive immunity to patients with which of the following
disease states?
A. Bone marrow transplant
B. Idiopathic thrombocytopenic purpura
C. AIDS
D. All of the above
There are many preparations of salbutamol available in the
market for the use in asthma and other conditions of reversible
airways obstructions and premature labour. Which of the
following are the route of administration of salbutamol?
A. By mouth
B. Bu subcutaneous
C. Intramuscular
D. All of the above
All of the following are effective in the treatment of acute
gouty arthritis EXCEPT:
A. colchicine
B. indomethacin
C. corticosteroids
D. Allopurinol
Signs and symptoms of theophylline toxicity include
A. sinus tachycardia
B. insomnia
C. seizures
D. all of the above

313
Pharmacy

1396.
2013.

1397.
2014.

2015.
1398.

1399.
2016.

2017.
1400.

2018.
1401.

2019.
1402.

1403.
2020.

2021.
1404.

Insulin resistance is often due to


A. excessive exercise
B. upper body obesity
C. both A & B
D. neither A nor B
Side effects associated with oral iron therapy may include
A. Nausea
B. diarrhea or constipation
C. dark stools
D. all of the above
Diarrhea is a side effect commonly associated with which of
the following drugs?
A. Codeine
B. Quinidine
C. Procainamide
D. Acetaminophen
Which of the following can be used to treat constipation or
chronic, watery diarrhea?
A. Hyperosmolar laxative
B. Bulk forming laxative
C. Saline laxative
D. Stimulant laxative
Ergot alkaloids are contraindicated for use in patients with
A. venous insufficiency
B. ischemic heart disease
C. sepsis
D. all of the above
Which of the following erythromycin salts has been
associated with cholestasis?
A. Ethylsuccinate
B. Gluceptate
C. Lactobionate
D. Estolate
Anticholinergic agents may aggravate which of the following
A. Narrow angle glaucoma
B. GI obstruction
C. Genitourinary tract disease
D. all of the above
Side effects of heparin include
A. Thrombocytopenia
B. gingivitis
C. glaucoma
D. hyperglycemia
Which of the following drugs may interact with digoxin?
A. Quinidine
B. Antacids
C. Cholestyramine
D. A, B, and C

314
Pharmacy

1550. (A) absorption of amphotericin B from the GI tract


2022.
(B) absorption of flucytosine from the GI tract
A.
A>B
B.
B>A
C.
A=B

1551. Which of the following antifungal agents would be only


2023.
given orally for the treatment of dermatomycosis in a 31year-old man?
A.
Amphotericin B
B.
Flucytosine
C.
Griseofulvin
D.
Miconazole

1552. A drug used for the oral treatment of dermatophytosis of


2024.
toe nails, finger nails and hair bulb is:
A.
fluconazole
B.
griseofulvin
C.
ketoconazole
D.
nystatin

2025.
1553. Which of the following oral medications could be used in a
31-year-old woman to treat vulvovaginal candidiasis?
A.
Butoconazole
B.
Fluconazole
C.
Amphotericin B
D.
Terconazole

2026.
1554. Which of the following are clinical uses of metronidazole?
A. Trichomoniasis
B. Giardia lamblia
C. Intestinal amebiasis
D. All of the above

2027.
1555. What are the side effects of metronidazole?
A. Disulfiram like effect
B. Metallic taste
C. Anorexia
D. All of the above
1556. Ferrous sulfate is valuable for the treatmentof which of the following
2028.
disorders in a 26 year old pregnant woman?
A. Pernicious anemia
B. Macrocytic anemia
C. Hypochromic anemia
D. Trifacial neuralgia

1557. When iron is ingested:


2029.
A. only that portion in the form of hemoglobin is absorbed
B. as inorganic iron salts, it must be reduced to the
divalent (ferrous) form before absorption
C. about 50 percent of the amount ingested is absorbed
D. a smaller percentage of the dose is absorbed by the
anemic individual

315
Pharmacy

2030.
1635. Which one of the following is LEAST likely to be seen in a person
taking glucocorticoids?
A. osteoporosis
B. hyperglycemia
C. mood depression
D. electrolyte imbalance

2031.
1636. Which one of the following has the strongest sodium-retaining
activity?

A.
B.
C.
D.

Cortisol
Cortisone
Oprednisolone
Corticosterone

2032.
1637. Which one of the following has the most potent anti-inflammatory
effect?
A. Corticosterone
B. Desoxycorticosterone
C. Both A & B
D. Neither A nor B

1638. Alternate day glucocorticoid therapy (using twice the daily


2033.
maintenance dose every other day) offers which of the following
advantages over daily steroid therapy?
A. Minimizing hypothalamic-pituitary adrenal axis
suppression

B. Decreased tendency to cushingoid state


C. Can permit normal growth in children dependent on
steroids
D. All of the above
2034.
1639. Dexamethasone is used for:
A. suppression of inflammatory disorders
B. diagnosis of Cushings disease
C. Both A & B
D. Neither A nor B

1640. In acute care management, a single large dose of glucocorticoid.


2035.
A. is virtually without adverse effects in the absence of
specific contraindications
B. may be life saving in acute cerebral edema and possibly
beneficial in septic shock
C. may mask symptoms important in accurate diagnosis

D. All of the above

You might also like